You are on page 1of 94

EXAMPLE 22 LEFT (to be done)

Chapter 6 & 6: Capital Budgeting:


Concept and Techniques
Meaning and introduction
A corporate entity exists to serve some goals. Its objectives are defined in its mission statement.
Some companies thrive to develop new products, some target new services and some other may
have the desire to provide quality products to its customers. The mission of the business is carried
out by developing various goods and services. The production of goods and services require plant
and machinery, tools, technologies, workers, raw material and other productive assets. The assets of
the company consist of long-term assets like plant and machinery, land and building. It also includes
some short-term assets like stocks of raw material and so on. In order to carry out these goals,
companies make proper planning to smoothly run its operations. One of the major functions of the
firm which has garnered significant attention is the management of its financial matters. This
module focuses on investment decision of the corporate entity. The investment decision of long-
term assets of the firm is called capital budgeting decision which is the subject matter of this
module. The short-term assets are managed under working capital decision and this topic is
discussed in a separate module. Thus, the capital budgeting decision is the long-term investment
decision of the firm.
The investment in fixed assets is called Capital Budgeting decision because expenses incurred on
fixed assets are capital expenditure in nature and there is need of planning for such expenses
(Planning means budgeting also).

Nature/features of investment decisions


Capital Budgeting has certain peculiar features which are as follows—
(a) It is the long-term investment plan of the firm whose payoff structure spreads over a period
of time. This period is usually more than a year.
(b) It involves a pattern of cash flows over the years. Initially funds are tied up in purchasing
equipment and later on it is followed by the generation of revenue for future years.
(c) The element of risk is involved in the investment as the funds are employed immediately;
however, the return is anticipated to occur in future time period. As future is uncertain, we
cannot be certain about the realization of our expected return.
(d) It involves sizable amount of funds which makes it costly to rectify if planning does not work
properly. The capital budgeting decisions are very crucial for the success of the business.
(e) Risky decisions
(f) Irreversible decisions
A simple example of capital budgeting decisions may be the decision to open a new retail outlet in a
city by Reliance fresh. The decision to start a new manufacturing unit in India by Audi is also an
example of capital budgeting decision. The desire of Audi is to stay in business for long period of
time involves substantial financial commitment on its part which is enough to qualify it as capital
budgeting decision.
Features of capital budgeting decisions
1. Long-term capital expenditures
2. Investment in long-term assets (fixed assets)
3. Huge investment
4. Risky decisions
5. Irreversible decisions
6. long-term effects
7. Evaluation is necessary
8. Profits accruing from the project are scattered over several years

Importance of Capital Budgeting Decisions


The capital budgeting decision is one of the most important decisions taken by the business entity.
The main reasons for its importance are—
Chapter 5 and 6, Capital Budgeting: 1
1. The survival of the firm depends on choosing the right kind of project. Therefore, proper
care should be taken to ensure that the beneficial projects are taken by using capital
budgeting analysis.
2. Once the survival of the firm is secured, the next task is to make efforts to achieve consistent
growth. The growth comes from making profitable investment decision which is the subject
matter of capital budgeting decision.
3. The long-term decisions are risky decisions due to element of uncertainty regarding future
events.
4. The amount of funds involved is very high. The decision once taken cannot simply be
reversed without incurring huge costs.

Types of Investment Decisions


There are various types of investment decisions which are taken by the firms. They can be simply
starting a new business or expansion and diversification of existing business. The main investment
decisions are as follows—
1. Setting up a new business which requires land and building, plant and machinery and so on.
2. The existing corporate entities may also take any of the following investment decision in the
course of running of the business—
a. Expansion: It refers to widening the production capacity of a firm. Suppose Apple is
manufacturing 1,00,000 units of its i-phone each year with its given capacity. In case of
increase in demand or its expectation of increasing its sales, it decides to manufacture
2,00,000 units of cell phones. This is the example of expansion which requires more
capacity and new machinery and equipment to deal with increased production decision.
b. Diversification: It means entering into new areas of business to increase its products
base. Suppose a telecom operator such as Airtel which provides its telecommunication
services, now decides to manufacture cell phones. This will be termed as diversification
and is taken by using capital budgeting techniques.
c. Replacement: In case of a going concern, it is natural that some of its equipment will
wear out as the time proceeds. In order to maintain the production capacity, firm needs
to replace those assets with new ones. This replacement decision is the most common
investment decisions of the firms.
d. Modernization: The use of new technology and instruments to provide good and
services is known as modernization. The use of computers instead of typewriters is the
simple example of modernization.
The investment decision can also be classified on the basis of their manner of selection. This
categorization is as follows—
1. Independent investment decision: Suppose a firm plans to manufacture tables and chairs.
It assesses its costs and benefits. The decision is taken on the basis of its profitability
without considering any other option. This kind of investment decision is independent
decision which is taken on its merit.
2. Mutually exclusive decision: Another class of decision is mutually exclusive. Under this
situation, there are various ways available for using given resources. For a student, whether
to attend the lecture or watch a movie is a mutually exclusive decision. The selection of one
alternative automatically involves rejection of other alternative or alternatives. Similarly, in
case of firms there are alternative ways available for producing goods and services. For
example, suppose there are three machines A, B and C are available to produce any goods.
The selection of one machine will result in rejection of other machines. This type of decision
where selection of one alternative results in rejection of rest of the available options is
called mutually exclusive decisions.
3. Contingent investment decision: Investment decisions which are contingent on
happening of any other event or decision are known as contingent investment decisions. For
Example, when a project is taken by the firm in remote areas, there is a need to build roads,
medical house and other infrastructure to support the production process. These additional
investment plans are contingent on taking the investment decision in remote area. Thus,
these decisions are contingent investment decisions.

Chapter 5 and 6, Capital Budgeting: 2


Investment Decision Criteria
The main process of taking investment decision is a three-step process—
1. The cash flows associated with any project are forecasted. It involves a series of cash
flows over a period of time. Initially when an investment is made, outflow of cash takes
place and then it is followed by the generation of revenue from the project’s activities. Thus,
the first step in capital budgeting decision is to estimate the cash flows associated with that
decision.
2. The cash flows for various years are not directly comparable. For example, suppose
there is a project which costs ₹1,00,000 and it would generate the net cash flows of ₹15,000,
₹20,000, ₹22,000, ₹2,50,000 and ₹30,000 for the five year’s life of the project. The cash
flows which will be received in future cannot be just added to compute the total amount of
revenue which is estimated to be earned by the project. The reason for this lies in the
concept of time value of money. This concept states that ₹100 received today is not the same
as ₹100 to be received after a year. The value of ₹100 to be received in future is less than the
value of ₹100 to be received today. In order to compute the value of ₹100 which is to be
received after one year now, we need to discount it at appropriate rate of interest. Similarly,
in order to make the money received in future time period comparable, we need to discount
these sums at the appropriate discount rate. This relevant discount rate is known as
weighted average cost of capital or overall cost of capital. In the present module, we will
assume it as given as this topic is discussed in great detail in some other module.
3. Once we compute the relevant cash flows and we have appropriate discount rate at
our disposal to compute their present values, we can proceed to use various capital
budgeting techniques to make investment decision. The most common and easiest way
to make investment decision is to compare the investment or the cost of the project with the
present value of the cash flows to be generated by the project.
Suppose in above example, the present value of all the cash inflows for the five years is
₹1,20,000. Under this situation, the benefits in present value terms exceed the cost of
undertaking the project which is ₹1,00,000. More precisely, the net amount generated by
the project is ₹20,000, it is also known as net present value (NPV). Thus, a project is
accepted when it has positive NPV, otherwise it is rejected. There are other capital
budgeting techniques also which are discussed in other modules.

Chapter 5 and 6, Capital Budgeting: 3


Evaluation techniques/methods
Basically, there are three methods to evaluate a project or fixed assets:
Traditional methods/Non- Modern Social cost/benefit methods
discounted cash flow methods/Sophisticated
techniques techniques/Discounted
cash flow techniques
1. Payback Period Method 1. Net Present Value 1. Economic Rate of
(PBP) Method (NPV) Return Method
2. Average/Accounting 2. Profitability 2. Social Rate of Return
Rate of Return Method Index/Benefit to Cost
(ARR) Ratio Method (PI/BC)
3. Internal Rate of
Return Method (IRR)
4. Modified Internal Rate
EVA (Economic Value
of Return Method
(MIRR) Added)=
5. Terminal Value After Tax Operating Profit
Method (TV) of Firm−[Capital
6. Discounted Payback Employed× 𝑘𝑜]
Period Method (DPBP)

Performa Income Statement


Sales ***** Profit *****
Less: Variable operating cost (*****) Less: Depreciation NON-CASH EXPENSE (*****)
Contribution ***** Profit before tax *****
Less: Fixed operating cost (*****) Less: Tax (*****)
Earnings before interest and taxes ***** Net Profit *****
Less: Interest (*****) Add: Depreciation (*****)
Earnings before taxes ***** Cash Flows ******
Less: Tax (*****)
Earnings after taxes or net income ******

Calculation of different cash flows


Initial cash out flows
Cost of new plant *****
+ Installation expenses *****
+ Other expenses *****
+ Additional working capital *****
+ Tax liability on account of capital gain on sales of old plant (if any) *****
– Tax benefit on account of capital loss on sales of old plant (if any) (*****)
Initial cash out flows ******

Subsequent cash inflows (annual)


Profits after tax *****
+ Depreciation *****
+ Financial charge (1–t) i.e., Interest (1–t) (If interest already have been subtracted) *****
– Repairs (if any) (*****)
– Capital expenditure (if any) (*****)
Subsequent (annual) cash inflows *****

Chapter 5 and 6, Capital Budgeting: 4


IMPORTANT
Terminal cash inflows
Annual cash inflows *****
+ Working capital released *****
+ Scrap value of the proposal (if any) *****
– Tax liability on account of capital gain on sales of plant/asset (if any) (*****)
+ Tax benefit on account of capital loss on sales of plant/asset (if any) *****
Terminal cash inflows *****

Formula to compute the Depreciation under straight line method


𝐼𝑛𝑖𝑡𝑖𝑎𝑙 𝑖𝑛𝑣𝑒𝑠𝑡𝑚𝑒𝑛𝑡 𝑜𝑟 𝑜𝑢𝑡𝑙𝑎𝑦 + 𝐼𝑛𝑠𝑎𝑙𝑙𝑎𝑡𝑖𝑜𝑛 𝑎𝑛𝑑 𝑜𝑡ℎ𝑒𝑟 𝑒𝑥𝑝𝑒𝑛𝑠𝑒𝑠 − 𝑆𝑎𝑙𝑣𝑎𝑔𝑒 𝑣𝑎𝑙𝑢𝑒
=
𝐸𝑠𝑡𝑖𝑚𝑎𝑡𝑒𝑑 𝑙𝑖𝑓𝑒 𝑜𝑓 𝑡ℎ𝑒 𝑝𝑟𝑜𝑗𝑒𝑐𝑡
(Working capital is not added to the cost of project (Initial Outlay) to compute the deprecation)
Notes:
1. While computing the cash flows it is to be checked whether the investment decision is a
single asset decision or a replacement/modernization decision. In case of single asset, the
computation of initial cash flows is simple but in case of replacement decision one has to
adjust the initial cash flows for the tax saving on capital loss (It is deducted from the initial
cash outflows) on sales of old plant or tax liability on capital gain (It is added to the initial
cash outflows) on sales of old plant.
2. While computing the terminal cash flows it is to be checked whether or not there are capital
gains or losses at the time of sales of the plant. In case there are capital losses then add the
tax saving should be added to cash inflows and in case there are capital gains the tax liability
should be deducted from cash inflows.
3. How to compute the capital gain or loss on sales of old plant or sales of the new plant after
maturity period—
Capital Gain - 𝐼𝑓 𝑠𝑎𝑙𝑒𝑠/𝑠𝑐𝑟𝑎𝑝 𝑣𝑎𝑙𝑢𝑒 > 𝑏𝑜𝑜𝑘 𝑣𝑎𝑙𝑢𝑒
Capital loss - 𝐼𝑓 𝑠𝑎𝑙𝑒𝑠/𝑠𝑐𝑟𝑎𝑝 𝑣𝑎𝑙𝑢𝑒 < 𝑏𝑜𝑜𝑘 𝑣𝑎𝑙𝑢𝑒
No loss/gain - 𝐼𝑓 𝑠𝑎𝑙𝑒𝑠/𝑠𝑐𝑟𝑎𝑝 𝑣𝑎𝑙𝑢𝑒 = 𝑏𝑜𝑜𝑘 𝑣𝑎𝑙𝑢𝑒
4. In case of replacement decision, we carry out the incremental analysis but in case of single
assets it is not so. Mean to say–we take incremental revenue, incremental depreciation, etc.
5. In case the method of depreciation is WDV then the computation of cash flows will be
carried out separately for each year.
6. 𝐿𝑎𝑠𝑡 𝑌𝑒𝑎𝑟’𝑠 𝐶𝑎𝑠ℎ 𝐹𝑙𝑜𝑤𝑠 𝑜𝑟 𝑇𝑒𝑟𝑚𝑖𝑛𝑎𝑙 𝑐𝑎𝑠ℎ 𝑖𝑛𝑓𝑙𝑜𝑤𝑠 = 𝐴𝑛𝑛𝑢𝑎𝑙 𝑐𝑎𝑠ℎ 𝐹𝑙𝑜𝑤𝑠 +
𝑊𝑜𝑟𝑘𝑖𝑛𝑔 𝑐𝑎𝑝𝑡𝑖𝑎𝑙 𝑟𝑒𝑙𝑒𝑎𝑠𝑒𝑑 + 𝑆𝑐𝑟𝑎𝑝 𝑣𝑎𝑙𝑢𝑒 𝑜𝑓 𝑡ℎ𝑒 𝑝𝑟𝑜𝑝𝑜𝑠𝑎𝑙 (𝑖𝑓 𝑎𝑛𝑦) −
𝑇𝑎𝑥 𝑙𝑖𝑎𝑏𝑖𝑙𝑖𝑡𝑦 𝑜𝑛 𝑐𝑎𝑝𝑖𝑡𝑎𝑙 𝑔𝑎𝑖𝑛𝑠 𝑜𝑛 𝑠𝑎𝑙𝑒𝑠 𝑜𝑓 𝑎𝑠𝑠𝑒𝑡 (𝑖𝑓 𝑎𝑛𝑦) +
𝑇𝑎𝑥 𝑠𝑎𝑣𝑖𝑛𝑔𝑠 𝑜𝑛 𝑐𝑎𝑝𝑖𝑡𝑎𝑙 𝑙𝑜𝑠𝑠𝑒𝑠 𝑜𝑛 𝑠𝑎𝑙𝑒𝑠 𝑜𝑓 𝑎𝑠𝑠𝑒𝑡 (𝑖𝑓 𝑎𝑛𝑦)

Additional aspects of capital budgeting decisions


These additional factors can be easily incorporated into the computation of cash flows associated
with the investment proposals—
1. Sunk costs: Sunk cost is the cost which is incurred in the past and cannot be changed now.
It is the historical expenses which has already taken place. Suppose a firm purchased a land
ten years ago and the land is not used for any purpose and lying vacant. The cost of the land
should not form the part of the new project, when the land is used for that project because
cost has already incurred and if we deduct this cost from the new project, it may make the
project unnecessarily less profitable.
2. Opportunity costs: It is the benefit from the next best alternative. Suppose this land has
alternate use that it can be given on rent. This amount of rent is the loss in revenue which
results when we use the land for the new project and this rent is not earned. As this loss in
revenue is the result of taking up the new project, it should be deducted from the revenues
generated by the new project. Thus, all opportunity costs should be the part of cash flow
estimation analysis.
3. Allocated overheads: A business concern usually has some fixed costs which cannot be
assigned to any specific business unit. These are the costs which will be incurred
irrespective of the production process. When a new machine is purchased or a new project
is undertaken, these should be ignored and not be the part of cash flow analysis. For

Chapter 5 and 6, Capital Budgeting: 5


example, suppose the allocated overheads are of ₹90,000 and company has nine machines.
This cost is allocated to these costs equally and each machine is charged ₹10,000. Suppose
now firm purchases a new machine and the number of machines increases to ten. Further, it
is stated that the amount of ₹9000 (90,000/10) would be charged to each machine. Now the
question before us is whether this amount of ₹9,000 should be taken into consideration
while computing cash flows of new machine? These allocated overheads would be incurred
whether we purchase new machine or not. Hence, it should not be taken into consideration
while estimating the cash flows of the new machine. However, if it is stated that if we take
up new machine, the amount of overheads will increase to, let’s say, ₹1,05,000. Under this
situation, this additional amount of ₹15,000 (1,05,000-90,000) is taken as the cash outflow
due to new machine.
4. Marketing survey: When a firm undertakes a market survey before launching a new
product, it involves time and cost. At the time of assessing the cash flows of the new project,
this marketing survey expense should not be the part of cash flow analysis. This expense is
incurred in the past and cannot be changed whether we take up the new project or not.
5. Financing aspect of Investment Decisions: In terms of computation of net cash flows, it
means that we should not take into consideration the payment of interest on debentures
and dividend while estimating cash flows. When profit after tax is given after considering
the interest payment, the net cash flows are computed by making appropriate adjustments
for interest payments. The actual mechanism can be explained with the help of following
example. VERY IMPORTANT TO UNDERSTAND
For example, suppose the following information is given—
Particulars ₹
Revenue 1,00,000
Less: Expenses 40,000
EBDIT (Earnings before depreciation, interest and taxes) 60,000
Less: Depreciation 10,000
EBIT (Earnings before interest and taxes) 50,000
Less: Interest 20,000
EBT (Earnings before taxes) 30,000
Less: Tax@40% 12,000
EAT (Earnings after taxes) 18,000
Compute net cash flows from the above data.
Particulars ₹
EAT (Earnings before taxes) 18,000
Add: Interest Tax Shield [𝐼𝑛𝑡𝑒𝑟𝑒𝑠𝑡 (1 − 𝑇𝑎𝑥)] 12,000
Add: Depreciation 10,000
Net Cash Flow 40,000
The tax benefit which arises due to tax deductibility of interest payment is called interest tax shield
which is equal to interest multiplied by the tax rate. The amount of PAT (Profits after taxes) is
arrived at by taking into account the interest tax shield.

Example 1
Raja Private Limited wants to invest in production plant which will cost ₹1,90,000. The cost of
installation will amount to ₹60,000. The increase in working capital will be ₹40,000. The plant is
expected to provide cash inflows before depreciation and taxes of ₹1,00,000 for a period of 5 years
after which it can be sold for ₹50,000. The firm is in tax bracket of 50% and uses straight line
method of depreciation. Calculate all the cash flows associated with the machine.

Solution
In this question following cash flows will be calculated—
1. Initial cash outflows/outlay
Cost of the plant 1,90,000
Add: Installation cost 60,000
Add: Working capital increase/Additional working capital 40,000
Initial cash outflows/outlay 2,90,000
Chapter 5 and 6, Capital Budgeting: 6
THERE IS NO CAPITAL LOSS/GAIN BECAUSE BOOK VALUE OF ASSET = SCRAP VALUE OF ASSET = 50000
BOOK VALUE = 190000+60000 - 200000(40000*5) = 50000 = SCRAP VALUE
2. Annual cash inflows (from year 1 to 4)
Cash inflows before depreciation and taxes 1,00,000
Less: Annual depreciation on plant -40,000
Profits before taxes 60,000
Less: Taxes @ 50% -30,000
Profits after taxes 30,000
Add: Annual depreciation on plant (Non-cash expenditure) 40,000
Net cash inflows (from year 1 to 4) 70,000

3. Terminal cash inflows or cash inflows for 5th year


Annual cash inflows 70,000
Add: Scrap value of plant 50,000
Add: Working capital released 40,000
Terminal cash inflows or cash inflows for 5th year 1,60,000

4. Annual depreciation
𝐶𝑜𝑠𝑡 𝑜𝑓 𝑡ℎ𝑒 𝑝𝑙𝑎𝑛𝑡 + 𝐼𝑛𝑠𝑡𝑎𝑙𝑙𝑎𝑡𝑖𝑜𝑛 𝑐ℎ𝑎𝑟𝑔𝑒𝑠 − 𝑆𝑐𝑟𝑎𝑝 𝑣𝑎𝑙𝑢𝑒
𝐴𝑛𝑛𝑢𝑎𝑙 𝑑𝑒𝑝𝑟𝑒𝑐𝑖𝑎𝑡𝑖𝑜𝑛 =
𝑈𝑠𝑒𝑓𝑢𝑙 𝑙𝑖𝑓𝑒 𝑜𝑓 𝑡ℎ𝑒 𝑝𝑙𝑎𝑛𝑡
1,90,000 + 60,000 − 50,000 2,00,000
⇒ ⇒ ⇒ ₹40,000 𝑝𝑒𝑟 𝑎𝑛𝑛𝑢𝑚
5 5

Example 2
Rameshwaram Private Limited wants to install a production plant costing ₹6,00,000 with an
installation charge of ₹1,50,000. Scrap value of the plant at the end of 10 years will be ₹3,00,000.
Sales for the first year will be ₹22,00,000 and will grow at 5% per annum. The profit after tax
would be 10% of sales and the working capital requirement will be 5% of sales of the next
year. Calculate cash inflows for the plant. Company uses straight line method of the depreciation.

Solution
1. Initial cash outflows/outlay
Cost of the plant 6,00,000
Add: Installation cost 1,50,000
Add: Working capital increase (₹22,00,000 × 5%) 1,10,000
Initial cash outflows/outlay 8,60,000

2. Annual depreciation
𝐶𝑜𝑠𝑡 𝑜𝑓 𝑡ℎ𝑒 𝑝𝑙𝑎𝑛𝑡 + 𝐼𝑛𝑠𝑡𝑎𝑙𝑙𝑎𝑡𝑖𝑜𝑛 𝑐ℎ𝑎𝑟𝑔𝑒𝑠 − 𝑆𝑐𝑟𝑎𝑝 𝑣𝑎𝑙𝑢𝑒
𝐴𝑛𝑛𝑢𝑎𝑙 𝑑𝑒𝑝𝑟𝑒𝑐𝑖𝑎𝑡𝑖𝑜𝑛 =
𝑈𝑠𝑒𝑓𝑢𝑙 𝑙𝑖𝑓𝑒 𝑜𝑓 𝑡ℎ𝑒 𝑝𝑙𝑎𝑛𝑡
AFTER TAX PROFIT 6,00,000 + 1,50,000 − 3,00,000 4,50,000
MEIN DEPRICIATION ⇒ ⇒ ⇒ ₹45,000 𝑝𝑒𝑟 𝑎𝑛𝑛𝑢𝑚
MINUS HO CHUKA 10 10
HOGA TO WE HAVE
TO ADD IT BACK 3. Cash inflows
Cash flows after tax but
before depreciation
Profit after Working (Profit after taxes +
taxes (10% Capital Depreciation - Working
Year Sales of sales) Depreciation (Incremental) capital)
1 22,00,000 2,20,000 45,000 5,500 2,59,500
2 23,10,000 2,31,000 45,000 5,775 2,70,225
3 24,25,500 2,42,550 45,000 6,064 2,81,486
4 25,46,775 2,54,678 45,000 6,367 2,93,311
5 26,74,114 2,67,411 45,000 6,685 3,05,726
6 28,07,819 2,80,782 45,000 7,020 3,18,763
7 29,48,210 2,94,821 45,000 7,370 3,32,451
8 30,95,621 3,09,562 45,000 7,739 3,46,823
Chapter 5 and 6, Capital Budgeting: 7
9 32,50,402 3,25,040 45,000 8,126 3,61,914
10 34,12,922 3,41,292 45,000 -- *8,56,938
*3,41,292+45,000+1,10,000 (Release of initial working capital)+3,00,000 (Scrap value)+60,646
(Release of incremental working capital See Table 4)

4. Incremental working capital


Working capital (5%
Year Sales of sales) Incremental working capital
0 0 *1,10,000 --
1 22,00,000 1,15,500 **5,500 (year 1 – year 0)
2 23,10,000 1,21,275 5,775 (year 2 – year 1)
3 24,25,500 1,27,339 6,064 (year 3 – year 2)
4 25,46,775 1,33,706 6,367 (year 4 – year 3)
5 26,74,114 1,40,391 6,685 (year 5 – year 4)
6 28,07,819 1,47,411 7,020 (year 6 – year 5)
7 29,48,210 1,54,781 7,370 (year 7 – year 6)
8 30,95,621 1,62,520 7,739 (year 8 – year 7)
9 32,50,402 1,70,646 8,126 (year 9 – year 8)
10 34,12,922 0 ***
Total incremental working capital 60,646
*In the 0 year the working capital will be based on the expected sales of the first year i.e.
5
22,00,000 × = ₹1,10,000
100
**Alternatively, the incremental working capital can also be calculated by multiplying the
increase of sales by 5% i.e.
5 5
(23,10,000 − 22,00,000) × ⇒ 1,10,000 × = ₹5,500
100 100
***In the 10 year there will be release of working capital instead of requirement.
th

Example 3
Rosa Private Limited is considering to replace a five-year-old machinery the written down value
of which is zero at present. There are two options available—either to update the technology by
spending ₹1,50,000 with economic life of 5 years or to buy a new machine costing ₹3,50,000 the
economic life of which is 5 years. The updated machinery can be sold for ₹30,000 after 5 years and
the new machine will have ₹80,000 scrap value at the end of 5th year. The annual costs of these
machines are as follows—
Particulars Existing (₹) Updated machine (₹) New machine (₹)
Salaries/wages 50,000 40,000 15,000
Manager’s/Supervisor’s Salary 25,000 11,000 8,000
Maintenance 30,000 9,000 3,000
Power 23,000 20,000 17,000
The tax rate is 40%. Assume straight line method of depreciation. Calculate the cash flows under
different options available to the company.

Solution
1. Initial cash outflows/outlay
Particulars Updated machine (₹) New machine (₹)
Cost of machinery 1,50,000 3,50,000

2. Annual depreciation
Updated machine New machine

Chapter 5 and 6, Capital Budgeting: 8


𝐴𝑛𝑛𝑢𝑎𝑙 𝑑𝑒𝑝𝑟𝑒𝑐𝑖𝑎𝑡𝑖𝑜𝑛 𝐴𝑛𝑛𝑢𝑎𝑙 𝑑𝑒𝑝𝑟𝑒𝑐𝑖𝑎𝑡𝑖𝑜𝑛
𝐶𝑜𝑠𝑡 𝑜𝑓 𝑡ℎ𝑒 𝑚𝑎𝑐ℎ𝑖𝑛𝑒 − 𝑆𝑐𝑟𝑎𝑝 𝑣𝑎𝑙𝑢𝑒 𝐶𝑜𝑠𝑡 𝑜𝑓 𝑡ℎ𝑒 𝑚𝑎𝑐ℎ𝑖𝑛𝑒 − 𝑆𝑐𝑟𝑎𝑝 𝑣𝑎𝑙𝑢𝑒
= =
𝑈𝑠𝑒𝑓𝑢𝑙 𝑙𝑖𝑓𝑒 𝑜𝑓 𝑡ℎ𝑒 𝑚𝑎𝑐ℎ𝑖𝑛𝑒 𝑈𝑠𝑒𝑓𝑢𝑙 𝑙𝑖𝑓𝑒 𝑜𝑓 𝑡ℎ𝑒 𝑚𝑎𝑐ℎ𝑖𝑛𝑒
1,50,000 − 30,000 1,20,000 3,50,000 − 80,000 2,70,000
⇒ ⇒ ⇒ ⇒
5 5 5 5
⇒ ₹24,000 𝑝𝑒𝑟 𝑎𝑛𝑛𝑢𝑚 ⇒ ₹54,000 𝑝𝑒𝑟 𝑎𝑛𝑛𝑢𝑚
GOOD POINT "BASED ON SAVINGS"
3. Cash inflows (based on savings) (from year 1 to 4)
Particulars Updated machine (₹) New machine (₹)
Salaries/wages (50,000-40,000) 10,000 (50,000-15,000) 35,000
Manager’s/Supervisor’s Salary (25,000-11,000) 14,000 (25,000-8,000) 17,000
Maintenance (30,000-9,000) 21,000 (30,000-3,000) 27,000
Power (23,000-20,000) 3,000 (23,000-17,000) 6,000
Total savings 48,000 85,000
Less: Depreciation -24,000 -54,000
Savings before tax 24000 31,000
Less: Tax @ 40% -9,600 -12,400
Savings after tax 14,400 18,600
Add: Depreciation 24,000 54,000
Cash flows after taxes or cash savings 38,400 72,600

3. Terminal cash inflows or cash inflows for 5th year


Particulars Updated machine New machine
(₹) ( ₹)
Annual cash inflows 38,400 72,600
Add: Scrap value 30,000 80,000
Terminal cash inflows or cash inflows for 5 year
th 68,400 1,52,600

Example 4
Cost of a machine is ₹1,00,000 and installation charges are 20,000. The machine has a life of 5
years and after it the scrap value will be ₹10,000. Machine can produce 2,000 units per annum from
year 1 to 2 and 3,000 units per annum from year 3 to 5. The product is expected to fetch ₹15 per
unit in the first 3 years and ₹18 per unit in the last 2 years. The operating cost is ₹5,000 per annum
for the first 3 years and ₹8,000 per annum for the last 2 years. Straight line method of depreciation
is being followed by the company and tax rate is 40%. Calculate the cash flows.

Solution
1. Initial cash outflows/outlay
Cost of the machine 1,00,000
Add: Installation cost 20,000
Initial cash outflows/outlay 1,20,000

2. Annual depreciation
𝐶𝑜𝑠𝑡 𝑜𝑓 𝑡ℎ𝑒 𝑚𝑎𝑐ℎ𝑖𝑛𝑒 + 𝐼𝑛𝑠𝑡𝑎𝑙𝑙𝑎𝑡𝑖𝑜𝑛 𝑐ℎ𝑎𝑟𝑔𝑒𝑠 − 𝑆𝑐𝑟𝑎𝑝 𝑣𝑎𝑙𝑢𝑒
𝐴𝑛𝑛𝑢𝑎𝑙 𝑑𝑒𝑝𝑟𝑒𝑐𝑖𝑎𝑡𝑖𝑜𝑛 =
𝑈𝑠𝑒𝑓𝑢𝑙 𝑙𝑖𝑓𝑒 𝑜𝑓 𝑡ℎ𝑒 𝑚𝑎𝑐ℎ𝑖𝑛𝑒
1,00,000 + 20,000 − 10,000 1,10,000
⇒ ⇒ ⇒ ₹22,000 𝑝𝑒𝑟 𝑎𝑛𝑛𝑢𝑚
5 5

3. Annual cash inflows


Years
Particulars 1 2 3 4 5
Output (units) 2,000 2,000 3,000 3,000 3,000
Selling price 15 15 15 18 18
Sales/Revenue (𝑈𝑛𝑖𝑡𝑠 × 𝑃𝑟𝑖𝑐𝑒) 30,000 30,000 45,000 54,000 54,000
Less: Operating expenses -5,000 -5,000 -5,000 -8,000 -8,000
Chapter 5 and 6, Capital Budgeting: 9
Less: Depreciation -22,000 -22,000 -22,000 -22,000 -22,000
Profits before taxes 3,000 3,000 18,000 24,000 24,000
Less: Taxes @ 40% -1,200 -1,200 -7,200 -9,600 -9,600
Profits after taxes 1,800 1,800 10,800 14,400 14,400
Add: Depreciation 22,000 22,000 22,000 22,000 22,000
Add: Scrap value 0 0 0 0 10,000 IMPORTANT TO
Cash flows after taxes 23,800 23,800 32,800 36,400 46,400 NOTE

Example 5 (Important) CONCEPT OF INTEREST ADDITION IS EXPLAINED

Income statement of a project is as follows—


Income statement
Particulars ₹ Particulars ₹
To cost of goods sold 1,50,000 By sales 3,00,000
To general expenses 50,000
To depreciation 10,000
To interest 15,000
To tax @ 50% 37,500
To net profit 37,500 _
Total 3,00,000 Total 3,00,000
Calculate cash flows.

Solution
Estimation of operating cash inflows
Sales 3,00,000
Less: Cost of goods sold -1,50,000
Less: General expenses -50,000
Less: Depreciation -10,000
Earnings before interest and taxes (EBIT) 90,000
Less: Taxes @ 50% -45,000
Earnings after taxes (EAT) 45,000
Add: Depreciation 10,000
Cash inflows 55,000

Estimation of operating cash inflows (ALTERNATE METHOD) (Click to see the format)
Net profit (Profits after depreciation, interest and tax) 37,500
Add: Depreciation 10,000
Add: Interest (1 − 𝑡) ⇒ [15,000 × (1 − 0.50)] 7,500
Cash inflows 55,000

Chapter 5 and 6, Capital Budgeting: 10


Type of capital budgeting decisions (from the point of view of
Decision Situations)
1. Accept/Reject: When we accept the
project on the basis of a single cutoff Annuity stream: When cash flows to be
period or cutoff rate then this is called received each year are constant, this is
accept/reject decisions. E.g. if 𝑘𝑂 < called annuity stream.
project’s rate of return then we accept the
project and vice versa. Mixed stream: When the cash flows to
2. Mutually Exclusive: When the selection of be received each year are not constant
one project leads to the rejection of other then this is called mixed stream.
projects. Only one is to be selected which
is the best one. Conventional Cash Flows: These are
3. Capital Rationing: When there are many the cash flows in which cash outflows
projects but investment is to be made in occurred in the 0 year and then there is a
some projects then they are ranked series of cash inflows for a particular
according to their profitability. And then time period. E.g.: Investment in 0 year
investment is made accordingly i.e. from and cash inflows from 1st to 5th year.
highest profitability to lowest profitability.
It must be noted that those projects are Non-Conventional Cash Flows: These
not considered here which were rejected are the cash flows in which cash outflows
earlier under the Accept/Reject criterion. do not occur in 0 year but in various
From the point of view of firm’s existence years in order to get the stream of cash
(a) New firm inflows. E.g.: Investment in 0, 2nd, 3rd
(b) Existing firm year and cash inflows from 1st to 7th year.
a. Replacement and modernization
b. Expansion
c. Diversification (or revenue increasing decisions)

Chapter 5 and 6, Capital Budgeting: 11


Evaluation techniques
1. Payback Period Method
Payback period is that period in which the original investment is recouped (recovered) back. The
project which recovers the amount in the shortest period is the best project.

Decision Rule

Accept/reject criteria
𝐼𝑓 𝑃𝑟𝑜𝑗𝑒𝑐𝑡’𝑠 𝑃𝑎𝑦 𝐵𝑎𝑐𝑘 𝑃𝑒𝑟𝑖𝑜𝑑 < 𝑆𝑡𝑎𝑛𝑑𝑎𝑟𝑑 𝑃𝑎𝑦𝑏𝑎𝑐𝑘 𝑃𝑒𝑟𝑖𝑜𝑑 = 𝐴𝑐𝑐𝑒𝑝𝑡 𝑡ℎ𝑒 𝑃𝑟𝑜𝑗𝑒𝑐𝑡
𝐼𝑓 𝑃𝑟𝑜𝑗𝑒𝑐𝑡’𝑠 𝑃𝑎𝑦 𝐵𝑎𝑐𝑘 𝑃𝑒𝑟𝑖𝑜𝑑 > 𝑆𝑡𝑎𝑛𝑑𝑎𝑟𝑑 𝑃𝑎𝑦𝑏𝑎𝑐𝑘 𝑃𝑒𝑟𝑖𝑜𝑑 = 𝑅𝑒𝑗𝑒𝑐𝑡 𝑡ℎ𝑒 𝑃𝑟𝑜𝑗𝑒𝑐𝑡

Mutually exclusive projects


Select the project with the lowest payback period

Capital rationing
Rank the projects from the lowest payback period to the highest and then invest accordingly.
Minimum payback period Rank I 
Moderate payback period Rank II 
Maximum payback period Rank III 

Merits and demerits of payback period method

Merits
1. It is simple and easy to compute. This method does not need any complex mathematical
tools to calculate the payback value. This task can be performed without any specialized
knowledge of financial tools.
2. Most of the firms place very high importance on the liquidity. It explains us the time when
the original investment can be recovered and companies can arrange funds accordingly.
3. It can give us very good analysis about the riskiness of the project. The longer the time it
takes to cover the initial investment, riskier is the project. It helps in identifying the risk of
the project on the basis of time framework.
4. Its interpretation is simple to understand. It is simple in reading and understanding it even
by a layman. It talks in the common man’s language.

Demerits
1. This method does not take into account the cash flows occurring after the payback period. It
ignores all these cash flows. For example, consider the case of following two projects which
have the same cost of ₹1,00,000 and the cash flows are given below—
Year Cash Flows (A) Cash Flows (B)
1 25,000 30,000
2 40,000 25,000
3 35,000 45,000
4 10,000 40,000
5 15,000 60,000
Both the projects have an equal payback period which is 3 years. On the basis of payback
criteria both projects are good at the same level. However, we can easily see that the project
B is far better than project A as the cash flows after payback period are high for project B.
Thus, the payback method fails to capture the cash flows after payback period.
2. This method does not consider the timing of the cash flows. All cash flows are of equal
importance whether they occur earlier or later. For example, there are two projects namely
X and Y, both have the equal investment outlay of ₹10,000. The cash flows are given
below—
Year Cash Flows (X) Cash Flows (Y)
Chapter 5 and 6, Capital Budgeting: 12
1 1,000 7,000
2 2,000 2,000
3 7,000 1,000
In this case, both the projects have the equal payback period of three years, however, more
money is recovered earlier in the project Y as compared to project X. The money which is
received earlier is better because of time value of money. Under this scenario also, the
payback method falls short of proving a better guide to take the project.
3. It does not take into account the salvage value of the project. Sometimes the value of salvage
is very high and can become an important factor in making a project profitable.
On the basis of above discussion of pros and cons of payback, we can conclude that it is the payback
method despite having some weakness is a useful measure of making investment decision when it is
difficult to forecast the cash flows after two or three years. Forecasting cash flows beyond three
years is very difficult and payback method comes to our rescue to deal with these situations. Also,
when the firm is funds starved, it would like to have the projects which will recover the costs as
early as possible. Payback method gives the answer to this question and helps the management in
making proper investment decision.

Computation of the Pay-back Period

Annuity stream CONTSTANT CASH FLOWS IN EVERY YEAR (SAME AMOUNT OF CASH FLOWS)

Following formula is used to compute the payback period in case of annuity stream: OR :
CONVENTIONAL
𝐼𝑛𝑖𝑡𝑖𝑎𝑙 𝐼𝑛𝑣𝑒𝑠𝑡𝑚𝑒𝑛𝑡 𝑜𝑟 𝐼𝑛𝑡𝑖𝑎𝑙 𝑂𝑢𝑡𝑙𝑎𝑦 CASH FLOWS
LEARN 𝑃𝑎𝑦𝑏𝑎𝑐𝑘 𝑝𝑒𝑟𝑖𝑜𝑑 =
𝐶𝑜𝑛𝑠𝑡𝑎𝑛𝑡 𝐶𝑎𝑠ℎ 𝐹𝑙𝑜𝑤𝑠

Mixed stream TWO OR MORE THAN 2 YEARS HAVING DIFFERENT CASH FLOWS
In case of mixed stream, the procedure given in the following example is to be followed—
E.g.: Using given cash outflows and cash inflows calculate the payback period—
Year CFAT Cumulative CFAT
0 (70,000) NEGATIVE BECAUSE CASH --
1 6,000 OUTFLOW OR INITIAL 6,000
INVESTMENT IN 0 YEAR
2 12,000 18,000
3 17,000 35,000
4 20,000 55,000
5 20,000 75,000
6 25,000 1,00,000
(70,000 − 55,000) 15,000
𝑃𝑎𝑦𝑏𝑎𝑐𝑘 𝑝𝑒𝑟𝑖𝑜𝑑 = 4 + ⇒4+
20,000 IN FRONT OF YEAR 5 20,000
⇒ 4.75 𝑦𝑒𝑎𝑟𝑠 𝑜𝑟 4 𝑦𝑒𝑎𝑟𝑠 𝑎𝑛𝑑 9 𝑚𝑜𝑛𝑡ℎ𝑠

2. Average/Accounting Rate of Return Method


Accounting/average rate of return is simply the average net profits of the project divided by the
average investment.

Decision Rule

Accept/reject criteria
If Project’s 𝐴𝑅𝑅 < 𝑆𝑡𝑎𝑛𝑑𝑎𝑟𝑑 𝐴𝑅𝑅 = 𝑅𝑒𝑗𝑒𝑐𝑡 𝑡ℎ𝑒 𝑃𝑟𝑜𝑗𝑒𝑐𝑡
If Project’s 𝐴𝑅𝑅 > 𝑆𝑡𝑎𝑛𝑑𝑎𝑟𝑑 𝐴𝑅𝑅 = 𝐴𝑐𝑐𝑒𝑝𝑡 𝑡ℎ𝑒 𝑃𝑟𝑜𝑗𝑒𝑐𝑡

Mutually exclusive projects


Select the project with the highest ARR

Capital rationing
Rank the projects from the highest ARR to the lowest and then invest accordingly.

Chapter 5 and 6, Capital Budgeting: 13


Maximum ARR Rank I 
Moderate ARR Rank II 
Minimum ARR Rank III 

Merits and demerits

Merits
1. It is simple to calculate and understand.
2. Its basis is on accounting data which can be simply accessed from accounts of company.
3. It measures the benefits in percentage which makes it easily comparable and provides an
easy rule to make investment decision.

Demerits
1. The first problem with this method is that it is based on account profit rather than cash
flows. We discussed in earlier section that the objective of financial management is to
maximize the wealth of the shareholders. To attain this goal, the focus must be on the cash
flows rather than on the accounting profits.
2. The other major shortcoming of this method is that it does not take into account the time
value of money. This method treats all cash flows on equal footing, it does not pay attention
to the timing of the cash flow. The amount of cash flow whether received early or late is
given equal importance which is contrary to the concept of time value of money.
3. This method ignores the size of the investment. Sometimes, the accounting rate of return
may be same for various projects but some may involve huge cash flows. Given the size of
investment, the decision cannot simply be taken on the basis of accounting rate of return.
For example, consider the following situation—
Machine Average annual profits Average investment ARR
A 10,000 1,00,000 10%
B 1,500 15,000 10%
C 1,000 10,000 10%
Thus, for all three machines A, B and C, the accounting rate of return is same however,
machine A requires huge cash outlay and the decision cannot be taken solely on the basis of
ARR. The availability of funds is also another concern which should be taken into
consideration while making investment decision.

How to compute the ARR?


𝐴𝑣𝑒𝑟𝑎𝑔𝑒 𝑁𝑒𝑡 𝑃𝑟𝑜𝑓𝑖𝑡𝑠 𝑜𝑓 𝑡ℎ𝑒 𝑃𝑟𝑜𝑗𝑒𝑐𝑡 ∗
𝐴𝑅𝑅 = × 100
𝐴𝑣𝑒𝑟𝑎𝑔𝑒 𝐼𝑛𝑣𝑒𝑠𝑡𝑚𝑒𝑛𝑡 𝑖𝑛 𝑡ℎ𝑒 𝑃𝑟𝑜𝑗𝑒𝑐𝑡 ∗∗

𝑇𝑜𝑡𝑎𝑙 𝐸𝑥𝑝𝑒𝑐𝑡𝑒𝑑 𝑁𝑒𝑡 𝑃𝑟𝑜𝑓𝑖𝑡𝑠 𝑜𝑓 𝑡ℎ𝑒 𝑃𝑟𝑜𝑗𝑒𝑐𝑡


∗ 𝐴𝑣𝑒𝑟𝑎𝑔𝑒 𝑁𝑒𝑡 𝑃𝑟𝑜𝑓𝑖𝑡𝑠 𝑜𝑓 𝑡ℎ𝑒 𝑃𝑟𝑜𝑗𝑒𝑐𝑡 =
𝐸𝑠𝑡𝑖𝑚𝑎𝑡𝑒𝑑 𝑈𝑠𝑒𝑓𝑢𝑙 𝐿𝑖𝑓𝑒 𝑜𝑓 𝑡ℎ𝑒 𝑃𝑟𝑜𝑗𝑒𝑐𝑡

𝐼𝑛𝑖𝑡𝑖𝑎𝑙 𝐼𝑛𝑣𝑒𝑠𝑡𝑚𝑒𝑛𝑡 + 𝑆𝑎𝑙𝑣𝑎𝑔𝑒 𝑉𝑎𝑙𝑢𝑒


∗∗ 𝐴𝑣𝑒𝑟𝑎𝑔𝑒 𝐼𝑛𝑣𝑒𝑠𝑡𝑚𝑒𝑛𝑡 = + 𝑊𝑜𝑟𝑘𝑖𝑛𝑔 𝐶𝑎𝑝𝑖𝑡𝑎𝑙
2
∗∗ 𝐴𝑣𝑒𝑟𝑎𝑔𝑒 𝐼𝑛𝑣𝑒𝑠𝑡𝑚𝑒𝑛𝑡
𝐼𝑛𝑖𝑡𝑖𝑎𝑙 𝐼𝑛𝑣𝑒𝑠𝑡𝑚𝑒𝑛𝑡 − 𝑆𝑎𝑙𝑣𝑎𝑔𝑒 𝑉𝑎𝑙𝑢𝑒
= + 𝑊𝑜𝑟𝑘𝑖𝑛𝑔 𝐶𝑎𝑝𝑖𝑡𝑎𝑙
2
+ 𝑆𝑎𝑙𝑣𝑎𝑔𝑒 𝑉𝑎𝑙𝑢𝑒

𝐼𝑛𝑖𝑡𝑖𝑎𝑙 𝐼𝑛𝑣𝑒𝑠𝑡𝑚𝑒𝑛𝑡/𝑂𝑢𝑡𝑙𝑎𝑦 − 𝑆𝑙𝑎𝑣𝑎𝑔𝑒 𝑉𝑎𝑙𝑢𝑒


𝐷𝑒𝑝𝑟𝑒𝑐𝑖𝑎𝑡𝑖𝑜𝑛 =
𝐸𝑠𝑡𝑖𝑚𝑎𝑡𝑒𝑑 𝐿𝑖𝑓𝑒 𝑜𝑓 𝑡ℎ𝑒 𝑃𝑟𝑜𝑗𝑒𝑐𝑡

Chapter 5 and 6, Capital Budgeting: 14


3. Net Present Value Method

Meaning
Net present value is simply the Present Value of Cash Inflows of a Project minus the Present Value of
Cash Outflows of the Project.

Decision Rule

Accept/reject criteria
At this stage we select as many assets as possible. Basically, we create a pool of so many assets
which are under consideration. The selection of so many assets does not mean that all of these will
be purchased. We will evaluate the selected assets either under mutually exclusive or capital
rationing criteria.
𝐼𝑓 𝑃𝑟𝑜𝑗𝑒𝑐𝑡’𝑠 𝑁𝑃𝑉 < 0 = 𝐷𝑜 𝑛𝑜𝑡 𝑠𝑒𝑙𝑒𝑐𝑡 𝑡ℎ𝑒 𝑎𝑠𝑠𝑒𝑡
𝐼𝑓 𝑃𝑟𝑜𝑗𝑒𝑐𝑡’𝑠 𝑁𝑃𝑉 > 0 = 𝑆𝑒𝑙𝑒𝑐𝑡 𝑡ℎ𝑒 𝑎𝑠𝑠𝑒𝑡

Mutually exclusive projects


Projects or assets are said to be mutually exclusive when the selection of one project leads to the
rejection of other projects. Only one project or asset is to be purchased which is the best one.
Select the project with the highest NPV

Capital rationing
When there are many projects but investment is to be made in some projects then they are ranked
according to their profitability. And then investment is made accordingly i.e., from highest
profitability to lowest profitability. It must be noted that those projects are not considered here
which are rejected earlier under the Accept/Reject criterion.
Rank the projects from the highest NPV to the lowest and then invest accordingly.
Maximum NPV Rank I 
Moderate NPV Rank II 
Minimum NPV Rank III 

Merits and demerits

Merits
1. One of the most necessary advantages of NPV technique is that it takes into consideration
the time value of money. This is the major improvement over the traditional approach to
capital budgeting.
2. It considers cash flows to analyze the proposal rather than the accounting profits. We saw
earlier that the accounting profits do not reflect the true picture of the financial position of
the firm. Accounting profits can be interpreted by using distinct accounting methods. Cash
flows do not suffer from these limitations.
3. Different discount rates can be easily incorporated into the NPV computations. Thus, we can
see the sensitivity of the discount rates on the profitability of the project.
4. This method considers the all the cash flows spreading over the whole life of the project.
5. NPV technique is related to the basic objective of financial management, i.e., maximization of
the wealth of the shareholders. The wealth of the shareholders will be maximized when firm
takes up the project which is generate extra cash flows in present value terms. In order to
answer the question whether a particular project generates value to the shareholders, we
use NPV analysis.

Demerits
1. It is hard to compute as compared to the traditional tools of capital budgeting such as
payback method and ARR method.

Chapter 5 and 6, Capital Budgeting: 15


2. It is based on one very important factor, i.e., discount rate which is the weighted average
cost of capital. Theoretically, this is very sound the logical concept, however, it is very
difficult to compute in practice. This is the major flaw of NPV method. A slight change in
discount rate can alter the value of NPV dramatically.
3. This is based on absolute amount of cash flows. Given a choice between two projects, we
will choose a project with high NPV, though there is a possibility that the project with high
NPV involves huge investment. Under the situation of limited capital, it is not possible for
the firm to take up those projects. The NPV analysis fails to take it into account.
4. When two projects have different life span, it is difficult to use NPV criteria to make
investment decision. For example, suppose there are two machines with 4 years and 6 years
life having the same NPV of let’s say, ₹24,000. NPV criteria would suggest that both the
machines are equally good; however, we can see that first machine needs to be replaced at
the end of 4th year whereas the second machine can work for 6 year. However, we can use a
modified form of discounting technique to incorporate this aspect which we will see later
on.
Despite these limitations, it is one of the most commonly used method for the evaluation of capital
budgeting decision and a significant improvement over the traditional methods of capital budgeting.

How to compute the NPV?


𝑁𝑃𝑉 = 𝑃𝑟𝑒𝑠𝑒𝑛𝑡 𝑉𝑎𝑙𝑢𝑒 𝑜𝑓 𝐶𝑎𝑠ℎ 𝐼𝑛𝑓𝑙𝑜𝑤𝑠 − 𝑃𝑟𝑒𝑠𝑒𝑛𝑡 𝑉𝑎𝑙𝑢𝑒 𝑜𝑓 𝐶𝑎𝑠ℎ 𝑂𝑢𝑡𝑓𝑙𝑜𝑤𝑠
𝑛 𝑛
𝐶𝐹𝐴𝑇𝑡 𝐶𝑂𝑡
𝑁𝑃𝑉 = ∑ 𝑡
−∑
(1 + 𝑘𝑂 ) (1 + 𝑘𝑂 )𝑡
𝑡=1 𝑡=0
𝑛 𝑛
1 1
𝑁𝑃𝑉 = ∑ 𝐶𝐹𝐴𝑇𝑡 × 𝑡
− ∑ 𝐶𝑂𝑡 ×
(1 + 𝑘𝑂 ) (1 + 𝑘𝑂 )𝑡
𝑡=1 𝑡=0
𝑛 𝑛
OUTFLOW OR INVESTMENT IS THERE IN 0 PERIOD
𝑁𝑃𝑉 = ∑ 𝐶𝐹𝐴𝑇𝑡 × 𝑃𝑉𝐹𝑘𝑂,𝑡 − ∑ 𝐶𝑂𝑡 × 𝑃𝑉𝐹𝑘𝑂 ,𝑡 INFLOWS START FROM 1 YEAR
𝑡=1 𝑡=0
𝑛

𝑁𝑃𝑉 = ∑ 𝐶𝐹𝐴𝑇𝑡 × 𝑃𝑉𝐹𝑘𝑂,𝑡 − 𝐶𝑂0


𝑡=1
(Because cash outflows generally occur in the 0 year and in the 0 year the present value
factor is always 1, so, in the 0 year, there is no need to calculate the present value of the cash
outflows)
Where,
NPV= Net present value
𝑡 = Time period
𝑛 = Estimated life of the asset
𝐶𝐹𝐴𝑇 = Cash flows after tax
𝑘𝑂 = Overall cost of capital or discount rate or rate of interest or minimum rate of return or
opportunity cost of capital or standard rate of return
𝐶𝑂0 = Cash outflows in the zero year or initial outlay or initial investment
1
(1+𝑘 )𝑡
= Present value factor for the given discount rate and for the given year
𝑂
𝑃𝑉𝐹𝑘𝑂 ,𝑡 = Present value factor for the given discount rate and for the given year
𝑃𝑉𝐴𝐹𝑘𝑂,𝑡 = Present value annuity factor for the given discount rate and for the given year
1
1−
(1+𝑘𝑂 )𝑡
[ 𝑘𝑂
] = Present value annuity factor for the given discount rate and for the given year

Initial Outlay (𝑪𝑶𝟎 )


Initial outlay is the investment made by the firm. It consists of the initial amount invested (including
installation expenses and other relevant expenses) and working capital introduced in the 0 year. In
case of sales of old plant, the scrap value shall be subtracted.

Chapter 5 and 6, Capital Budgeting: 16


Depreciation
𝐼𝑛𝑡𝑖𝑎𝑙 𝐼𝑛𝑣𝑒𝑠𝑡𝑚𝑒𝑛𝑡 𝑜𝑟 𝑂𝑢𝑡𝑙𝑎𝑦 + 𝐼𝑛𝑠𝑡𝑎𝑙𝑙𝑎𝑡𝑖𝑜𝑛 𝐸𝑥𝑝𝑒𝑛𝑠𝑒𝑠 − 𝑆𝑎𝑙𝑣𝑎𝑔𝑒 𝑉𝑎𝑙𝑢𝑒
𝐷𝑒𝑝𝑟𝑒𝑐𝑖𝑎𝑡𝑖𝑜𝑛 =
𝐸𝑠𝑡𝑖𝑚𝑎𝑡𝑒𝑑 𝐿𝑖𝑓𝑒 𝑜𝑓 𝑡ℎ𝑒 𝐴𝑠𝑠𝑒𝑡

Treatment of Working Capital


Working capital shall be added to the cash outflows. It shall also be added to the cash inflows of the
last/terminal year as it is assumed that it is recovered/released in the last/terminal year.

Treatment of Salvage Value


It is assumed that the salvage value is recovered in the last/terminal year, so, it shall be added to the
cash inflows of the last/terminal year.

Computation of NPV in case of Annuity Stream


In case of annuity stream the calculation is very easy and can be understood with the help of the
following example—

Example 6
Cash outflows or price of an asset is ₹30,000. This asset will provide cash flows of ₹10,000 every
year for a period of 5 years. What is the net present value (NPV) of this asset if the discount rate is
10%? On the basis of NPV what decision will you take regarding the purchase of this asset?

Solution

Direct method
𝑁𝑃𝑉 = 𝑃𝑟𝑒𝑠𝑒𝑛𝑡 𝑉𝑎𝑙𝑢𝑒 𝑜𝑓 𝐶𝑎𝑠ℎ 𝐼𝑛𝑓𝑙𝑜𝑤𝑠 − 𝐶𝑎𝑠ℎ 𝑂𝑢𝑡𝑓𝑙𝑜𝑤𝑠 𝑖𝑛 𝑡ℎ𝑒 𝑍𝑒𝑟𝑜 𝑌𝑒𝑎𝑟
𝑁𝑃𝑉 = 𝐴𝑛𝑛𝑢𝑎𝑙 𝐶𝑎𝑠ℎ 𝐼𝑛𝑓𝑙𝑜𝑤𝑠 × 𝑃𝑉𝐴𝐹𝑘𝑂,𝑡 − 𝐶𝑎𝑠ℎ 𝑂𝑢𝑡𝑓𝑙𝑜𝑤𝑠 𝑖𝑛 𝑡ℎ𝑒 𝑍𝑒𝑟𝑜 𝑌𝑒𝑎𝑟
𝑁𝑃𝑉 = 𝐴𝑛𝑛𝑢𝑎𝑙 𝐶𝑎𝑠ℎ 𝐼𝑛𝑓𝑙𝑜𝑤𝑠 × 𝑃𝑉𝐴𝐹10%,5 − 𝐶𝑎𝑠ℎ 𝑂𝑢𝑡𝑓𝑙𝑜𝑤𝑠 𝑖𝑛 𝑡ℎ𝑒 𝑍𝑒𝑟𝑜 𝑌𝑒𝑎𝑟
1
1−
(1 + 𝑘𝑂 )𝑡
𝑁𝑃𝑉 = 𝐴𝑛𝑛𝑢𝑎𝑙 𝐶𝑎𝑠ℎ 𝐼𝑛𝑓𝑙𝑜𝑤𝑠 × [ ] − 𝐶𝑎𝑠ℎ 𝑂𝑢𝑡𝑓𝑙𝑜𝑤𝑠 𝑖𝑛 𝑡ℎ𝑒 𝑍𝑒𝑟𝑜 𝑌𝑒𝑎𝑟
𝑘𝑂
1
1−
(1 + 0.10)5
𝑁𝑃𝑉 = ₹10,000 × [ ] − ₹30,000
0.10

𝑁𝑃𝑉 = ₹10,000 × 3.791 − ₹30,000


𝑁𝑃𝑉 = ₹37,910 − ₹30,000
𝑁𝑃𝑉 = ₹7,910
Decision: As the net present value (NPV) is more than 0 or the NPV is positive, so, we should take
the decision to purchase the asset.

Alternate method
𝑁𝑃𝑉 = 𝑃𝑟𝑒𝑠𝑒𝑛𝑡 𝑉𝑎𝑙𝑢𝑒 𝑜𝑓 𝐶𝑎𝑠ℎ 𝐼𝑛𝑓𝑙𝑜𝑤𝑠 − 𝑃𝑟𝑒𝑠𝑒𝑛𝑡 𝑉𝑎𝑙𝑢𝑒 𝑜𝑓 𝐶𝑎𝑠ℎ 𝑂𝑢𝑡𝑓𝑙𝑜𝑤𝑠
𝑛 𝑛
𝐶𝐹𝐴𝑇𝑡 𝐶𝑂𝑡
𝑁𝑃𝑉 = ∑ 𝑡
−∑
(1 + 𝑘𝑂 ) (1 + 𝑘𝑂 )𝑡
𝑡=1 𝑡=0
𝑛=5
1
𝑁𝑃𝑉 = ∑ 𝐶𝐹𝐴𝑇𝑡 × − 𝐶𝑂0
(1 + 𝑘𝑂 )𝑡
𝑡=1
1 1 1
𝑁𝑃𝑉 = 𝐶𝐹𝐴𝑇1 × + 𝐶𝐹𝐴𝑇 2 × + 𝐶𝐹𝐴𝑇3 ×
(1 + 0.10)1 (1 + 0.10)2 (1 + 0.10)3
1 1
+ 𝐶𝐹𝐴𝑇4 × + 𝐶𝐹𝐴𝑇5 × − 𝐶𝑂0
(1 + 0.10)4 (1 + 0.10)5

Chapter 5 and 6, Capital Budgeting: 17


𝑁𝑃𝑉 = 10,000 × 0.909 + 10,000 × 0.826 + 10,000 × 0.751 + 10,000 × 0.683
+ 10,000 × 0.621 − 30,000
𝑁𝑃𝑉 = 9,090 + 8,260 + 7,510 + 6,830 + 6,210 − 30,000
𝑁𝑃𝑉 = 37,920 − 30,000
𝑁𝑃𝑉 = ₹7,920
or
𝑁𝑃𝑉 = 10,000 × 0.909 + 10,000 × 0.826 + 10,000 × 0.751 + 10,000 × 0.683
+ 10,000 × 0.621 − 30,000
𝑁𝑃𝑉 = 10,000𝑆𝑒𝑒 𝑁𝑜𝑡𝑒−1 (0.909 + 0.826 + 0.751 + 0.683 + 0.621) − 30,000
𝑁𝑃𝑉 = 10,000 × 3.792𝑆𝑒𝑒 𝑁𝑜𝑡𝑒−2 − 30,000
𝑁𝑃𝑉 = 37,920 − 30,000
𝑁𝑃𝑉 = ₹7,920
Notes:
1. 10,000 has been taken as common number.
2. The 3.792 is the present value annuity factor (𝑃𝑉𝐴𝐹𝑘𝑂 ,𝑡 ).
Decision: As the net present value (NPV) is more than 0 or the NPV is positive, so, we should take
the decision to purchase the asset.

Activity
Complete the PVF table on your own.
Year 5% 10% 15% 20% 25% 30% 35% 40%
0 1.000 1 1 1 1 1 1 1
1 0.952 0.909 0.870 ? ? ? ? ?
2 0.907 0.826 ? 0.694 ? ? ? ?
3 0.864 0.751 0.658 ? ? ? ? ?
4 0.823 0.683 ? 0.482 ?
5 0.784 0.621 0.497 ? ? 1
6 0.746 ? ? 0.335 ? 𝑃𝑉𝐹𝑘𝑂,𝑡 =
7 0.711 ? 0.376 ? ? (1 + 𝑘𝑂 )𝑡
8 0.677 ? ? 0.233 ?

Computation of NPV in case of Mixed Stream


Example 7
A company is considering an investment proposal to purchase a machine costing ₹50,000. The
machine has a life expectancy of 5 years and no salvage value. The estimated cash inflows from year
1 to 5 are as follows: ₹10,000; ₹10,450; ₹11,800; ₹12,250 and ₹16,750. The appropriate discount
rate is 10%. What is the net present value of this machine?

Solution

First method
𝑁𝑃𝑉 = 𝑃𝑟𝑒𝑠𝑒𝑛𝑡 𝑉𝑎𝑙𝑢𝑒 𝑜𝑓 𝐶𝑎𝑠ℎ 𝐼𝑛𝑓𝑙𝑜𝑤𝑠 − 𝑃𝑟𝑒𝑠𝑒𝑛𝑡 𝑉𝑎𝑙𝑢𝑒 𝑜𝑓 𝐶𝑎𝑠ℎ 𝑂𝑢𝑡𝑓𝑙𝑜𝑤𝑠
𝑛 𝑛
𝐶𝐹𝐴𝑇𝑡 𝐶𝑂𝑡
𝑁𝑃𝑉 = ∑ − ∑
(1 + 𝑘𝑂 )𝑡 (1 + 𝑘𝑂 )𝑡
𝑡=1 𝑡=0
𝑛=5
1
𝑁𝑃𝑉 = ∑ 𝐶𝐹𝐴𝑇𝑡 × − 𝐶𝑂0
(1 + 𝑘𝑂 )𝑡
𝑡=1
1 1 1
𝑁𝑃𝑉 = 𝐶𝐹𝐴𝑇1 × 1
+ 𝐶𝐹𝐴𝑇2 × 2
+ 𝐶𝐹𝐴𝑇3 ×
(1 + 0.10) (1 + 0.10) (1 + 0.10)3
1 1
+ 𝐶𝐹𝐴𝑇4 × + 𝐶𝐹𝐴𝑇5 × − 𝐶𝑂0
(1 + 0.10)4 (1 + 0.10)5
𝑁𝑃𝑉 = 10,000 × 0.909 + 10,450 × 0.826 + 11,800 × 0.751 + 12,250 × 0.683
+ 16,750 × 0.621 − 50,000

Chapter 5 and 6, Capital Budgeting: 18


𝑁𝑃𝑉 = 9,090 + 8,632 + 8,862 + 8,367 + 10,402 − 50,000
𝑁𝑃𝑉 = 45,353 − 50,000
𝑁𝑃𝑉 = (₹4,647)
Decision: Since the NPV is negative, so, the asset should not be purchased.

Second method
Year Cash Flows After Tax Present Value Factor at Present Value of Cash
(CFAT) 10% (PVF10%, t)See Note Flows After Tax
1 10,000 0.909 9,090
2 10,450 0.826 8,632
3 11,800 0.751 8,862
4 12,250 0.683 8,367
5 16,750 0.621 10,402
Total present value 45,353
Less: Initial cash outlay -50,000
Net Present Value (NPV) - 4,647
Decision: Since the NPV is negative, so, the asset should not be purchased.
Note: Present value factors can be calculated using the formula—
1
𝑃𝑉𝐹%,𝑛 =
(1 + 𝑘𝑂 )𝑡
1 1 1
𝑃𝑉𝐹10%,1 = = 0.909 𝑃𝑉𝐹10%,2 = = 0.826 𝑃𝑉𝐹10%,3 = = 0.751
(1 + 0.10)1 (1 + 0.10)2 (1 + 0.10)3
1 1
𝑃𝑉𝐹10%,4 = = 0.683 𝑃𝑉𝐹10%,5 = = 0.621
(1 + 0.10)4 (1 + 0.10)5

Example 8
A project involves the following cash flow stream over its life of 5 years:
Initial Investment ₹3,00,000
Cash Inflows:
Year Amount
1 ₹70,000
2 ₹70,000
3 ₹90,000
4 ₹90,000
5 ₹1,10,000
The discount rate may be taken at 12%. Calculate the net present value of the project.

Solution
Year Cash Flows After Tax Present Value Factor at Present Value of
(CFAT) 12% (PVF12%, t)See Note Cash Inflows
1 70,000 0. 893 62,510
2 70,000 0.797 55,790
3 90,000 0.712 64,080
4 90,000 0.636 57,240
5 1,10,000 0.567 62,370
Total present value 3,01,990
Less: Initial cash outlay -3,00,000
Net Present Value (NPV) 1,990
Decision: Since the net present value (NPV) is positive, so, the project shall be undertaken.
Note: Present value factors can be calculated using the formula—
1
𝑃𝑉𝐹%,𝑛 =
(1 + 𝑘𝑂 )𝑡
1 1 1
𝑃𝑉𝐹12%,1 = = 0.893 𝑃𝑉𝐹12%,2 = = 0.797 𝑃𝑉𝐹12%,3 = = 0.712
(1 + 0.12)1 (1 + 0.12)2 (1 + 0.12)3
1 1
𝑃𝑉𝐹12%,4 = = 0.636 𝑃𝑉𝐹12%,5 = = 0.567
(1 + 0.12)4 (1 + 0.12)5

Chapter 5 and 6, Capital Budgeting: 19


VERY IMPORTANT
Example 9
A company proposes to purchase a machine. The cost of the machine is ₹5,00,000 the salvage value
of which is ₹50,000 at the end of 6th year. The working capital required at the time of purchase is
₹1,00,000. Further investment of ₹1,00,000 is also required at the end of 3rd year. The cash inflows
over next 6 years are ₹1,00,000; ₹1,50,000; ₹2,00,000; ₹2,50,000; ₹3,00,000 and ₹3,50,000. The
appropriate discount rate is 12%. Calculate the NPV of the machine and suggest whether to
purchase this machine or not.

Solution

First method
𝑁𝑃𝑉 = 𝑃𝑟𝑒𝑠𝑒𝑛𝑡 𝑉𝑎𝑙𝑢𝑒 𝑜𝑓 𝐶𝑎𝑠ℎ 𝐼𝑛𝑓𝑙𝑜𝑤𝑠 − 𝑃𝑟𝑒𝑠𝑒𝑛𝑡 𝑉𝑎𝑙𝑢𝑒 𝑜𝑓 𝐶𝑎𝑠ℎ 𝑂𝑢𝑡𝑓𝑙𝑜𝑤𝑠
𝑛 𝑛
𝐶𝐹𝐴𝑇𝑡 𝐶𝑂𝑡
𝑁𝑃𝑉 = ∑ − ∑
(1 + 𝑘𝑂 )𝑡 (1 + 𝑘𝑂 )𝑡
𝑡=1 𝑡=0
𝑛=6 𝑛=6
𝐶𝐹𝐴𝑇𝑡 𝐶𝑂𝑡
𝑁𝑃𝑉 = ∑ 𝑡
−∑
(1 + 0.12) (1 + 0.12)𝑡
𝑡=1 𝑡=0
1 1 1
𝑁𝑃𝑉 = [𝐶𝐹𝐴𝑇1 × 1
+ 𝐶𝐹𝐴𝑇2 × 2
+ 𝐶𝐹𝐴𝑇3 ×
(1 + 0.12) (1 + 0.12) (1 + 0.12)3
1 1 1
+ 𝐶𝐹𝐴𝑇4 × + 𝐶𝐹𝐴𝑇 5 × × 𝐶𝐹𝐴𝑇 6 ×
(1 + 0.12)4 (1 + 0.12)5 (1 + 0.12)6
1 1
+ 𝐶𝐹𝐴𝑇6 × + 𝐶𝐹𝐴𝑇6 × ]
(1 + 0.12)6 (1 + 0.12)6
1 1 1
− [𝐶𝑂0 × 0
+ 𝐶𝑂0 × 0
+ 𝐶𝑂3 × ]
(1 + 0.12) (1 + 0.12) (1 + 0.12)3
𝑁𝑃𝑉 = [1,00,000 × 0.893 + 1,50,000 × 0.797 + 2,00,000 × 0.712 + 2,50,000 × 0.636
+ 3,00,000 × 0.567 + 3,50,000 × 0.507 + 50,000 × 0.507
+ 1,00,000 × 0.507]
− [5,00,000 × 1.000 + 1,00,000 × 1.000 + 1,00,000 × 0.712]
𝑁𝑃𝑉 = [89,300 + 1,19,550 + 1,42,400 + 1,59,000 + 1,70,100 + 1,77,450 + 25,350
+ 50,700] − [5,00,000 + 1,00,000 + 71,200]
𝑁𝑃𝑉 = 9,33,850 − 6,71,200
𝑁𝑃𝑉 = ₹2,62,650
Decision: Since the NPV is positive, so, the machine should be purchased.
Note−1:
Cash inflows Cash outflows
Year Amount Remarks Year Amount Remarks
0 -- -- 0 5,00,000 Cost of the machine
1 1,00,000 Cash inflows 0 1,00,000 Working capital required
2 1,50,000 Cash inflows 3 1,00,000 Additional investment
3 2,00,000 Cash inflows
4 2,50,000 Cash inflows
5 3,00,000 Cash inflows
6 3,50,000 Cash inflows
6 50,000 Salvage value recovered
6 1,00,000 Working capital released

Second method
Cash inflows Cash outflows
Year Amount Present Value Present Year Amount Present Value Present
Factor at 12% Value of Factors at 12% Value of
(PVF12%, t)See Cash (PVF12%, t) See Cash
Note-2 Inflows Note-2 Outflows

Chapter 5 and 6, Capital Budgeting: 20


1 1,00,000 0.893 89,300 0 5,00,000 1.000 5,00,000
2 1,50,000 0.797 1,19,550 0 1,00,000 1.000 1,00,000
3 2,00,000 0.712 1,42,400 3 1,00,000 0.712 71,200
4 2,50,000 0.636 1,59,000
5 3,00,000 0.567 1,70,100
6 3,50,000 0.507 1,77,450
6 50,000 0.507 25,350
6 1,00,000 0.507 50,700
Present Value of Cash Inflows 9,33,850 Present value of Cash Outflows 6,71,200
𝑁𝑃𝑉 = 𝑃𝑟𝑒𝑠𝑒𝑛𝑡 𝑉𝑎𝑙𝑢𝑒 𝑜𝑓 𝐶𝑎𝑠ℎ 𝐼𝑛𝑓𝑙𝑜𝑤𝑠 − 𝑃𝑟𝑒𝑠𝑒𝑛𝑡 𝑉𝑎𝑙𝑢𝑒 𝑜𝑓 𝐶𝑎𝑠ℎ 𝑂𝑢𝑡𝑓𝑙𝑜𝑤𝑠
𝑁𝑃𝑉 = 9,33,850 − 6,71,200
𝑁𝑃𝑉 = ₹2,62,650

Decision: Since the NPV is positive, so, the machine should be purchased.
Note−2: Present value factors can be calculated using the formula:
1
𝑃𝑉𝐹%,𝑛 =
(1 + 𝑘𝑂 )𝑡
1 1 1
𝑃𝑉𝐹12%,0 = = 1.000 𝑃𝑉𝐹12%,1 = = 0.893 𝑃𝑉𝐹12%,2 = = 0.797
(1 + 0.12)0 (1 + 0.12)1 (1 + 0.12)2
1 1 1
𝑃𝑉𝐹12%,3 = = 0.712 𝑃𝑉𝐹12%,4 = = 0.636 𝑃𝑉𝐹12%,5 = = 0.567
(1 + 0.12)3 (1 + 0.12)4 (1 + 0.12)5
1
𝑃𝑉𝐹12%,6 = = 0.507
(1 + 0.12)6

4. Profitability Index (PI)/Benefit to Cost (BC) Ratio Method


Profitability index is calculated by dividing the present value of the cash inflows by the present value
of the cash outflows.

Decision Rule

Accept/reject criteria
If Project’s 𝑃𝐼 𝑜𝑟 𝐵𝐶 𝑟𝑎𝑡𝑖𝑜 < 1: 𝑅𝑒𝑗𝑒𝑐𝑡 𝑡ℎ𝑒 𝑃𝑟𝑜𝑗𝑒𝑐𝑡
If Project’s 𝑃𝐼 𝑜𝑟 𝐵𝐶 𝑟𝑎𝑡𝑖𝑜 > 1: 𝐴𝑐𝑐𝑒𝑝𝑡 𝑡ℎ𝑒 𝑃𝑟𝑜𝑗𝑒𝑐𝑡

Mutually exclusive projects


Select the project with the highest PI or BC ratio.

Capital rationing
Rank the projects from the highest PI to the lowest and then invest accordingly.
Maximum PI or BC ratio Rank I 
Moderate PI or BC ratio Rank II 
Minimum PI or BC ratio Rank III 

Merits and demerits

Merits
1. PI method differentiates between cash flows occurring at different points of time and thus,
considers time value of money.
2. PI like NPV is based on cash flows rather than accounting profit.
3. PI also incorporates the riskiness of the project in capital budgeting analysis through use of
an appropriate discount rate.
4. PI tells the present value of cash inflows generated per rupee of cash outflow. It thus, can
identify whether a project would increase firm’s value or shareholder’s wealth.

Chapter 5 and 6, Capital Budgeting: 21


5. PI technique is very useful in ranking the projects and choosing projects which give
maximum cash inflows for a given amount of capital investment or cash outflow in case of
capital rationing.

Demerits
1. It is difficult to compute as compared to the traditional methods of capital budgeting such as
ARR and Payback period method.
2. It requires computation of required rate of return to be used to discount future cash flows.
3. Computations of present values and forecasting of future cash flows is needed.

How to compute PI/BC Ratio?


𝑃𝑟𝑒𝑠𝑒𝑛𝑡 𝑣𝑎𝑙𝑢𝑒 𝑜𝑓 𝑡ℎ𝑒 𝑐𝑎𝑠ℎ 𝑖𝑛𝑓𝑙𝑜𝑤𝑠
𝑃𝐼 𝑜𝑟 𝐵𝐶 𝑅𝑎𝑡𝑖𝑜 =
𝑃𝑟𝑒𝑠𝑒𝑛𝑡 𝑣𝑎𝑙𝑢𝑒 𝑜𝑓 𝑡ℎ𝑒 𝑐𝑎𝑠ℎ 𝑜𝑢𝑡𝑓𝑙𝑜𝑤𝑠

5. Internal Rate of Return (IRR) Method

Meaning
Internal rate of return is that discount rate which equates the present value of the cash
inflows with the present value of the cash outflows. In other words, we can say that Internal
Rate of Return is that rate of discount at which the Net Present Value is Zero. IRR is also called the
marginal efficiency of capital, marginal productivity of capital, yield on investment, time adjusted
rate of return, marginal rate of return, etc.
At this discount rate the following condition is satisfied—
𝑃𝑟𝑒𝑠𝑒𝑛𝑡 𝑣𝑎𝑙𝑢𝑒 𝑜𝑓 𝑐𝑎𝑠ℎ 𝑖𝑛𝑓𝑙𝑜𝑤𝑠 = 𝑃𝑟𝑒𝑠𝑒𝑛𝑡 𝑣𝑎𝑙𝑢𝑒 𝑜𝑓 𝑐𝑎𝑠ℎ 𝑜𝑢𝑡𝑓𝑙𝑜𝑤𝑠 𝑁𝑜𝑡𝑒
⇒ 𝑃𝑟𝑒𝑠𝑒𝑛𝑡 𝑣𝑎𝑙𝑢𝑒 𝑜𝑓 𝑐𝑎𝑠ℎ 𝑖𝑛𝑓𝑙𝑜𝑤𝑠 − 𝑃𝑟𝑒𝑠𝑒𝑛𝑡 𝑣𝑎𝑙𝑢𝑒 𝑜𝑓 𝑐𝑎𝑠ℎ 𝑜𝑢𝑡𝑓𝑙𝑜𝑤𝑠 𝑁𝑜𝑡𝑒 = 0
⇒ 𝑁𝑃𝑉 = 0
Symbolically,
𝑛 𝑛
𝐶𝐹𝐴𝑇𝑡 𝐶𝑂𝑡
∑ 𝑡
=∑
(1 + 𝑘𝑂 ) (1 + 𝑘𝑂 )𝑡
𝑡=1 𝑡=0
𝑛 𝑛
𝐶𝐹𝐴𝑇𝑡 𝐶𝑂𝑡
⇒∑ 𝑡
−∑ =0
(1 + 𝑘𝑂 ) (1 + 𝑘𝑂 )𝑡
𝑡=1 𝑡=0
⇒ 𝑁𝑃𝑉 = 0
Suppose we put 10% in place of kO and both sides become equal or NPV would be zero. Then this
10% is the internal rate of return.
Note: If cash outflows are there in the zero year only (multiple cash outflows are not there) then
there is no need to calculate the present value of the cash outflows.

Use of internal rate of return


Internal rate of return is used to evaluate the decisions related to long-term assets purchase.
Basically, it is the actual return that a project or machine earns.

Decision Rule
Following rules are used to evaluate the project or machine.

Accept/reject criteria
At this stage we select as many assets as possible. Basically, we create a pool of so many assets
which are under consideration. The selection of so many assets does not mean that all of these will
be purchased. We will evaluate the selected assets under either mutually exclusive or capital
rationing criteria.
𝐼𝑓 𝐴𝑠𝑠𝑒𝑡’𝑠 𝐼𝑅𝑅 < 𝑆𝑡𝑎𝑛𝑑𝑎𝑟𝑑 𝑅𝑎𝑡𝑒 𝑜𝑓 𝑅𝑒𝑡𝑢𝑟𝑛 = 𝐷𝑜 𝑛𝑜𝑡 𝑠𝑒𝑙𝑒𝑐𝑡 𝑡ℎ𝑒 𝑎𝑠𝑠𝑒𝑡
𝐼𝑓 𝐴𝑠𝑠𝑒𝑡’𝑠 𝐼𝑅𝑅 > 𝑆𝑡𝑎𝑛𝑑𝑎𝑟𝑑 𝑅𝑎𝑡𝑒 𝑜𝑓 𝑅𝑒𝑡𝑢𝑟𝑛 = 𝑆𝑒𝑙𝑒𝑐𝑡 𝑡ℎ𝑒 𝑎𝑠𝑠𝑒𝑡

Chapter 5 and 6, Capital Budgeting: 22


Mutually exclusive projects
Projects or assets are said to be mutually exclusive when the selection of one project leads to the
rejection of other projects. Only one project or asset is to be purchased which is the best one.
Select the project with the highest IRR (But we do not suggest relying solely on the results
given by the IRR rather one should consider and prefer the NPV)

Capital rationing
When there are many projects but investment is to be made in some projects then they are ranked
according to their profitability. And then investment is made accordingly i.e., from highest
profitability to lowest profitability. It must be noted that those projects are not considered here
which were rejected earlier under the Accept/Reject criterion.
Rank the projects from the highest IRR to the lowest and then invest accordingly. (But we do not
suggest relying solely on the ranks given by the IRR rather one should consider and prefer the
ranking given by the NPV)
Maximum IRR Rank I 
Moderate IRR Rank II 
Minimum IRR Rank III 

Advantages of IRR
1. IRR technique takes into account the time value of money and accordingly different cash
flows are given different importance depending on the time period when they arise.
2. It is related to the wealth maximization objective which is the main aim of modern financial
management. This is because when we accept an investment proposal which has internal
return in excess of cost of capital, this additional return results in the generation of more
wealth to shareholders.
3. This technique is also based on all the cash flows of a project just like NPV. The complete life
of the project is taken into account before any investment decision is made which results in
sound investment decision.
4. IRR uses cash flows to arrive at the rate of return rather than the accounting profits.

Disadvantages of IRR
1. The computation of IRR is tedious and at times it becomes very complex as well.
2. There are some situations when we can have more than one IRR for the project and in some
other situations, we have no IRR value. We will consider these situations.
3. One of the main limitations of IRR is related to the assumption under which it is calculated.
The assumption underlying the computation of IRR is that the amount of cash inflows which
is received in intermediate time period is reinvested at the rate which is equal to the
internal rate of return. This assumption is also called reinvestment rate assumption. It
means if the IRR of project is 20%, company is in a position to invest the intermediate cash
flows at 20% which is not feasible. The firms are able to invest the cash inflows only at the
rate which is available in the market.

How do we compute the IRR?


There are two types of streams of cash flows—(a) Annuity stream and (b) Mixed stream.
Annuity stream: When cash flows to be received each year are constant, this is called annuity
stream.
Mixed stream: When the cash flows to be received each year are not constant, then this is called
mixed stream.
In case of annuity stream the procedure is simple but in case of mixed stream the procedure is
slightly complicated and we have to use the Trial-and-Error method.

Annuity Stream
Step I Calculate the Fake Payback Period (FPBP)—

Chapter 5 and 6, Capital Budgeting: 23


𝐼𝑛𝑡𝑖𝑎𝑙 𝑂𝑢𝑡𝑙𝑎𝑦 𝑜𝑟 𝐼𝑛𝑖𝑡𝑖𝑎𝑙 𝐼𝑛𝑣𝑒𝑠𝑡𝑚𝑒𝑛𝑡
𝐶𝑜𝑛𝑠𝑡𝑎𝑛𝑡 𝐶𝑎𝑠ℎ 𝐼𝑛𝑓𝑙𝑜𝑤𝑠
Step II Now locate the fake payback period in the Present Value Annuity Factors Table
(see Annexure-Table A-4) corresponding to the life of the project. If you have
successfully located the exact value of the fake payback period then note down the
percentage corresponding to the located fake payback period. This percentage is the
IRR.

But so many times it is not possible to locate the exact value of the fake payback
period in the table. The value of the fake payback period lies between two values.
Note down the corresponding percentages to these two values and go to the next
step.
Step III (i) Compute the Present Value and Net Present Value at lower discount rate
(ii) Compute the Present Value and Net Present Value at higher discount rate

Note:
𝑃𝑉 = 𝑆𝑖𝑛𝑔𝑙𝑒 𝑐𝑎𝑠ℎ 𝑖𝑛𝑓𝑙𝑜𝑤 × 𝑃𝑉𝐴𝐹𝑟,𝑛
𝑁𝑃𝑉 = (𝑆𝑖𝑛𝑔𝑙𝑒 𝑐𝑎𝑠ℎ 𝑖𝑛𝑓𝑙𝑜𝑤 × 𝑃𝑉𝐴𝐹𝑟,𝑛 ) − 𝐶𝑎𝑠ℎ 𝑜𝑢𝑡𝑓𝑙𝑜𝑤𝑠
Step IV Now apply the following formula:
𝑃𝑉𝐿𝐷𝑅 − 𝑃𝑉 𝑜𝑓 𝑐𝑎𝑠ℎ 𝑜𝑢𝑡𝑓𝑙𝑜𝑤𝑠 𝑜𝑟 𝐼𝑛𝑖𝑡𝑖𝑎𝑙 𝑜𝑢𝑡𝑙𝑎𝑦
𝐼𝑅𝑅 = 𝐿𝐷𝑅 +
𝑃𝑉𝐿𝐷𝑅 − 𝑃𝑉𝐻𝐷𝑅
× |𝐷𝑖𝑓𝑓𝑒𝑟𝑒𝑛𝑐𝑒 𝑜𝑓 𝑅𝑎𝑡𝑒𝑠|
or
𝑁𝑃𝑉𝐿𝐷𝑅
𝐼𝑅𝑅 = 𝐿𝐷𝑅 + × |𝐷𝑖𝑓𝑓𝑒𝑟𝑒𝑛𝑐𝑒 𝑜𝑓 𝑅𝑎𝑡𝑒𝑠|
𝑁𝑃𝑉𝐿𝐷𝑅 − 𝑁𝑃𝑉𝐻𝐷𝑅
or
𝐹𝑎𝑐𝑡𝑜𝑟𝐻𝐷𝑅 − 𝐹𝑎𝑘𝑒 𝑝𝑎𝑦𝑏𝑎𝑐𝑘 𝑝𝑒𝑟𝑖𝑜𝑑
𝐼𝑅𝑅 = HDR
𝐿𝐷𝑅 +- × |𝐷𝑖𝑓𝑓𝑒𝑟𝑒𝑛𝑐𝑒 𝑜𝑓 𝑅𝑎𝑡𝑒𝑠|
𝐹𝑎𝑐𝑡𝑜𝑟𝐻𝐷𝑅 − 𝐹𝑎𝑐𝑡𝑜𝑟𝐿𝐷𝑅
(The above formulae are for the linear interpolation)
𝑁𝑃𝑉 = 𝑁𝑒𝑡 𝑝𝑟𝑒𝑠𝑒𝑛𝑡 𝑣𝑎𝑙𝑢𝑒
𝐿𝐷𝑅 = 𝐿𝑜𝑤𝑒𝑟 𝐷𝑖𝑠𝑐𝑜𝑢𝑛𝑡 𝑅𝑎𝑡𝑒
𝐻𝐷𝑅 = 𝐻𝑖𝑔ℎ𝑒𝑟 𝐷𝑜𝑠𝑐𝑜𝑢𝑛𝑡 𝑅𝑎𝑡𝑒
Difference of rates is taken after ignoring the signs.

Mixed Stream
In case of mixed stream as we already have discussed earlier the Trial-and-Error approach shall be
used. In this approach take a percentage and compute the present value of the cash inflows of the
project. If the present value of cash inflows is equal to the present value of cash outflows (or cash
outflows in the zero year) then the percentage taken is the IRR.
But in almost all the cases of the mixed stream we cannot get the single percentage at which the
present value of cash inflows is equal to the present value of the cash outflows (or cash outflows in
the zero year). So, first of all take a percentage and calculate the present value of the cash inflows. If
the present value of cash inflows is less than the present value of the cash outflows (or cash
outflows in the zero year) then reduce the percentage or if the present value of cash inflows is more
than the present value of the cash outflows (or cash outflows in the zero year) then increase the
percentage. Arrive at two percentages in such a way that at one percentage (Lower) the present
value of cash inflows is more than the present value of the cash outflows (or cash outflows in the
zero year) and at another percentage (Higher) the present value of cash inflows is less than the
present value of the cash outflows (or cash outflows in the zero year). VERY
IRR can also be computed using NPV IMPORTANT
Take a percentage and compute the NPV of the project. If at this percentage the NPV is zero then
this is the IRR.
But in almost all the cases of the mixed stream we cannot get the single percentage at which the
NPV is zero. Now take a percentage and calculate the NPV. If the NPV is negative then reduce the
percentage to get a positive NPV and if the NPV is positive then increase the percentage to get a

Chapter 5 and 6, Capital Budgeting: 24


negative NPV. Now arrive at the two percentages—at one percentage (Lower) the NPV should be
positive and at other (Higher) it should be negative.
Now apply the following formula—
𝑃𝑉𝐿𝐷𝑅 − 𝑃𝑉 𝑜𝑓 𝑐𝑎𝑠ℎ 𝑜𝑢𝑡𝑓𝑙𝑜𝑤𝑠 𝑜𝑟 𝐼𝑛𝑖𝑡𝑖𝑎𝑙 𝑜𝑢𝑡𝑙𝑎𝑦
𝐼𝑅𝑅 = 𝐿𝐷𝑅 + × |𝐷𝑖𝑓𝑓𝑒𝑟𝑒𝑛𝑐𝑒 𝑜𝑓 𝑅𝑎𝑡𝑒𝑠|
𝑃𝑉𝐿𝐷𝑅 − 𝑃𝑉𝐻𝐷𝑅
or
𝑁𝑃𝑉𝐿𝐷𝑅
𝐼𝑅𝑅 = 𝐿𝐷𝑅 + × |𝐷𝑖𝑓𝑓𝑒𝑟𝑒𝑛𝑐𝑒 𝑜𝑓 𝑅𝑎𝑡𝑒𝑠|
𝑁𝑃𝑉𝐿𝐷𝑅 − 𝑁𝑃𝑉𝐻𝐷𝑅
(The above formulae are for the linear interpolation)
Where,
𝑁𝑃𝑉 = 𝑁𝑒𝑡 𝑝𝑟𝑒𝑠𝑒𝑛𝑡 𝑣𝑎𝑙𝑢𝑒
𝑃𝑉 = 𝑃𝑟𝑒𝑠𝑒𝑛𝑡 𝑣𝑎𝑙𝑢𝑒
𝐿𝐷𝑅 = 𝐿𝑜𝑤𝑒𝑟 𝐷𝑖𝑠𝑐𝑜𝑢𝑛𝑡 𝑅𝑎𝑡𝑒
𝐻𝐷𝑅 = 𝐻𝑖𝑔ℎ𝑒𝑟 𝐷𝑖𝑠𝑐𝑜𝑢𝑛𝑡 𝑅𝑎𝑡𝑒
Difference of rates is taken after ignoring the signs.
Notes:
1. So many times, selection of the percentage is a problem for the students. They are not able
to decide the percentage with which to start the working. Take any percentage and compute
the PV of cash inflows or NPV. Now the PV of cash inflows or NPV will give you the direction
regarding the next percentage.
2. It is the question of interest that what should be the difference between two percentages. It
may be 1, 2, 3, 4 or even 5. But we are of the opinion that this difference should be of 1
because at this difference the approximation is the best. In the examination the difference
up to 5% is acceptable and it helps in solving the question in minimum time.

Example 10
The cost of a machine is ₹37,910. Cash inflows are ₹10,000 per annum for a period of 5 years.
Calculate the IRR.

Solution
Step I: Calculate the Fake Payback Period (FPBP)
𝐼𝑛𝑡𝑖𝑎𝑙 𝑂𝑢𝑡𝑙𝑎𝑦 𝑜𝑟 𝐼𝑛𝑖𝑡𝑖𝑎𝑙 𝐼𝑛𝑣𝑒𝑠𝑡𝑚𝑒𝑛𝑡 37,910
= = 3.791
𝐶𝑜𝑛𝑠𝑡𝑎𝑛𝑡 𝐶𝑎𝑠ℎ 𝐼𝑛𝑓𝑙𝑜𝑤𝑠 10,000

Step II: Now locate the Fake Payback Period which is 3.791 in the Present Value Annuity Factors
Table (see annexure-Table A-4) corresponding to the life of the machine. Corresponding to the life
of the machine (i.e., 5 years) the value of 3.791 is against the 10% in the table. We have successfully
located the value of the fake payback period. So, IRR is 10%.
Note to step II:
𝑁𝑃𝑉 = (𝑆𝑖𝑛𝑔𝑙𝑒 𝑐𝑎𝑠ℎ 𝑖𝑛𝑓𝑙𝑜𝑤 × 𝑃𝑉𝐴𝐹10%,5 ) − 𝐶𝑎𝑠ℎ 𝑜𝑢𝑡𝑓𝑙𝑜𝑤𝑠 ⇒ (10,000 × 3.791) − 37,910 = 0.
NPV is zero at 10%, so, the IRR is 10%.

Step III:
Not applicable.

Step IV:
Not applicable.

Example 11
The cost of a machine is ₹3,00,000. Cash inflows are ₹1,00,000 per annum for a period of 5 years.
Calculate the IRR.

Solution
Step I: Calculate the Fake Payback Period (FPBP)

Chapter 5 and 6, Capital Budgeting: 25


𝐼𝑛𝑡𝑖𝑎𝑙 𝑂𝑢𝑡𝑙𝑎𝑦 𝑜𝑟 𝐼𝑛𝑖𝑡𝑖𝑎𝑙 𝐼𝑛𝑣𝑒𝑠𝑡𝑚𝑒𝑛𝑡 3,00,000
= =3
𝐶𝑜𝑛𝑠𝑡𝑎𝑛𝑡 𝐶𝑎𝑠ℎ 𝐼𝑛𝑓𝑙𝑜𝑤𝑠 1,00,000

Step II: Now locate the Fake Payback Period which is 3 in the Present Value Annuity Factors
Table (see annexure-Table A-4) corresponding to the life of the machine. Corresponding to the life
of the machine there are two values viz. 3.058 (at 19%) and 2.991 (at 20%). So, the fake payback
period of 3 lies between 3.058 and 2.991 hence, the IRR lies between 20% and 19%. Now proceed
to the next step.

Step III:
(i) Compute the Present Value and Net Present Value at lower discount rate i.e., 19%:
𝑁𝑃𝑉 = 𝑃𝑟𝑒𝑠𝑒𝑛𝑡 𝑣𝑎𝑙𝑢𝑒 𝑜𝑓 𝑐𝑎𝑠ℎ 𝑖𝑛𝑓𝑙𝑜𝑤𝑠 − 𝑃𝑟𝑒𝑠𝑒𝑛𝑡 𝑣𝑎𝑙𝑢𝑒 𝑜𝑓 𝑐𝑎𝑠ℎ 𝑜𝑢𝑡𝑓𝑙𝑜𝑤𝑠
𝑁𝑃𝑉 = 𝐴𝑛𝑛𝑢𝑎𝑙 𝐶𝑎𝑠ℎ 𝐼𝑛𝑓𝑙𝑜𝑤𝑠 × 𝑃𝑉𝐴𝐹𝑘𝑂,𝑡 − 𝐶𝑎𝑠ℎ 𝑂𝑢𝑡𝑓𝑙𝑜𝑤𝑠 𝑖𝑛 𝑡ℎ𝑒 𝑍𝑒𝑟𝑜 𝑌𝑒𝑎𝑟
𝑁𝑃𝑉 = 𝐴𝑛𝑛𝑢𝑎𝑙 𝐶𝑎𝑠ℎ 𝐼𝑛𝑓𝑙𝑜𝑤𝑠 × 𝑃𝑉𝐴𝐹19%,5 − 𝐶𝑎𝑠ℎ 𝑂𝑢𝑡𝑓𝑙𝑜𝑤𝑠 𝑖𝑛 𝑡ℎ𝑒 𝑍𝑒𝑟𝑜 𝑌𝑒𝑎𝑟
1
1−
(1 + 𝑘𝑂 )𝑡
𝑁𝑃𝑉 = 𝐴𝑛𝑛𝑢𝑎𝑙 𝐶𝑎𝑠ℎ 𝐼𝑛𝑓𝑙𝑜𝑤𝑠 × [ ] − 𝐶𝑎𝑠ℎ 𝑂𝑢𝑡𝑓𝑙𝑜𝑤𝑠 𝑖𝑛 𝑡ℎ𝑒 𝑍𝑒𝑟𝑜 𝑌𝑒𝑎𝑟
𝑘𝑂
1
1−
(1 + 0.19)5
𝑁𝑃𝑉 = ₹1,00,000 × [ ] − 𝑅𝑠. 3,00,000
0.19

𝑁𝑃𝑉 = ₹1,00,000 × 3.058 − ₹3,00,000


𝑁𝑃𝑉 = ₹3,05,800 − ₹3,00,000
𝑁𝑃𝑉 = ₹5,800
(ii) Compute the Present Value and Net Present Value at higher discount rate i.e., 20%:
𝑁𝑃𝑉 = 𝑃𝑟𝑒𝑠𝑒𝑛𝑡 𝑣𝑎𝑙𝑢𝑒 𝑜𝑓 𝑐𝑎𝑠ℎ 𝑖𝑛𝑓𝑙𝑜𝑤𝑠 − 𝑃𝑟𝑒𝑠𝑒𝑛𝑡 𝑣𝑎𝑙𝑢𝑒 𝑜𝑓 𝑐𝑎𝑠ℎ 𝑜𝑢𝑡𝑓𝑙𝑜𝑤𝑠
𝑁𝑃𝑉 = 𝐴𝑛𝑛𝑢𝑎𝑙 𝐶𝑎𝑠ℎ 𝐼𝑛𝑓𝑙𝑜𝑤𝑠 × 𝑃𝑉𝐴𝐹𝑘𝑂,𝑡 − 𝐶𝑎𝑠ℎ 𝑂𝑢𝑡𝑓𝑙𝑜𝑤𝑠 𝑖𝑛 𝑡ℎ𝑒 𝑍𝑒𝑟𝑜 𝑌𝑒𝑎𝑟
𝑁𝑃𝑉 = 𝐴𝑛𝑛𝑢𝑎𝑙 𝐶𝑎𝑠ℎ 𝐼𝑛𝑓𝑙𝑜𝑤𝑠 × 𝑃𝑉𝐴𝐹20%,5 − 𝐶𝑎𝑠ℎ 𝑂𝑢𝑡𝑓𝑙𝑜𝑤𝑠 𝑖𝑛 𝑡ℎ𝑒 𝑍𝑒𝑟𝑜 𝑌𝑒𝑎𝑟
1
1−
(1 + 𝑘𝑂 )𝑡
𝑁𝑃𝑉 = 𝐴𝑛𝑛𝑢𝑎𝑙 𝐶𝑎𝑠ℎ 𝐼𝑛𝑓𝑙𝑜𝑤𝑠 × [ ] − 𝐶𝑎𝑠ℎ 𝑂𝑢𝑡𝑓𝑙𝑜𝑤𝑠 𝑖𝑛 𝑡ℎ𝑒 𝑍𝑒𝑟𝑜 𝑌𝑒𝑎𝑟
𝑘𝑂
1
1−
(1 + 0.19)5
𝑁𝑃𝑉 = ₹1,00,000 × [ ] − ₹3,00,000
0.19

𝑁𝑃𝑉 = ₹1,00,000 × 2.991 − ₹3,00,000


𝑁𝑃𝑉 = ₹2,99,100 − ₹3,00,000
𝑁𝑃𝑉 = (₹900)

Step IV:
Now apply the following formula to interpolate the IRR:
𝑁𝑃𝑉𝐿𝐷𝑅
𝐼𝑅𝑅 (𝑟) = 𝐿𝐷𝑅 + × |𝐷𝑖𝑓𝑓𝑒𝑟𝑒𝑛𝑐𝑒 𝑜𝑓 𝑅𝑎𝑡𝑒𝑠|
𝑁𝑃𝑉𝐿𝐷𝑅 − 𝑁𝑃𝑉𝐻𝐷𝑅
𝑁𝑃𝑉19%
𝐼𝑅𝑅 (𝑟) = 19 + ×1
𝑁𝑃𝑉19% − 𝑁𝑃𝑉20%
₹ 5,800
𝐼𝑅𝑅 (𝑟) = 19 + ×1
₹5,800 − (−₹900)
𝐼𝑅𝑅 (𝑟) = 19 + 0.866
𝐼𝑅𝑅 (𝑟) = 19.866
or
𝑃𝑉𝐿𝐷𝑅 − 𝑃𝑉 𝑜𝑓 𝑐𝑎𝑠ℎ 𝑜𝑢𝑡𝑓𝑙𝑜𝑤𝑠 𝑜𝑟 𝐼𝑛𝑖𝑡𝑖𝑎𝑙 𝑜𝑢𝑡𝑙𝑎𝑦
𝐼𝑅𝑅 (𝑟) = 𝐿𝐷𝑅 + × |𝐷𝑖𝑓𝑓𝑒𝑟𝑒𝑛𝑐𝑒 𝑜𝑓 𝑅𝑎𝑡𝑒𝑠|
𝑃𝑉𝐿𝐷𝑅 − 𝑃𝑉𝐻𝐷𝑅
𝑃𝑉19% − 𝑃𝑉 𝑜𝑓 𝑐𝑎𝑠ℎ 𝑜𝑢𝑡𝑓𝑙𝑜𝑤𝑠 𝑜𝑟 𝐼𝑛𝑖𝑡𝑖𝑎𝑙 𝑜𝑢𝑡𝑙𝑎𝑦
𝐼𝑅𝑅 (𝑟) = 19 + ×1
𝑃𝑉19% − 𝑃𝑉20%

Chapter 5 and 6, Capital Budgeting: 26


3,05,800 − 3,00,000
𝐼𝑅𝑅 (𝑟) = 19 + ×1
3,05,800 − 2,99,100
𝐼𝑅𝑅 (𝑟) = 19 + 0.866
𝐼𝑅𝑅 (𝑟) = 19.866
or
𝐹𝑎𝑐𝑡𝑜𝑟𝐻𝐷𝑅 − 𝐹𝑎𝑘𝑒 𝑃𝑎𝑦𝑏𝑎𝑐𝑘 𝑃𝑒𝑟𝑖𝑜𝑑
𝐼𝑅𝑅 (𝑟) = 𝐿𝐷𝑅 + × |𝐷𝑖𝑓𝑓𝑒𝑟𝑒𝑛𝑐𝑒 𝑜𝑓 𝑅𝑎𝑡𝑒𝑠|
𝐹𝑎𝑐𝑡𝑜𝑟𝐻𝐷𝑅 − 𝐹𝑎𝑐𝑡𝑜𝑟𝐿𝐷𝑅
𝐹𝑎𝑐𝑡𝑜𝑟20% − 𝐹𝑎𝑘𝑒 𝑃𝑎𝑦𝑏𝑎𝑐𝑘 𝑃𝑒𝑟𝑖𝑜𝑑
𝐼𝑅𝑅 (𝑟) = 19 + ×1
𝐹𝑎𝑐𝑡𝑜𝑟20% − 𝐹𝑎𝑐𝑡𝑜𝑟19%
2.991 − 3
𝐼𝑅𝑅 (𝑟) = 19 + ×1
2.991 − 3.058
𝐼𝑅𝑅 (𝑟) = 19 + 0.866
𝐼𝑅𝑅 (𝑟) = 19.866
Note:
In Excel® the IRR comes out 19.858%.

Example 12
The cost of a machine is ₹3,00,000. Cash inflows are as follows—

Year Amount
1 ₹70,000
2 ₹70,000
3 ₹90,000
4 ₹90,000
5 ₹90,000
Calculate the IRR.

Solution
In this question mixed stream is given, so, trial and error method will be used. Let us calculate the
NPV at the discount rate of 10%.
Year Cash Inflows Present Value Factors at 10% (PVF10%, t) Present Value at 10%
1 70,000 0.909 63,630
2 70,000 0.826 57,820
3 90,000 0.751 67,590
4 90,000 0.683 61,470
5 90,000 0.621 55,890
Total present value 3,06,400
Less: Initial cash outlay -3,00,000
Net Present Value (NPV) 6,400
Since NPV is positive, so, we shall increase the discount rate. Now let us calculate the NPV at the
discount rate of 11%.
Year Cash Inflows Present Value Factors at 11% (PVF11%, t) Present Value at 11%
1 70,000 0.901 63,070
2 70,000 0.812 56,840
3 90,000 0.731 65,790
4 90,000 0.659 59,310
5 90,000 0.593 53,370
Total present value 2,98,380
Less: Initial cash outlay -3,00,000
Net Present Value (NPV) -1,620
Now the NPV is negative at 11% and positive at 10%. It means that the zero NPV lies between
(₹1,620) and ₹6,400 and IRR lies between 11% and 10%. So, there is need to apply the formula for
the interpolation.
𝑁𝑃𝑉𝐿𝐷𝑅
𝐼𝑅𝑅 = 𝐿𝐷𝑅 + × |𝐷𝑖𝑓𝑓𝑒𝑟𝑒𝑛𝑐𝑒 𝑜𝑓 𝑅𝑎𝑡𝑒𝑠|
𝑁𝑃𝑉𝐿𝐷𝑅 − 𝑁𝑃𝑉𝐻𝐷𝑅

Chapter 5 and 6, Capital Budgeting: 27


𝑁𝑃𝑉10%
𝐼𝑅𝑅 = 10 + ×1
𝑁𝑃𝑉10% − 𝑁𝑃𝑉11%
₹6,400
𝐼𝑅𝑅 = 10 + ×1
₹6,400 − (−₹ 1,620)
𝐼𝑅𝑅 = 10 + 0.798
𝐼𝑅𝑅 = 10.798
𝐼𝑅𝑅 ≅ 10.80
or
𝑃𝑉𝐿𝐷𝑅 − 𝑃𝑉 𝑜𝑓 𝑐𝑎𝑠ℎ 𝑜𝑢𝑡𝑓𝑙𝑜𝑤𝑠 𝑜𝑟 𝐼𝑛𝑖𝑡𝑖𝑎𝑙 𝑜𝑢𝑡𝑙𝑎𝑦
𝐼𝑅𝑅 = 𝐿𝐷𝑅 + × |𝐷𝑖𝑓𝑓𝑒𝑟𝑒𝑛𝑐𝑒 𝑜𝑓 𝑅𝑎𝑡𝑒𝑠|
𝑃𝑉𝐿𝐷𝑅 − 𝑃𝑉𝐻𝐷𝑅
𝑃𝑉10% − 𝑃𝑉 𝑜𝑓 𝑐𝑎𝑠ℎ 𝑜𝑢𝑡𝑓𝑙𝑜𝑤𝑠 𝑜𝑟 𝐼𝑛𝑖𝑡𝑖𝑎𝑙 𝑜𝑢𝑡𝑙𝑎𝑦
𝐼𝑅𝑅 = 10 + ×1
𝑃𝑉10% − 𝑃𝑉11%
3,06,400 − 3,00,000
𝐼𝑅𝑅 = 10 + ×1
3,06,400 − 2,98,380
𝐼𝑅𝑅 = 10 + 0.798
𝐼𝑅𝑅 = 10.798
𝐼𝑅𝑅 ≅ 10.80
Note:
In Excel® the IRR comes out 10.796%.

Example 13
The cost of a machine is ₹42,000. Cash inflows are as follows:

Year Amount
1 ₹12,000
2 ₹12,000
3 ₹14,000
4 ₹15,000
5 ₹16,000
Calculate the IRR.

Solution
In this question mixed stream is given, so, trial and error method will be used. Let us calculate the
NPV at the discount rate of 10%.
Year Cash Inflows Present Value Factors at 10% (PVF10%, t) Present Value at 10%
1 12,000 0.909 10,908
2 12,000 0.826 9,912
3 14,000 0.751 10,514
4 15,000 0.683 10,245
5 16,000 0.621 9,936
Total present value 51,515
Less: Initial cash outlay -42,000
Net Present Value (NPV) 9,515
Since NPV is positive, so, we shall increase the discount rate. Now let us calculate the NPV at the
discount rate of 15%.
Year Cash Inflows Present Value Factors at 15% (PVF15%, t) Present Value at 15%
1 12,000 0.870 10,440
2 12,000 0.756 9,072
3 14,000 0.658 9,212
4 15,000 0.572 8,580
5 16,000 0.497 7,952
Total present value 45,256
Less: Initial cash outlay -42,000
Net Present Value (NPV) 3,256
Since NPV is positive, so, we shall increase the discount rate. Now let us calculate the NPV at the
discount rate of 20%.
Chapter 5 and 6, Capital Budgeting: 28
Year Cash Inflows Present Value Factors at 20% (PVF20%, t) Present Value at 20%
1 12,000 0.833 9,996
2 12,000 0.694 8,328
3 14,000 0.579 8,106
4 15,000 0.482 7,230
5 16,000 0.402 6,432
Total present value 40,092
Less: Initial cash outlay -42,000
Net Present Value (NPV) (1,908)
Now the NPV is negative at 20% and positive at 15%. It means that the zero NPV lies between
(₹1,908) and ₹3,256 and IRR lies between 20% and 15%. So, there is need to apply the formula for
the interpolation.
𝑁𝑃𝑉𝐿𝐷𝑅
𝐼𝑅𝑅 = 𝐿𝐷𝑅 + × |𝐷𝑖𝑓𝑓𝑒𝑟𝑒𝑛𝑐𝑒 𝑜𝑓 𝑅𝑎𝑡𝑒𝑠|
𝑁𝑃𝑉𝐿𝐷𝑅 − 𝑁𝑃𝑉𝐻𝐷𝑅
𝑁𝑃𝑉15%
𝐼𝑅𝑅 = 15 + ×5
𝑁𝑃𝑉15% − 𝑁𝑃𝑉20%
₹3,256
𝐼𝑅𝑅 = 15 + ×5
₹3,256 − (−₹1,908)
𝐼𝑅𝑅 = 15 + 3.153
𝐼𝑅𝑅 = 18.153
or
𝑃𝑉𝐿𝐷𝑅 − 𝑃𝑉 𝑜𝑓 𝑐𝑎𝑠ℎ 𝑜𝑢𝑡𝑓𝑙𝑜𝑤𝑠 𝑜𝑟 𝐼𝑛𝑖𝑡𝑖𝑎𝑙 𝑜𝑢𝑡𝑙𝑎𝑦
𝐼𝑅𝑅 = 𝐿𝐷𝑅 + × |𝐷𝑖𝑓𝑓𝑒𝑟𝑒𝑛𝑐𝑒 𝑜𝑓 𝑅𝑎𝑡𝑒𝑠|
𝑃𝑉𝐿𝐷𝑅 − 𝑃𝑉𝐻𝐷𝑅
𝑃𝑉15% − 𝑃𝑉 𝑜𝑓 𝑐𝑎𝑠ℎ 𝑜𝑢𝑡𝑓𝑙𝑜𝑤𝑠 𝑜𝑟 𝐼𝑛𝑖𝑡𝑖𝑎𝑙 𝑜𝑢𝑡𝑙𝑎𝑦
𝐼𝑅𝑅 = 15 + ×5
𝑃𝑉15% − 𝑃𝑉20%
45,256 − 42,000
𝐼𝑅𝑅 = 15 + ×5
45,256 − 40,092
𝐼𝑅𝑅 = 15 + 3.153
𝐼𝑅𝑅 = 18.153
Notes:
1. In Excel® the IRR comes out 18.039%.
2. At 18.153% the NPV is not exactly zero but -114.578. It’s due to the fact that we have taken
15% and 20% to interpolate the answer. If we take 18% and 19% then answer would be
close to 18% (through interpolation it would be 18.038 and NPV would be 0.861) and in
such a case the NPV would be close to zero.
3. At 18% the PV is ₹42,038 (NPV is ₹38) and at 19% the PV is ₹41,043 (NPV is −₹957).
Interpolate the IRR and it will be 18.038.

6. Terminal Value (TV) Method


The assumption behind the Terminal Value (TV) method is that each cash inflow is re-invested in
another asset at a certain rate of return from the moment it is received until the termination of the
project.

Decision Rule

Accept/reject criteria
𝐼𝑓 𝑃𝑟𝑜𝑗𝑒𝑐𝑡’𝑠 𝑁𝑃𝑉 < 0 = 𝑅𝑒𝑗𝑒𝑐𝑡 𝑡ℎ𝑒 𝑃𝑟𝑜𝑗𝑒𝑐𝑡
𝐼𝑓 𝑃𝑟𝑜𝑗𝑒𝑐𝑡’𝑠 𝑁𝑃𝑉 > 0 = 𝐴𝑐𝑐𝑒𝑝𝑡 𝑡ℎ𝑒 𝑃𝑟𝑜𝑗𝑒𝑐𝑡

Mutually exclusive projects


Select the project with the highest NPV

Capital rationing
Rank the projects from the highest NPV to the lowest and then invest accordingly.
Maximum NPV Rank I 
Chapter 5 and 6, Capital Budgeting: 29
Moderate NPV Rank II 
Minimum NPV Rank III 

Example 14
DISCOUNT RATE
Initial outlay: ₹10,000; Life of the project: 5 years; 𝑘𝑂 = 10%; and Cash inflows: ₹4,000 each year
for 5 years. Expected interest rates at which cash inflows will be reinvested—
Year Percent rate of interest
1 6
2 6
3 8
4 8
5 8

Solution ANNEXURE 3 TABLE A1, FUTURE VALUE INTEREST FACTORS


Year Cash Interest Time/years for Compounding Compound sum
inflows rate investment factor (FVIFr,n)
1 4,000 6 4 𝐹𝑉𝐼𝐹6%,4 = 1.262 5,048
2 4,000 6 3 𝐹𝑉𝐼𝐹6%,3 = 1.191 4,764
3 4,000 8 2 𝐹𝑉𝐼𝐹8%,2 = 1.166 4,664
4 4,000 8 1 𝐹𝑉𝐼𝐹8%,1 = 1.080 4,320
5 4,000 8 0 𝐹𝑉𝐼𝐹8%,0 = 1.000 4,000
Total 22,796
𝑁𝑃𝑉 = 𝑃𝑟𝑒𝑠𝑒𝑛𝑡 𝑣𝑎𝑙𝑢𝑒 𝑜𝑓 𝑐𝑎𝑠ℎ 𝑖𝑛𝑓𝑙𝑜𝑤𝑠
− 𝑃𝑟𝑒𝑠𝑒𝑛𝑡 𝑣𝑎𝑙𝑢𝑒 𝑜𝑓 𝑐𝑎𝑠ℎ 𝑜𝑢𝑡𝑓𝑙𝑜𝑤𝑠 𝑜𝑟 𝑐𝑎𝑠ℎ 𝑜𝑢𝑡𝑓𝑙𝑜𝑤𝑠 𝑖𝑛 𝑧𝑒𝑟𝑜 𝑦𝑒𝑎𝑟
𝑁𝑃𝑉 = (𝐶𝑜𝑚𝑝𝑜𝑢𝑛𝑑𝑒𝑑 𝑆𝑢𝑚 × 𝑃𝑉𝐹10%,5 ) − 𝐶𝑎𝑠ℎ 𝑂𝑢𝑡𝑓𝑙𝑜𝑤𝑠 ⇒ (22,796 × 0.621) − 10,000
= ₹4,156

7. Modified Internal Rate of Return Method (MIRR)


The implied assumption in the IRR is⎯that the intermediate cash flows are reinvested at the IRR.
But this is not possible because the cash inflows can be invested at the market rate of the return of
the proposal whereas the IRR is not the rate of return of the project. In order to overcome the
problem of reinvestment rate of return, MIRR is there, and the assumption is that the cash flows can
be reinvested at the market rate of return.

Decision Rule

Accept/reject criteria
𝐼𝑓 𝑃𝑟𝑜𝑗𝑒𝑐𝑡’𝑠 𝑀𝐼𝑅𝑅 < 𝑆𝑡𝑎𝑛𝑑𝑎𝑟𝑑 𝑅𝑎𝑡𝑒 𝑜𝑓 𝑅𝑒𝑡𝑢𝑟𝑛 = 𝑅𝑒𝑗𝑒𝑐𝑡 𝑡ℎ𝑒 𝑃𝑟𝑜𝑗𝑒𝑐𝑡
𝐼𝑓 𝑃𝑟𝑜𝑗𝑒𝑐𝑡’𝑠 𝑀𝐼𝑅𝑅 > 𝑆𝑡𝑎𝑛𝑑𝑎𝑟𝑑 𝑅𝑎𝑡𝑒 𝑜𝑓 𝑅𝑒𝑡𝑢𝑟𝑛 = 𝐴𝑐𝑐𝑒𝑝𝑡 𝑡ℎ𝑒 𝑃𝑟𝑜𝑗𝑒𝑐𝑡

Mutually exclusive projects


Select the project with the highest MIRR (But we do not suggest relying solely on the results given
by the IRR rather one should consider and prefer the NPV)

Capital rationing
Rank the projects from the highest IRR to the lowest and then invest accordingly. (But we do not
suggest relying solely on the ranks given by the IRR rather one should consider and prefer the
ranking given by the NPV)
Maximum MIRR Rank I 
Moderate MIRR Rank II 
Minimum MIRR Rank III 

Chapter 5 and 6, Capital Budgeting: 30


Merits and demerits

Merits
1. MIRR overcomes limitations of IRR like IRR’s multiple rates of return in some cases,
difference in ranking according to NPV and IRR in case of mutually exclusive projects.
2. MIRR like NPV & IRR is based on cash flows rather than accounting profit.
3. MIRR also incorporates the riskiness of the project in capital budgeting analysis through use
of an appropriate discount rate.
4. MIRR tells whether a project would increase firm’s value or shareholder’s wealth.
5. MIRR method differentiates between cash flows occurring at different points of time and
thus, considers time value of money.

Demerits
1. It is difficult to compute as compared to other techniques of capital budgeting.
2. It requires computations of required rate of return to be used to discount future cash flows.
3. Needs to compute present values and forecasting of future cash flows.
Procedure to compute MIRR is as follows—
1. Find out the single terminal value (cost of capital will be the rate used to calculate the NPV).
2. Find out the IRR of the proposal using two cash flows i.e., initial investment and the terminal
value.
3. The IRR so arrived at is the MIRR.

Example 15
Mukta Limited is evaluating a proposal having initial investment of ₹20,000. Cash flows
are⎯₹4,000, ₹6,000, ₹7,000, ₹8,000 and ₹8,000 over a period of 5 years. Cost of capital is 10%. Find
out the MIRR?

Solution
Year Cash Compounding factor @ 10% Compounded Modified cash
flows 𝐹𝑉𝐼𝐹𝑟,𝑛 = (1 + 𝑟)𝑛 value @ 10% flows
0 (20,000) -- -- (20,000)
1 4,000 𝐹𝑉𝐼𝐹10%,4 = 1.464 5,856
2 6,000 𝐹𝑉𝐼𝐹10%,3 = 1.331 7,986
3 7,000 𝐹𝑉𝐼𝐹10%,2 = 1.210 8,470
4 8,000 𝐹𝑉𝐼𝐹10%,1 = 1.100 8,800
5 8,000 𝐹𝑉𝐼𝐹10%,0 =1.000 8,000 39,112
Terminal Cash Flows or Modified Cash Flows 39,112
Now there are two cash flows: Initial cash flows⎯₹20,000 and Terminal cash flows⎯₹39,112. Now
calculate IRR using these two cash flows.
𝑃𝑉 𝑎𝑡 14% = (39,112 × 0.519) ⇒ 20,299.13 𝑎𝑛𝑑 𝑃𝑉 𝑎𝑡 15% = (39,112 × 0.497) ⇒ 19,438.66.
𝑁𝑃𝑉 𝑎𝑡 14% = (39,112 × 0.519) − 20,000 ⇒ 299.13 𝑎𝑛𝑑 𝑁𝑃𝑉 𝑎𝑡 15% = (39,112 × 0.497) −
20,000 ⇒ −561.336.
𝑃𝑉𝐿𝐷𝑅 − 𝐶𝑎𝑠ℎ 𝑂𝑢𝑡𝑓𝑙𝑜𝑤𝑠
𝑀𝐼𝑅𝑅(𝑟) = 𝐿𝐷𝑅 + × |𝐷𝑖𝑓𝑓𝑒𝑟𝑒𝑛𝑐𝑒 𝑜𝑓 𝑟𝑎𝑡𝑒𝑠|
𝑃𝑉𝐿𝐷𝑅 − 𝑃𝑉𝐻𝐷𝑅
20,299.13 − 20,000
⇒ 14 + × 1 = 14.35%
20,299.13 − 19,438.66
or
𝑁𝑃𝑉𝐿𝐷𝑅
𝑀𝐼𝑅𝑅(𝑟) = 𝐿𝐷𝑅 + × |𝐷𝑖𝑓𝑓𝑒𝑟𝑒𝑛𝑐𝑒 𝑜𝑓 𝑟𝑎𝑡𝑒𝑠|
𝑁𝑃𝑉𝐿𝐷𝑅 − 𝑁𝑃𝑉𝐻𝐷𝑅
299.13
⇒ 14 + × 1 = 14.35%
299.13 − (−561.336)
(IRR is 17.15%, NPV @ 10% is ₹4,281 and MIRR is 14.36%)
Note: In Excel® the MIRR comes out 14.356%

Chapter 5 and 6, Capital Budgeting: 31


7. Discounted Pay Back Period Method
In this method the cash flows which are used to compute the Pay Back Period are discounted or we
can say that payback period is calculated using present values of the cash flows. The remaining
procedure is same as that of the Simple Pay Back Period Method.

Merits and demerits

Merits
1. It provides a good measure of liquidity of project.
2. Unlike payback period, it considers time value of money.
3. Incorporates riskiness of project through appropriate discount rate.

Demerits
1. It is more difficult to calculate than simple payback period.
2. Requires computation of discount rate, which is a tedious task.
3. Like payback period, it also ignores cash flows occurring after the payback period.

8. Economic Rate of Return (ERR) Method


𝑃𝑟𝑒𝑠𝑒𝑛𝑡 𝑣𝑎𝑙𝑢𝑒 𝑜𝑓 𝑒𝑥𝑝𝑜𝑟𝑡𝑠
𝐸𝑅𝑅 =
𝑃𝑟𝑒𝑠𝑒𝑛𝑡 𝑣𝑎𝑙𝑢𝑒 𝑜𝑓 𝑖𝑚𝑝𝑜𝑟𝑡𝑠

Decision Rule
𝐼𝑓 𝑃𝑟𝑜𝑗𝑒𝑐𝑡’𝑠 𝐸𝑅𝑅 > 1 = 𝐴𝑐𝑐𝑒𝑝𝑡 𝑡ℎ𝑒 𝑃𝑟𝑜𝑗𝑒𝑐𝑡
𝐼𝑓 𝑃𝑟𝑜𝑗𝑒𝑐𝑡’𝑠 𝐸𝑅𝑅 < 1 = 𝑅𝑒𝑗𝑒𝑐𝑡 𝑡ℎ𝑒 𝑃𝑟𝑜𝑗𝑒𝑐𝑡

9. Social Rate of Return (SRR) Method


𝑃𝑟𝑒𝑠𝑒𝑛𝑡 𝑣𝑎𝑙𝑢𝑒 𝑜𝑓 𝑏𝑒𝑛𝑒𝑓𝑖𝑡𝑠 𝑡𝑜 𝑡ℎ𝑒 𝑠𝑜𝑐𝑖𝑒𝑡𝑦
𝑆𝑅𝑅 =
𝑃𝑟𝑒𝑠𝑒𝑛𝑡 𝑣𝑎𝑙𝑢𝑒 𝑜𝑓 𝑡ℎ𝑒 𝑐𝑜𝑠𝑡 𝑡𝑜 𝑡ℎ𝑒 𝑠𝑜𝑐𝑜𝑖𝑒𝑡𝑦

Decision Rule
𝐼𝑓 𝑃𝑟𝑜𝑗𝑒𝑐𝑡’𝑠 𝑆𝑅𝑅 > 1 = 𝐴𝑐𝑐𝑒𝑝𝑡 𝑡ℎ𝑒 𝑃𝑟𝑜𝑗𝑒𝑐𝑡
𝐼𝑓 𝑃𝑟𝑜𝑗𝑒𝑐𝑡’𝑠 𝑆𝑅𝑅 < 1 = 𝑅𝑒𝑗𝑒𝑐𝑡 𝑡ℎ𝑒 𝑃𝑟𝑜𝑗𝑒𝑐𝑡

10. Economic Value Added™ (EVA™)*


Because of its introduction in the last few years, it is still not used in many firms but is fast
becoming a very popular measure of performance for many US companies.
Economic Value Added is a specific type of residual income calculated as after-tax profits minus the
(after-tax) weighted-average cost of capital multiplied by total assets less current liabilities:
𝐸𝑉𝐴 = 𝐴𝑓𝑡𝑒𝑟 𝑡𝑎𝑥 𝑜𝑝𝑒𝑟𝑎𝑡𝑖𝑛𝑔 𝑖𝑛𝑐𝑜𝑚𝑒 − [𝑘𝑂 × (𝑇𝑜𝑡𝑎𝑙 𝐴𝑠𝑠𝑒𝑡𝑠 − 𝑇𝑜𝑡𝑎𝑙 𝐿𝑖𝑎𝑏𝑖𝑙𝑖𝑡𝑖𝑒𝑠 )]
𝑜𝑟
𝐸𝑉𝐴 = 𝐸𝐵𝐼𝑇 − [𝑘𝑂 × (𝑇𝑜𝑡𝑎𝑙 𝐴𝑠𝑠𝑒𝑡𝑠 − 𝑇𝑜𝑡𝑎𝑙 𝐿𝑖𝑎𝑏𝑖𝑙𝑖𝑡𝑖𝑒𝑠)]

Examples
• Coca Cola: In the late 80s Coca-Cola applied EVA to its product lines. The analysis
suggested that several of its product lines were unprofitable. Consequently, Coca-Cola
discontinued its investments in businesses such as pasta and wines.
• United States Postal Service (UPS): In 1995, the United States Postal Service (USPS)
adopted the EVA to improve control and productivity. The USPS employs almost $15 billion
in capital, implying at a 12% rate it must earn $180 million just to cover its capital cost.
Different firms adopt EVA for different reasons. (See SPX 1995 Annual Report reprinted in Anthony,
Banker et. al. pp 587)

* Economic Value Added and EVA are trademarks of Stern Stewart & Company.
Chapter 5 and 6, Capital Budgeting: 32
Chapter 5 and 6, Capital Budgeting: 33
Illustration
Example 16 (Illustration 5)
Calculate ARR and payback period for project A and B using the data given below—
Item Project A Project B
Cost ₹56,125 ₹56,125
Net income after depreciation and taxes—
Year ₹ ₹
1 3,375 11,375
2 5,375 9,375
3 7,375 7,375
4 9,375 5,375
5 11,375 3,375
Depreciation has been charged on straight line method. Life of the both the projects is 5 years.
(B. Com. Honors, Delhi University, 2003)

Solution

(i) Average Rate of Return


3,375 + 5,375 + 7,375 + 9,375 + 11,375
𝐴𝑣𝑒𝑟𝑎𝑔𝑒 𝑖𝑛𝑐𝑜𝑚𝑒(𝐴) = ⇒ ₹7,375
5
11,375 + 9,375 + 7,375 + 5,375 + 3,375
𝐴𝑣𝑒𝑟𝑎𝑔𝑒 𝑖𝑛𝑐𝑜𝑚𝑒(𝐵) = ⇒ ₹7,375
5

56,125
𝐴𝑣𝑒𝑟𝑎𝑔𝑒 𝑖𝑛𝑣𝑒𝑠𝑡𝑚𝑒𝑛𝑡(𝐴) = ⇒ ₹28,062.50
2
56,125
𝐴𝑣𝑒𝑟𝑎𝑔𝑒 𝑖𝑛𝑣𝑒𝑠𝑡𝑚𝑒𝑛𝑡(𝐵) = ⇒ ₹28,062.50
2
7,375
𝐴𝑣𝑒𝑎𝑟𝑎𝑔𝑒 𝑟𝑎𝑡𝑒 𝑜𝑓 𝑟𝑒𝑡𝑢𝑟𝑛(𝐴) = × 100 ⇒ 26.28%
28,062.50
7,375
𝐴𝑣𝑒𝑎𝑟𝑎𝑔𝑒 𝑟𝑎𝑡𝑒 𝑜𝑓 𝑟𝑒𝑡𝑢𝑟𝑛(𝐵) = × 100 ⇒ 26.28%
28,062.50

(ii) Payback period


Depreciation (A and 𝐶𝐹𝐴𝑇
B) i.e. = 𝐸𝐴𝑇 + 𝐷𝑒𝑝. Cumulative CFAT
Project A Project B 𝟓𝟔, 𝟏𝟐𝟓 Project Project Project Project
Year EAT EAT = ₹𝟏𝟏, 𝟐𝟐𝟓 (A) (B) (A) (B)
𝟓
1 3,375 11,375 11,225 14,600 22,600 14,600 22,600
2 5,375 9,375 11,225 16,600 20,600 31,200 43,200
3 7,375 7,375 11,225 18,600 18,600 49,800 61,800
4 9,375 5,375 11,225 20,600 16,600 70,400 78,400
5 11,375 3,375 11,225 22,600 14,600 93,000 93,000
(56,125 − 49,800)
𝑃𝑎𝑦𝑏𝑎𝑐𝑘 𝑝𝑒𝑟𝑖𝑜𝑑(𝐴) = 3 𝑦𝑒𝑎𝑟𝑠 + ⇒ 3.31 𝑦𝑒𝑎𝑟𝑠
20,600
(56,125 − 43,200)
𝑃𝑎𝑦𝑏𝑎𝑐𝑘 𝑝𝑒𝑟𝑖𝑜𝑑(𝐵) = 2 𝑦𝑒𝑎𝑟𝑠 + ⇒ 2.695 𝑦𝑒𝑎𝑟𝑠
18,600

Example 17 (Illustration 7)
A firm with the cost of capital of 10% is considering two mutually exclusive projects X and Y, details
of which are as follows—
Year Project X Project Y
0 -1,00,000 -1,00,000

Chapter 5 and 6, Capital Budgeting: 34


1 10,000 50,000
2 20,000 40,000
3 30,000 20,000
4 45,000 10,000
5 60,000 10,000
Compute the NPV, PI and IRR for X and Y. Which project should be accepted and why?
(B. Com. Honors, Delhi University, 2012)

Solution

(i) Net present value


CFAT Present value at 10%
Year X Y 𝑷𝑽𝑭 𝟏𝟎%,𝒏 X Y
1 10,000 50,000 0.909 9,090 45,450
2 20,000 40,000 0.826 16,520 33,040
3 30,000 20,000 0.751 22,530 15,020
4 45,000 10,000 0683 30,735 6,830
5 60,000 10,000 0.621 37,260 6,210
Total present value 1,16,135 1,06,550
Less: Initial cash outlay -1,00,000 -1,00,000
Net present value (NPV) 16,135 6,550
Decision: Since the NPV of project X is higher, so, it should be accepted.

(ii) Profitability index


𝑃𝑟𝑒𝑠𝑒𝑛𝑡 𝑣𝑎𝑙𝑢𝑒 𝑜𝑓 𝑡ℎ𝑒 𝑐𝑎𝑠ℎ 𝑖𝑛𝑓𝑙𝑜𝑤𝑠
𝑃𝐼/𝐵𝐶 𝑅𝑎𝑡𝑖𝑜 =
𝑃𝑟𝑒𝑠𝑒𝑛𝑡 𝑣𝑎𝑙𝑢𝑒 𝑜𝑓 𝑡ℎ𝑒 𝑐𝑎𝑠ℎ 𝑜𝑢𝑡𝑓𝑙𝑜𝑤𝑠
1,16,135
𝑃𝐼/𝐵𝐶 𝑅𝑎𝑡𝑖𝑜(𝐴) = = 1.16135
1,00,000
1,06,550
𝑃𝐼/𝐵𝐶 𝑅𝑎𝑡𝑖𝑜(𝐵) = = 1.06550
1,00,000
Decision: Since the PI of project X is higher, so, it should be accepted.

(iii) Internal rate of return


Project X
Present value at
Year CFAT 𝑷𝑽𝑭𝟏𝟒%,𝒏 𝑷𝑽𝑭𝟏𝟓%,𝒏 14% 15%
1 10,000 0.877 0.870 8,770 8,700
2 20,000 0.769 0.756 15,380 15,120
3 30,000 0.675 0.658 20,250 19,740
4 45,000 0.592 0.572 26,640 25,740
5 60,000 0.519 0.497 31,140 29,820
Total present value 1,02,180 99,120
Less: Initial cash outlay -1,00,000 -1,00,000
Net present value (NPV) 2,180 -880
𝑁𝑃𝑉𝐿𝐷𝑅
𝐼𝑅𝑅 = 𝐿𝐷𝑅 + × |𝐷𝑖𝑓𝑓𝑒𝑟𝑒𝑛𝑐𝑒 𝑜𝑓 𝑅𝑎𝑡𝑒𝑠| ⇒ 𝐼𝑅𝑅
𝑁𝑃𝑉𝐿𝐷𝑅 − 𝑁𝑃𝑉𝐻𝐷𝑅
2,180 2,180
= 14 + × |1| ⇒ 14 + × 1 ⇒ 14 + 0.7124
2,180 − (−880) 3,060
⇒ 14.7124% 𝑜𝑟 0.147124
Project Y
Present value at
Year CFAT 𝑷𝑽𝑭𝟏𝟑%,𝒏 𝑷𝑽𝑭𝟏𝟒%,𝒏 13% 14%
1 50,000 0.885 0.877 44,250 43,850
2 40,000 0.783 0.769 31,320 30,760
3 20,000 0.693 0.675 13,860 13,500
Chapter 5 and 6, Capital Budgeting: 35
4 10,000 0.613 0.592 6,130 5,920
5 10,000 0.543 0.519 5,430 5,190
Total present value 1,00,990 99220
Less: Initial cash outlay -1,00,000 -1,00,000
Net present value (NPV) 990 -780
𝑁𝑃𝑉𝐿𝐷𝑅
𝐼𝑅𝑅 = 𝐿𝐷𝑅 + × |𝐷𝑖𝑓𝑓𝑒𝑟𝑒𝑛𝑐𝑒 𝑜𝑓 𝑅𝑎𝑡𝑒𝑠| ⇒ 𝐼𝑅𝑅
𝑁𝑃𝑉𝐿𝐷𝑅 − 𝑁𝑃𝑉𝐻𝐷𝑅
990 990
= 13 + × |1| ⇒ 13 + × 1 ⇒ 13 + 0.559
990 − (−780) 1,770
⇒ 13.559% 𝑜𝑟 0.13559
Decision: Since the IRR of project X is higher, so, it should be accepted.

Example 18 (Illustration 8)
East Delhi Company Private Limited is considering the purchase of new manufacturing plants. Two
alternative plants A and B are available, each having the initial outlay of ₹10,00,000 and require
₹50,000 as additional working capital at the end of 1st year. Expected net cash flows are as follows—
Year Plant A Plant B
1 1,00,000 3,00,000
2 3,00,000 4,00,000
3 4,00,000 5,00,000
4 6,00,000 3,00,000
5 4,00,000 2,00,000
Minimum rate of return is 10%. Calculate the profitability of the plants and which plant should be
accepted?
(B. Com. Honors, Delhi University, 2013)

Solution
Present value of cash Present value of cash
𝑪𝑶𝒕 𝑪𝑭𝑨𝑻𝒕 outflows inflows
Year A B A B 𝑷𝑽𝑭𝟏𝟎%,𝒏 A B A B
0 10,00,000 10,00,000 -- -- 1.000 10,00,000 10,00,000 -- --
1 50,000 50,000 1,00,000 3,00,000 0.909 45,450 45,450 90,900 2,72,700
2 -- -- 3,00,000 4,00,000 0.826 -- -- 2,47,800 3,30,400
3 -- -- 4,00,000 5,00,000 0.751 -- -- 3,00,400 3,75,500
4 -- -- 6,00,000 3,00,000 0.683 -- -- 4,09,800 2,04,900
5 -- -- 4,00,000 2,00,000 0.621 -- -- 2,48,400 1,24,200
5 -- -- 50,000 50,000 0.621 -- -- 31,050 31,050
Present value of cash outflows and inflows 10,45,450 10,45,450 13,28,350 13,38,750
𝑁𝑃𝑉 = 𝑃𝑟𝑒𝑠𝑒𝑛𝑡 𝑣𝑎𝑙𝑢𝑒 𝑜𝑓 𝑐𝑎𝑠ℎ 𝑖𝑛𝑓𝑙𝑜𝑤𝑠 − 𝑃𝑟𝑒𝑠𝑒𝑛𝑡 𝑣𝑎𝑙𝑢𝑒 𝑜𝑓 𝑐𝑎𝑠ℎ 𝑜𝑢𝑡𝑓𝑙𝑜𝑤𝑠
𝑁𝑃𝑉(𝐴) = 13,28,350 − 10,45,450 = ₹2,82,900
𝑁𝑃𝑉(𝐵) = 13,38,750 − 10,45,450 = ₹2,93,300
Decision: Since the NPV of project B is higher, so, it should be accepted.

Example 19 (Illustration 9)
A company is considering as to which of the two mutually exclusive projects it should undertake.
Project ‘A’ costs ₹2,00,000 and project ‘B’ costs ₹2,20,000. Both the projects have the same life of 5
years. The company anticipates a cost of capital of 10% and the after-tax cash flows are ₹80,000;
₹80,000; ₹60,000; ₹60,000 and ₹20,000 for the next 5 years for project ‘A’ and ₹50,000; ₹50,000;
₹60,000; ₹90,000 and ₹90,000 for the next 5 years for project ‘B’. Determine the Payback period
and PI for project ‘A’ and ‘B’. Which project would you recommend?
(B. Com. Honors, Delhi University, 2015)

Solution

(i) Payback period


𝑪𝑭𝑨𝑻𝒕 Cumulative CFAT
Year A B A B
1 80,000 50,000 80,000 50,000
Chapter 5 and 6, Capital Budgeting: 36
2 80,000 50,000 1,60,000 1,00,000
3 60,000 60,000 2,20,000 1,60,000
4 60,000 90,000 2,80,000 2,50,000
5 20,000 90,000 3,00,000 3,40,000
(2,00,000 − 1,60,000)
𝑃𝑎𝑦𝑏𝑎𝑐𝑘 𝑝𝑒𝑟𝑖𝑜𝑑(𝐴) = 2 𝑦𝑒𝑎𝑟𝑠 + ⇒ 2.67 𝑦𝑒𝑎𝑟𝑠
60,000
(2,20,000 − 1,60,000)
𝑃𝑎𝑦𝑏𝑎𝑐𝑘 𝑝𝑒𝑟𝑖𝑜𝑑(𝐵) = 3 𝑦𝑒𝑎𝑟𝑠 + ⇒ 3.67 𝑦𝑒𝑎𝑟𝑠
90,000
Decision: Since the payback period of project ‘A’ is lowers, so, it should be accepted.

(ii) Profitability index


𝑪𝑭𝑨𝑻𝒕 Present value
Year A B 𝑷𝑽𝑭𝟏𝟎%,𝒏 A B
1 80,000 50,000 0.909 72,720 45,450
2 80,000 50,000 0.826 66,080 41,300
3 60,000 60,000 0.751 45,060 45,060
4 60,000 90,000 0.683 40,980 61,470
5 20,000 90,000 0.621 12,420 55,890
Total present value 2,37,260 2,49,170
𝑃𝑟𝑒𝑠𝑒𝑛𝑡 𝑣𝑎𝑙𝑢𝑒 𝑜𝑓 𝑡ℎ𝑒 𝑐𝑎𝑠ℎ 𝑖𝑛𝑓𝑙𝑜𝑤𝑠
𝑃𝐼/𝐵𝐶 𝑅𝑎𝑡𝑖𝑜 =
𝑃𝑟𝑒𝑠𝑒𝑛𝑡 𝑣𝑎𝑙𝑢𝑒 𝑜𝑓 𝑡ℎ𝑒 𝑐𝑎𝑠ℎ 𝑜𝑢𝑡𝑓𝑙𝑜𝑤𝑠
2,37,260
𝑃𝐼/𝐵𝐶 𝑅𝑎𝑡𝑖𝑜(𝐴) = = 1.186
2,00,000
2,49,170
𝑃𝐼/𝐵𝐶 𝑅𝑎𝑡𝑖𝑜(𝐵) = = 1.133
2,20,000
Decision: Since the PI of project A is higher, so, it should be accepted.
BASE OF
UPCOMING
QUESTIONS, Example 20 (Illustration 27)
DO THIS
PROPERLY A company is planning to replace a machine which is required for 4 more years. Its book value is
₹1,60,000 and it will be fully depreciated at the end of 2 years. It will generate revenue of ₹5,20,000
per year for 4 years and incur expenses of ₹3,80,000 per year. Its salvage value now is ₹1,20,000
which will become zero after 4 years. It can be replaced by a machine costing ₹4,80,000. The new
machine is expected to generate revenue of ₹9,20,000 per year and incur annual expenses of
₹5,80,000. Additional working capital of ₹2,00,000 will be required if new machine is bought.
Depreciation on new machine will be charged at the rate of ₹80,000 per year to make its book value
equal to its expected salvage value of ₹1,60,000 at the end of fourth year. Tax rate for the company
is 30% and cost of capital is 15%. Advise the company about the replacement.
(B. Com. Honors, Delhi University, 2017)

Solution
In this question replacement is there, so, the incremental cash flows shall be calculated.
1. Initial cash outflows/outlay
Cost of the machine 4,80,000
Less: Scrap value of old machine -1,20,000
Less: Tax savings on loss on sales of old machine (See Note) -12,000
Add: Additional working capital investment 2,00,000
Cash outflows 5,48,000
Note:
Salvage value ₹1,20,000
Book value ₹1,60,000
Loss on sales of machinery ₹40,000
Tax savings on capital loss is ₹12,000 i.e. (𝐶𝑎𝑝𝑖𝑡𝑎𝑙 𝑙𝑜𝑠𝑠 × 𝑇𝑎𝑥 𝑟𝑎𝑡𝑒) ⇒ (₹40,000 × 0.30)

2. Incremental cash inflows


Particulars Year

Chapter 5 and 6, Capital Budgeting: 37


1 2 3 4
Incremental revenue (9,20,000-5,20,000) 4,00,000 4,00,000 4,00,000 4,00,000
Less: Incremental expenses (5,80,000-3,80,000) -2,00,000 -2,00,000 -2,00,000 -2,00,000
Incremental profits before depreciation and tax 2,00,000 2,00,000 2,00,000 2,00,000
Less: Incremental depreciation (See below table 3) 0 0 -80,000 -80,000
Incremental profits before tax 2,00,000 2,00,000 1,20,000 1,20,000
Less: Tax @ 30% -60,000 -60,000 -36,000 -36,000
Incremental profits after tax 1,40,000 1,40,000 84,000 84,000
Add: Incremental depreciation 0 0 80,000 80,000
Add: Scrap value of new machine at the end of 4th year -- -- -- 1,60,000
Add: Recovery of working capital -- -- -- 2,00,000
Incremental cash inflows 1,40,000 1,40,000 1,64,000 5,24,000

3. Incremental depreciation
Year
Particulars 1 2 3 4
Depreciation on new machine 80,000 80,000 80,000 80,000
Depreciation on old machine 80,000 80,000 0 0
Incremental depreciation 0 0 80,000 80,000

4. Calculation of net present value


Year 𝑪𝑭𝑨𝑻𝒕 𝑷𝑽𝑭𝟏𝟓%,𝒏 (A-3) Present value at 15%
1 1,40,000 0.870 1,21,800
2 1,40,000 0.756 1,05,840
3 1,64,000 0.658 1,07,912
4 5,24,000 0.572 2,99,728
Total present value 6,35,280
Less: Initial cash outlay -5,48,000
Net Present Value (NPV) 87,280
Decision: Since the NPV is positive, the machine may be replaced with the new machine.

Example 21 (Illustration 13)


A company is considering a proposal for production of a new product. The company expects to
sell 1,00,000 units of the new product each year at a selling price of ₹5 per unit. Variable cost will be
₹2 per unit. Regardless of the level of production, the company will incur cash cost of ₹50,000 per
year if project is undertaken. The machine for making of the product will cost ₹5,00,000 and can be
sold for ₹60,000 at the end of its life of 5 years. Additional working capital required will be ₹50,000.
Overhead cost allocated to the new product will be ₹24,000 per year. Tax rate is 30% and cost of
capital for the company is 15%. The company charges depreciation at 25% of the written down
value. Should the company buy the new machine?
(B. Com. Honors, Delhi University, 2017)

Solution
1. Initial cash outflows/outlay
Cost of the machine 5,00,000
Add: Additional working capital investment 50,000
Cash outflows 5,50,000

2. Cash inflows
Year
Particulars 1 2 3 4 5
Sales (1,00,000 𝑢𝑛𝑖𝑡𝑠 × ₹5 𝑝𝑒𝑟 𝑢𝑛𝑖𝑡) 5,00,000 5,00,000 5,00,000 5,00,000 5,00,000
Less: Variable cost (1,00,000 𝑢𝑛𝑖𝑡𝑠 × ₹2 𝑝𝑒𝑟 𝑢𝑛𝑖𝑡) -2,00,000 -2,00,000 -2,00,000 -2,00,000 2,00,000
Less: Cash fixed cost -50,000 -50,000 -50,000 -50,000 -50,000
Profits before depreciation and tax 2,50,000 2,50,000 2,50,000 2,50,000 2,50,000
Less: Dep. @ 25% on WDV (See below table 3) -1,25,000 -93,750 -70,312 -52,735 -39,551
Profits before tax 1,25,000 1,56,250 1,79,688 1,97,265 2,10,449
Less: Tax @ 30% -37,500 -46,875 -53,906 -59,180 -63,135
Profits after tax 87,500 1,09,375 1,25,782 1,38,085 1,47,314
Add: Depreciation (See below table 3) 1,25,000 93,750 70,312 52,735 39,551
Chapter 5 and 6, Capital Budgeting: 38
Add: Scrap value of new machine at the end of 5th year -- -- -- -- 60,000
Add: Recovery of working capital -- -- -- -- 50,000
Add: Tax savings on capital loss (See below table 4) -- -- -- -- 17,596
Cash inflows 2,12,500 2,03,125 1,96,094 1,90,820 3,14,461

3. Depreciation
Year
Particulars 1 2 3 4 5
Book value at the beginning 5,00,000 3,75,000 2,81,250 2,10,938 1,58,203
Less: Depreciation on machine @ 25% on
-1,25,000 -93,750 -70,312 -52,735 -39,551
written down value
Book value at the end 3,75,000 2,81,250 2,10,938 1,58,023 1,18,652

4. Tax saving on capital loss


Salvage value 60,000 Tax savings on capital loss is
Book value 1,18,652 ₹17,596 i.e. (𝐶𝑎𝑝𝑖𝑡𝑎𝑙 𝑙𝑜𝑠𝑠 × 𝑇𝑎𝑥 𝑟𝑎𝑡𝑒) ⇒ (₹58,652 ×
Loss on sales of machinery 58,652 0.30)

5. Calculation of net present value


Year 𝑪𝑭𝑨𝑻𝒕 𝑷𝑽𝑭𝟏𝟓%,𝒏 Present value at 15%
1 2,12,500 0.870 1,84,875
2 2,03,125 0.756 1,53,563
3 1,96,094 0.658 1,29,030
4 1,90,820 0.572 1,09,149
5 3,14,461 0.497 1,56,287
Total present value 7,32,904
Less: Initial cash outlay -5,50,000
Net Present Value (NPV) 1,82,904
Decision: Since the NPV is positive, so, the company should buy the machine.
Note: Allocated overhead costs have been ignored while estimating cash inflows as these will have
to be paid regardless of acceptance or non-acceptance of this project.

Example 22
The income statement of PQR Limited for the current year as follows—
Particulars Amount (₹) Amount (₹)
Sales 7,00,000
Less: Costs
Material 2,00,000
Labor 2,50,000
Other operating costs 80,000
Depreciation 70,000 -6,00,000
Earnings before interest and taxes 1,00,000
Less: Tax @ 40% -40,000
Earnings after taxes 60,000
The plant manager proposes to replace an existing machine by another machine costing ₹2,40,000.
The new machine will have 8 years life having no salvage value. It is estimated that the new
machine will reduce the labor cost by ₹50,000 per year. The old machine will realize ₹40,000. The
income statement does not include the depreciation on old machine (the one that is going to be
replaced) as the same had been fully depreciated for tax purposes last year though it will continue
to function; if not replaced, for a few years more. It is believed that there will be no change in other
expenses and revenues of the firm due to its replacement. The company requires an after-tax return
of 12%. The tax rate applicable to the company is 30%. Suggest whether the company should buy
the new machine, assuming that the company follows the straight-line method of depreciation and
the same is allowed for tax purposes.
(B. Com. Honors, Delhi University, 2018)

Chapter 5 and 6, Capital Budgeting: 39


Solution
Since there is difference in tax rates in the question, calculations using any tax rate will be taken as
correct. We will assume the tax rate as 30% in the solution. However, if you solve the question using
40% even then your solution will be correct.
The statement says— “replace an existing machine by another machine”, so, the incremental cash
flows shall be computed as follows—
1. Initial cash outflows/outlay
Cost of the machine 2,40,000
Less: Scrap value of old machine -40,000
Add: Tax liability on gain on sales of old machine (See Note) 12,000
Cash outflows 2,12,000
Note:
Salvage value ₹40,000
Book value ₹0
Profit on sales of machinery ₹40,000
Tax liability on capital gain is ₹12,000 i.e., (𝐶𝑎𝑝𝑖𝑡𝑎𝑙 𝑔𝑎𝑖𝑛 × 𝑇𝑎𝑥 𝑟𝑎𝑡𝑒) ⇒ (₹40,000 × 0.30)

2. Incremental cash inflows


Year
Particulars 1 to 8
Savings in labor cost 50,000
Less: Incremental depreciation (See below table 3) -30,000
Incremental profits before tax 20,000
Less: Tax @ 30% -6000
Incremental profits after tax 14,000
Add: Incremental depreciation 30,000
Incremental cash inflows 44,000

3. Incremental depreciation
Particulars Year 1 to 8
Depreciation on new machine (₹2,40,000/8 years) 30,000
Depreciation on old machine 0
Incremental depreciation 30,000

4. Calculation of net present value


Year 𝑪𝑭𝑨𝑻𝒕 𝑷𝑽𝑭𝟏𝟐%,𝒏 (A-3) Present value at 12%
1 44,000 0.893 39,292
2 44,000 0.797 35,068
3 44,000 0.712 31,328
4 44,000 0.636 27,984
5 44,000 0.567 24,948
6 44,000 0.507 22,308
7 44,000 0.452 19,888
8 44,000 0.404 17,776
Total present value 2,18,592
Less: Initial cash outlay -2,12,000
Net Present Value (NPV) 6,592
Present value of incremental cash inflows [₹44,000 × 𝑃𝑉𝐴𝐹12%,8 ] ⇒ 44,000 × 4.968 ⇒ ₹2,18,592
Less: Cash outflows ₹2,12,000
Net present value ₹ 6,592
Decision: Since the NPV is positive, the machine may be replaced with the new machine.

Alternate method (as suggested by the then examiner of the


Fundamentals of Financial Management)
1. Initial cash outflows/outlay

Chapter 5 and 6, Capital Budgeting: 40


Cost of the machine 2,40,000
Less: Scrap value of old machine -40,000
Add: Tax liability on gain on sales of old machine (See Note) 12,000
Cash outflows 2,12,000
Note:
Salvage value ₹40,000
Book value ₹0
Profit on sales of machinery ₹40,000
Tax liability on capital gain is ₹12,000 i.e. (𝐶𝑎𝑝𝑖𝑡𝑎𝑙 𝑔𝑎𝑖𝑛 × 𝑇𝑎𝑥 𝑟𝑎𝑡𝑒) ⇒ (₹40,000 × 0.30)

2. Incremental cash inflows


After
Existing replacement
Sales
Less: Operating expenses See Note-
Profits before depreciation and tax 7,00,000 7,00,000 1
Less: Depreciation -5,30,000 See Note-1 -4,80,000
Profits before tax 1,70,000 2,20,000 See Note-
Less: Tax @ 40% and 30% -70,000 See Note-2 -1,00,000 2
Profits after taxes 1,00,000 1,20,000
Add: Depreciation -40,000 -36,000
Cash inflows 60,000 84,000
70,000 1,00,000
1,30,000 1,84,000
Cash inflows after replacement 1,84,000
Less: Cash inflows before replacement/existing -1,30,000
Incremental cash inflows 54,000
Note-1: Note-2:
Operating expenses: Depreciation:
Material ₹2,00,000 Depreciation given (excluding old) ₹70,000
Labor ₹2,50,000 Depreciation on new machine ₹30,000
Other operating costs ₹80,000 Total ₹1,00,000
Total operating cost (existing) ₹5,30,000
Less: Savings in labor -₹50,000
Operating cost after savings ₹4,80,000

Note-3: Under “existing” calculation the rate of tax is taken as 40% and under “after replacement” calculation the rate
of tax is assumed to be 30%.
Note-4: Same tax rate (either 30% or 40%) can also be taken under both the situations i.e., “Existing” and “After
replacement”. In case of 30% tax rate, incremental cash inflows will be ₹44,000 per annum (NPV would be ₹6,592) and
in case of 40% tax rate, cash inflows will be ₹42,000 per annum (NPV would be -₹3,344).

3. Calculation of net present value


Year 𝑪𝑭𝑨𝑻𝒕 𝑷𝑽𝑭𝟏𝟐%,𝒏 Present value at 15%
1 54,000 0.893 48,222
2 54,000 0.797 43,038
3 54,000 0.712 38,448
4 54,000 0.636 34,344
5 54,000 0.567 30,618
6 54,000 0.507 27,378
7 54,000 0.452 24,408
8 54,000 0.404 21,816
Total present value 2,68,272
Less: Initial cash outlay -2,12,000
Net Present Value (NPV) 56,272
Present value of incremental cash inflows [₹54,000 × 𝑃𝑉𝐴𝐹12%,8 ] ⇒ 54,000 × 4.968 ⇒ ₹2,68,272
Less: Cash outflows ₹2,12,000
Net present value ₹ 56,272

Chapter 5 and 6, Capital Budgeting: 41


Example 23
EFG Technology Limited is considering a new project for manufacturing of solar energy games kit
involving a capital expenditure of ₹600 lakh and working capital of ₹150 lakh. The plant has
capacity of annual production of 12 lakh units and capacity utilization during 6 years working life of
the project is expected to be as mentioned below—
Year 1 2 3 4 5 6
Capacity utilization (%) 33.33 66.67 90 100 100 100
The average price per unit of product is expected to be ₹200 netting a contribution of 40%. The 40% of average unit
annual fixed costs, excluding depreciation, are estimated to be ₹480 lakh per annum from the third price
a unit
is the sale price of

year onwards; for the first year and second year, it would be ₹240 lakh and ₹360 lakh respectively.
Depreciation is charged @ 33.33% on the basis of written down value (WDV) method. The rate of
income tax may be taken as 35%. The cost of capital is 15%.
At the end of the third year, an additional investment of ₹100 lakh would be required for the
working capital. The terminal value of fixed assets (sold as scrap) may be taken as 10% and for the
current assets at 100%. Give suggestion to EFG Technology Limited regarding taking up the new
project.
(B. Com. Honors, Delhi University, 2018)

Solution
1. Initial cash outflows/outlay
₹(Lakhs)
Cost of the project 600
Add: Working capital investment 150
Cash outflows 750

2. Cash inflows
Year
Particulars 1 2 3 4 5 6
Production capacity (units in lakhs) 12 12 12 12 12 12
Capacity utilization 33.33% 66.67% 90% 100% 100% 100%
Annual output
(𝐴𝑛𝑛𝑢𝑎𝑙 𝑝𝑟𝑜𝑑𝑢𝑐𝑡𝑖𝑜𝑛 𝑐𝑎𝑝𝑎𝑐𝑖𝑡𝑦 × 4 8 10.8 12 12 12
𝑐𝑎𝑝𝑎𝑐𝑖𝑡𝑦 𝑢𝑡𝑖𝑙𝑖𝑧𝑎𝑡𝑖𝑜𝑛)
Annual sales @ ₹200 per unit 800 1,600 2,160 2,400 2,400 2,400
Annual contribution @ 40% of sales 320.00 640.00 864.00 960.00 960.00 960.00
Less: Annual fixed cost (excluding dep.) -240.00 -360.00 480.00 480.00 480.00 480.00
Less: Depreciation (see Table 3 below) -200.00 -133.33 -88.89 -59.26 -39.51 -26.34
Profits before taxes -120.00 146.67 295.11 420.74 440.49 453.66
Less: Tax @ 35% This must be saving because of loss -42.00 -51.33 -103.29 -147.26 -154.17 -158.78
Profits after tax -78.00 95.34 191.82 273.48 286.32 294.88
Add: Depreciation (see Table 3 below) 200.00 133.33 88.89 59.26 39.51 26.34
Add: Working capital recovered -- -- -- -- -- 250.00
Add: Scrap value (10% of cost of project) -- -- -- -- -- 60.00
Less: Tax liab. on gain (See below table 4) -- -- -- -- -- -2.56
Cash inflows 122 228.67 280.71 332.74 325.83 628.66

3. Depreciation and book value


Year
Particulars 1 2 3 4 5 6
Book value at the beginning 600 400.00 266.67 177.78 118.52 79.01
Less: Depreciation @ 33.33% on written down value -200 -133.33 -88.89 -59.26 -39.51 -26.34
Book value at the end 400 266.67 177.78 118.52 79.01 52.67

4. Tax liability on capital gain


Salvage value (10% of cost of project) 60.00 Tax liability on capital gain is
Book value 52.67 ₹2.56 lakh i.e. (𝐶𝑎𝑝𝑖𝑡𝑎𝑙 𝑔𝑎𝑖𝑛 × 𝑇𝑎𝑥 𝑟𝑎𝑡𝑒) ⇒
Gain on sales of machinery 7.33 (₹7.33 × 0.35)

5. Calculation of net present value


Year 𝑪𝑭𝑨𝑻𝒕 𝑷𝑽𝑭𝟏𝟓%,𝒏 Present value at 15%
Chapter 5 and 6, Capital Budgeting: 42
1 122.00 0.870 106.14
2 228.67 0.756 172.87
3 280.71 0.658 184.71
4 332.74 0.572 190.33
5 325.83 0.497 161.94
6 628.66 0.432 271.58
Total present value 1,087.57
Less: Present value of cash outflows (See below table 6) -815.80
Net Present Value (NPV) 271.77

6. Calculation of present value of cash outflows


Present value at
Year Cash outflows 𝑷𝑽𝑭𝟏𝟓%,𝒏 15%
0 750 1.000 750.00
3 100 0.658 65.80
Total present value of cash outflows 815.80
Decision: Since the NPV is positive, so, the project should be taken up.

Example 24
A company is considering computerization of its inventory and billing procedures at a cost of
₹2,00,000. Installation charges are ₹50,000. These outlays will be depreciated on a straight-line
basis to zero book value which will also be its salvage value at the end of its life of 5 years. The new
system will require two Computer specialists with annual salaries of ₹40,000 per person. It is also
estimated that annual maintenance expenses of ₹12,000 will have to be incurred. The new system
will lead to reduced production delays, thus, saving of ₹20,000 annually. It will also help to avoid
stock out costs of ₹25,000 per year. Timely billing will increase inflow by ₹8,000 per year. Six
clerical employees, with annual salaries of ₹20,000 each, will also become redundant. The company
tax rate is 30% and required rate of return is 12%. Evaluate the project. no longer required
What will be your decision if salvage value is taken as ₹30,000 for the purpose of calculating
depreciation, even though the machine will be worthless in terms of resale value after 5 years?
(B. Com. Honors, Delhi University, 2019)

Solution
1. Initial cash outflows/outlay
Cost of the machine 2,00,000
Add: Installation charges 50,000
Cash outflows 2,50,000

2. Cash inflows
Particulars Year (1 to 5)
Annual salaries (₹40,000 × 2 𝑝𝑒𝑟𝑠𝑜𝑛𝑠) -80,000
Annual maintenance expenses -12,000
Savings due to reduced production delays 20,000
Avoidance of stock out cost 25,000
Increase in inflow due to timely billing 8,000
Savings of salaries of six clerks (₹20,000 × 6 𝑐𝑙𝑒𝑟𝑘𝑠) 1,20,000
Net savings or profits 81,000
Less: Depreciation (₹2,50,000/5 𝑦𝑒𝑎𝑟𝑠) -50,000
Profits before tax 31,000
Less: Tax at 30% -9,300
Profits after tax 21,700
Add: Depreciation 50,000
Cash inflows 71,700

3. Calculation of net present value


Present value of incremental cash inflows [₹71,700 × 𝑃𝑉𝐴𝐹12%,5 ] ⇒ 71,700 × 3.605 ⇒ ₹2,58,749
Chapter 5 and 6, Capital Budgeting: 43
Less: Cash outflows ₹2,50,000
Net present value ₹ 8,479
Decision: Since the NPV is positive, the computerization of the inventory and billing procedures
shall be considered.

What will be your decision if salvage value is taken as ₹30,000 for the
purpose of calculating depreciation, even though the machine will be
worthless in terms of resale value after 5 years?
1. Initial cash outflows/outlay
Cost of the machine 2,00,000
Add: Installation charges 50,000
Cash outflows 2,50,000

2. Cash inflows
Particulars Year (1 to 5)
Annual salaries (₹40,000 × 2 𝑝𝑒𝑟𝑠𝑜𝑛𝑠) -80,000
Annual maintenance expenses -12,000
Savings due to reduced production delays 20,000
Avoidance of stock out cost 25,000
Increase in inflow due to timely billing 8,000
Savings of salaries of six clerks (₹20,000 × 6 𝑐𝑙𝑒𝑟𝑘𝑠) 1,20,000
Net savings or profits 81,000
Less: Depreciation (₹2,50,000 − ₹30,000)/5 𝑦𝑒𝑎𝑟𝑠 -44,000
Profits before tax 37,000
Less: tax at 30% -11,100
Profits after tax 25,900
Add: Depreciation 44,000
Cash inflows 69,900

3. Calculation of net present value


Present value of incremental cash inflows [₹69,900 × 𝑃𝑉𝐴𝐹12%,5 ] ⇒ 69,900 × 3.605 ⇒ ₹2,51,990
Less: Cash outflows ₹2,50,000
Net present value ₹ 1,990
Decision: Since the NPV is positive, the computerization of the inventory and billing procedures
shall be considered.

Alternate Solution (When capital gain is taken into account)


1. Initial cash outflows/outlay
Cost of the machine 2,00,000
Add: Installation charges 50,000
Cash outflows 2,50,000

2. Cash inflows
Particulars Year (1 to 5)
Annual salaries (₹40,000 × 2 𝑝𝑒𝑟𝑠𝑜𝑛𝑠) -80,000
Annual maintenance expenses -12,000
Savings due to reduced production delays 20,000
Avoidance of stock out cost 25,000
Increase in inflow due to timely billing 8,000
Savings of salaries of six clerks (₹20,000 × 6 𝑐𝑙𝑒𝑟𝑘𝑠) 1,20,000
Net savings or profits 81,000
Less: Depreciation (₹2,50,000 − ₹30,000)/5 𝑦𝑒𝑎𝑟𝑠 -44,000
Profits before tax 37,000
Less: tax at 30% -11,100
Profits after tax 25,900

Chapter 5 and 6, Capital Budgeting: 44


Add: Depreciation 44,000
Cash inflows 69,900

3. Calculation of net present value


Year Cash Flows PVF @ 12% PV @ 12%
0 -2,50,000 1 -2,50,000
1 69,900 0.893 62,420.7
2 69,900 0.797 55,710.3
3 69,900 0.712 49,768.8
4 69,900 0.636 44,456.4
5 78,900 (69,900+9,000 see Note 4 ) 0.567 44,736.3
Net Present Value 7092.5

4. Tax benefit on capital loss


Resale value 0 Tax benefit on capital loss is
Book value 30,000 ₹9,000 i.e. (𝐶𝑎𝑝𝑖𝑡𝑎𝑙 𝑙𝑜𝑠𝑠 × 𝑇𝑎𝑥 𝑟𝑎𝑡𝑒) ⇒
Loss on sales of machinery 30,000 (₹30,000 × 0.30)

Example 25
Able electronics is considering a proposal to replace a machine which was bought three years ago
for ₹10,00,000. It has a remaining life of 5 years after which it will have no salvage value. But if it is
sold now, it will realize ₹6,00,000. Its maintenance cost is expected to increase by ₹50,000 per
annum from the sixth year of its installation. A new, more efficient machine is available for
₹15,00,000. This machine can be sold for ₹6,00,000 after completing its life of 5 years. This new
machine will lead to increased productivity, thereby increasing the revenue by ₹1,50,000 per
annum. It will also reduce operating expenses by ₹1,00,000 per annum. The company’s tax rate is
50% and cost of capital is 10%. It uses straight line method of depreciation. Advise the company
about the replacement.
What will be your decision if the expected salvage value of the new machine is ₹2,00,000?
(B. Com. Honors, Delhi University, 2019)

Solution
1. Initial cash outflows/outlay
Cost of the machine 15,00,000
Less: Scrap value of old machine -6,00,000
Less: Tax savings on loss on sales of old machine (See Note) -12,500
Cash outflows 8,87,500
Note:
Salvage value ₹6,00,000
Book value at the end of 3rd year** ₹6,25,000
Loss on sales of machinery ₹25,000

Tax savings on capital loss is ₹12,500 i.e., (𝐶𝑎𝑝𝑖𝑡𝑎𝑙 𝑙𝑜𝑠𝑠 × 𝑇𝑎𝑥 𝑟𝑎𝑡𝑒) ⇒ (₹25,000 × 0.50)

*Annual depreciation is ₹1,25,000 i.e. ₹10,00,000/8 𝑦𝑒𝑎𝑟𝑠


**Book value at the end of 3 years is ₹6,25,000 i.e., ₹10,00,000 − (₹1,25,000 × 3 𝑦𝑒𝑎𝑟𝑠)

2. Incremental cash inflows


Particulars Year 1 to 2 Year 3 to 4 Year 5
Increase in revenues 1,50,000 1,50,000 1,50,000
Savings due to reduction in operating expenses 1,00,000 1,00,000 1,00,000
Increase in maintenance cost of old machinery* -- 50,000 50,000
Total savings/profits 2,50,000 3,00,000 3,00,000
Less: Incremental depreciation (See below table 3) -55,000 -55,000 -55,000
Profits before taxes 1,95,000 2,45,000 2,45,000

Chapter 5 and 6, Capital Budgeting: 45


Less: Tax @ 50% -97,500 -1,22,500 -1,22,500
Profits after taxes 97,500 1,22,500 1,22,500
Add: Incremental depreciation 55,000 55,000 55,000
Add: Salvage value -- -- 6,00,000
Incremental cash inflows 1,52,500 1,77,500 7,77,500
*Because the old machinery is not in use, so, the increase in maintenance cost will be a saving.

3. Incremental depreciation
Particulars Year 1 to 5
Depreciation on new machine [(₹15,00,000 − ₹6,00,000)/5 𝑦𝑒𝑎𝑟𝑠] 1,80,000
Depreciation on old machine [(₹10,00,000 − ₹0)/8 𝑦𝑒𝑎𝑟𝑠] -1,25,000
Incremental depreciation 55,000

4. Calculation of net present value


Year 𝑪𝑭𝑨𝑻𝒕 𝑷𝑽𝑭𝟏𝟎%,𝒏 Present value at 10%
1 1,52,500 0.909 1,38,623
2 1,52,500 0.826 1,25,965
3 1,77,500 0.751 1,33,302
4 1,77,500 0.683 1,21,233
5 7,77,500 0.621 4,82,828
Total present value 10,01,951
Less: Initial cash outlay -8,87,500
Net Present Value (NPV) 1,14,451
Decision: Since the NPV is positive, the machine may be replaced with the new machine.

What will be your decision if the expected salvage value of the new
machine is ₹2,00,000?
1. Initial cash outflows/outlay
Cost of the machine 15,00,000
Less: Scrap value of old machine -6,00,000
Less: Tax savings on gain on sales of old machine (See Note) -12,500
Cash outflows 8,87,500
Note:
Salvage value ₹6,00,000
Book value at the end of 3 year**
rd ₹6,25,000
Loss on sales of machinery ₹25,000

Tax savings on capital loss is ₹12,500 i.e. (𝐶𝑎𝑝𝑖𝑡𝑎𝑙 𝑙𝑜𝑠𝑠 × 𝑇𝑎𝑥 𝑟𝑎𝑡𝑒) ⇒ (₹25,000 × 0.50)

*Annual depreciation is ₹1,25,000 i.e. ₹10,00,000/8 𝑦𝑒𝑎𝑟𝑠


**Book value at the end of 3 years is ₹6,25,000 i.e. ₹10,00,000 − (₹1,25,000 × 3 𝑦𝑒𝑎𝑟𝑠)

2. Incremental cash inflows


Particulars Year 1 to 2 Year 3 to 4 Year 5
Increase in revenues 1,50,000 1,50,000 1,50,000
Savings due to reduction in operating expenses 1,00,000 1,00,000 1,00,000
Increase in maintenance cost of old machinery* -- 50,000 50,000
Total savings/profits 2,50,000 3,00,000 3,00,000
Less: Incremental depreciation (See below table 3) -1,35,000 -1,35,000 -1,35,000
Profits before taxes 1,15,000 1,65,000 1,65,000
Less: Tax @ 50% -57,500 -82,500 -82,500
Profits after taxes 57,500 82,500 82,500
Add: Incremental depreciation 1,35,000 1,35,000 1,35,000
Add: Salvage value -- -- 2,00,000
Incremental cash inflows 1,92,500 2,17,500 4,17,500
*Because the old machinery is not in use, so, the increase in maintenance cost will be a saving

Chapter 5 and 6, Capital Budgeting: 46


3. Incremental depreciation
Particulars Year 1 to 5
Depreciation on new machine [(₹15,00,000 − 2,00,000)/5 𝑦𝑒𝑎𝑟𝑠] 2,60,000
Depreciation on old machine -1,25,000
Incremental depreciation 1,35,000

4. Calculation of net present value


Year 𝑪𝑭𝑨𝑻𝒕 𝑷𝑽𝑭𝟏𝟎%,𝒏 Present value at 10%
1 1,92,500 0.909 1,74,983
2 1,92,500 0.826 1,59,005
3 2,17,500 0.751 1,63,343
4 2,17,500 0.683 1,48,553
5 4,17,500 0.621 2,59,268
Total present value 9,05,152
Less: Initial cash outlay -8,87,500
Net Present Value (NPV) 17,652
Decision: Since the NPV is positive, the machine may be replaced with the new machine.

Example 26 (Illustration 12)


FH hospital is considering to purchase a CT-scan machine. Presently, the hospital is outsourcing
the CT-scan Machine and is earning a commission of ₹15,000 per month (net of tax). The following
details are given regarding the machine— after deduction of applicable tax
Cost of CT-scan machine ₹15,00,000
Operating cost per annum (excluding depreciation) ₹2,25,000
Expected revenue per annum ₹7,90,000
Salvage value of the machine (after 5 years) ₹3,00,000
Expected life of the machine 5 years
Assuming tax rate of 30% and discount rate of 12%, whether it would be profitable to the hospital
to purchase the machine? Give your recommendation under net present value method and
profitability index method.
(CA, IPCC, May 2014)

Solution
1. Initial cash outflows/outlay
Cost of the machine ₹15,00,000

2. Cash inflows
Year 1 to 4 Year 5
Revenue 7,90,000 7,90,000
Less: Operating cost -2,25,000 -2,25,000
Less: Depreciation (₹15,00,000 − ₹3,00,000)/5 𝑦𝑒𝑎𝑟𝑠 -2,40,000 -2,40,000
Profits before tax 3,25,000 3,25,000
Less: Tax @ 30% -97,500 -97,500
Profits after tax 2,27,500 2,27,500
Add: Depreciation 2,40,000 2,40,000
Less: Loss of commission income (₹15,000 × 12 𝑚𝑜𝑛𝑡ℎ𝑠) -1,80,000 -1,80,000
Add: Scrap value -- 3,00,000
Net annual cash inflows 2,87,500 5,87,500

3. Calculation of net present value


Year 𝑪𝑭𝑨𝑻𝒕 𝑷𝑽𝑭𝟏𝟐%,𝒏 Present value at 12%
1 2,87,500 0.893 2,56,737.50
2 2,87,500 0.797 2,29,137.50
3 2,87,500 0.712 2,04,700.00
4 2,87,500 0.636 1,82,850.00

Chapter 5 and 6, Capital Budgeting: 47


5 5,87,500 0.567 3,33,112.50
Total present value 12,06,537.50
Less: Initial cash outlay -15,00,000.00
Net Present Value (NPV) -2,93,462.50

4. Profitability index
𝑃𝑟𝑒𝑠𝑒𝑛𝑡 𝑣𝑎𝑙𝑢𝑒 𝑜𝑓 𝑡ℎ𝑒 𝑐𝑎𝑠ℎ 𝑖𝑛𝑓𝑙𝑜𝑤𝑠
𝑃𝐼/𝐵𝐶 𝑅𝑎𝑡𝑖𝑜 =
𝑃𝑟𝑒𝑠𝑒𝑛𝑡 𝑣𝑎𝑙𝑢𝑒 𝑜𝑓 𝑡ℎ𝑒 𝑐𝑎𝑠ℎ 𝑜𝑢𝑡𝑓𝑙𝑜𝑤𝑠
12,06,537.50
𝑃𝐼/𝐵𝐶 𝑅𝑎𝑡𝑖𝑜 = = 0.8044
15,00,000

Decision: Since the NPV is negative and the PI is less than 1, so, the project should not be accepted.

Example 27 (Illustration 13)


A company is considering a proposal for production of a new product. The company expects to sell
1,00,000 units of the new product each year at a selling price of ₹5 per unit. Variable cost will be ₹2
per unit. Regardless of the level of production, the company will incur cash cost of ₹50,000 per year
if project is undertaken. The machine for making the product will cost ₹5,00,000 and can be sold for
₹60,000 after the end of its life of 5 years. Additional working capital required will be ₹50,000. The Learn this point
overhead allocated to the new product will be ₹24,000 per year. Tax rate is 30% and cost of capital
for the company is 15%. The company charges depreciation at 25% of the written down value.
Should the company buy the machine?
(B. Com. Honors, Delhi University, 2017)

Solution
1. Initial cash outflows/outlay
Cost of the machine 5,00,000
Add: Additional working capital investment 50,000
Cash outflows 5,50,000

2. Cash inflows
Year
Particulars 1 2 3 4 5
Sales (1,00,000 𝑢𝑛𝑖𝑡𝑠 × ₹5 𝑝𝑒𝑟 𝑢𝑛𝑖𝑡) 5,00,000 5,00,000 5,00,000 5,00,000 5,00,000
Less: Variable cost -2,00,000 -2,00,000 -2,00,000 -2,00,000 2,00,000
Less: Cash fixed cost -50,000 -50,000 -50,000 -50,000 -50,000
Profits before depreciation and tax 2,50,000 2,50,000 2,50,000 2,50,000 2,50,000
Less: Depreciation @ 25% on written down value -1,25,000 -93,750 -70,312 -52,735 -39,551
Profits before tax 1,25,000 1,56,250 1,79,688 1,97,265 2,10,449
Less: Tax @ 30% -37,500 -46,875 -53,906 -59,180 -63,135
Profits after tax 87,500 1,09,375 1,25,782 1,38,085 1,47,314
Add: Depreciation (See below table 3) 1,25,000 93,750 70,312 52,735 39,551
Add: Scrap value of new machine at the end of 5th year -- -- -- -- 60,000
Add: Recovery of working capital -- -- -- -- 50,000
Add: Tax savings on capital loss (See below table 4) -- -- -- -- 17,596
Cash inflows 2,12,500 2,03,125 1,96,094 1,90,820 3,14,461

3. Depreciation
Year
Particulars 1 2 3 4 5
Book value at the beginning 5,00,000 3,75,000 2,81,250 2,10,938 1,58,203
Less: Depreciation on machine @ 25% on
written down value -1,25,000 -93,750 -70,312 -52,735 -39,551
Book value at the end of the year 3,75,000 2,81,250 2,10,938 1,58,023 1,18,652

4. Tax saving on capital loss


Salvage value 60,000 Tax savings on capital loss is
Book value 1,18,652 ₹17,596 i.e. (𝐶𝑎𝑝𝑖𝑡𝑎𝑙 𝑙𝑜𝑠𝑠 × 𝑇𝑎𝑥 𝑟𝑎𝑡𝑒) ⇒ (₹58,652 ×
Loss on sales of machinery 58,652 0.30)

Chapter 5 and 6, Capital Budgeting: 48


5. Calculation of net present value
Year 𝑪𝑭𝑨𝑻𝒕 𝑷𝑽𝑭𝟏𝟓%,𝒏 Present value at 15%
1 2,12,500 0.870 1,84,875
2 2,03,125 0.756 1,53,563
3 1,96,094 0.658 1,29,030
4 1,90,820 0.572 1,09,149
5 3,14,461 0.497 1,56,287
Total present value 7,32,904
Less: Initial cash outlay -5,50,000
Net Present Value (NPV) 1,82,904
Decision: Since the NPV is positive, the machine may be replaced with the new machine.
Note: Allocated overhead costs have been ignored while estimating cash inflows as these will have
to be paid regardless of acceptance or non-acceptance of this project.

Example 28 (Illustration 14)


The Eastern Corporation Limited, a firm in the 40% tax bracket with a 15% required rate of
return is considering a new project. This project involves the introduction of a new product. This
project is expected to last five years and then to be terminated. Given the following information,
determine the after-tax cash flows associated with the project and then find the project’s net
present value and advise the company whether it should invest in project or not. Cost of the new
plant is ₹20,90,000 and shipping and installation cost is ₹30,000. Other details are as follows—
Year 1 2 3 4 5
Units sold 10,000 13,000 16,000 10,000 6,000
Sales price per unit 500 500 500 500 380
Variable cost per unit 260 260 260 260 260
Annual fixed cost 30,000 30,000 30,000 30,000 30,000
net - after
considering all
expenses, Working capital requirements: There will be an initial working capital requirement of ₹80,000
deductions, just to get production started. For each year, the total investment in net-working capital will be
liabilities
equal to 10% of the rupee value of sales for that year. Thus, the investment in working capital will
increase during years 1 through 3, then decrease in year 4. Finally, all/total working capital is
liquidated at the termination of the project at the end of the year 5.
The depreciation method: Use straight-line method for providing depreciation over five years
assuming that the plant and equipment will have no salvage value after five years.
(B. Com. Honors, Delhi University, 2010)

Solution
1. Initial cash outflows/outlay
Cost of new plant 20,90,000
Add: Shipping and installation cost 30,000
Add: Initial working capital requirement 80,000
Cash outflows 22,00,000

2. Depreciation and annual fixed cost


20,90,000 + 30,000
𝐴𝑛𝑛𝑢𝑎𝑙 𝑑𝑒𝑝𝑟𝑒𝑐𝑖𝑎𝑡𝑖𝑜𝑛 (1 − 5 𝑦𝑒𝑎𝑟𝑠) = 4,24,000
5
Annual fixed cost 30,000
Depreciation + Annual fixed cost 4,54,000

3. Profits before tax


Variable Dep. +
Units Selling cost at ₹260 Fixed Cost Total cost Profits
Year sold price (₹) Sales (₹) per unit (₹) (₹) before tax
(a) (b) (c) (d) (e) (f) (g = e + f) (h = d – g)
1 10,000 500 50,00,000 26,00,000 4,54,000 30,54,000 19,46,000

Chapter 5 and 6, Capital Budgeting: 49


2 13,000 500 65,00,000 33,80,000 4,54,000 38,34,000 22,66,000
3 16,000 500 80,00,000 41,60,000 4,54,000 46,14,000 33,86,000
4 10,000 500 50,00,000 26,00,000 4,54,000 30,54,000 19,46,000
5 6,000 380 22,80,000 15,60,000 4,54,000 20,14,000 2,66,000

4. Subsequent cash inflows


Working Increase /
capital req. decrease in
Profits Tax @ Profits (10% of working Net cash
Year before tax 40% after tax Depreciation sales) capital inflows
(d = b – (h = d + e –
(a) (b) (c) c) (e) (f) (g) g)
1 19,46,000 7,48,400 11,67,600 4,24,000 5,00,000 +4,20,000 11,71,600
2 26,66,000 10,66,400 15,99,600 4,24,000 6,50,000 +1,50,000 18,73,600
3 33,86,000 13,54,400 20,31,600 4,24,000 8,00,000 +1,50,000 23,05,600
4 19,46,000 7,78,400 11,67,600 4,24,000 5,00,000 -3,00,000 18,91,600
5 2,66,000 1,06,400 1,59,600 4,24,000 2,28,000 -2,72,000 10,83,600
5 -- -- -- -- -- -2,28,000

5. Calculation of net present value


Year 𝑪𝑭𝑨𝑻𝒕 𝑷𝑽𝑭𝟏𝟓%,𝒏 Present value at 15%
1 11,71,600 0.870 10,19,292
2 18,73,600 0.756 14,16,442
3 23,05,600 0.658 15,17,085
4 18,91,600 0.572 10,81,995
5 10,83,600 0.497 5,38,549
Total present value 55,73,363
Less: Initial cash outlay -22,00,000
Net Present Value (NPV) 33,73,363
Decision: Since the NPV is positive, so, the company should invest in the project.
VERY IMPORTANT
Example 29 (Illustration 16)
A particular project has a four-year life with yearly projected net profit of ₹10,000 after charging
yearly depreciation of ₹8,000 in order to write-off the capital cost of ₹32,000. Out of the capital cost,
₹20,000 is payable immediately (in 0 year) and balance in the next year (which will be needed for
evaluation). Stock amounting to ₹6,000 (to be invested in year 0) will be required throughout the
project and for debtors a further sum of ₹8,000 will have to be invested in year 1. The working
capital will be recouped in year 5. It is expected that the machinery will fetch a residual value of
₹2,000 at the end of 4th year. Income tax is payable @ 40% and it is paid 9 months after the end of
the year when profit is made. The residual value of ₹2,000 will also bear tax @ 40%. Although the
project is for 4 years, for computation of tax and realization of working capital, the computation will
be required up to 5 years. Cost of capital is 10%. Advise, whether the company should invest in the
project?
(ICWA Final, 2001; B. Com. Honors, Delhi University, 2013, Modified)

Solution
1. Cash outflows and inflows (combined)
Year
Particulars 0 1 2 3 4 5
- Capital cost -20,000 -12,000 -- -- -- --
- Investment in working capital -6,000 -8,000 -- -- -- --
+ Projected net profit after dep. -- 10,000 10,000 10,000 10,000 --
+ Depreciation -- 8,000 8,000 8,000 8,000 --
+ Residual value -- -- -- -- 2,000 --
- Payment of tax -- -- -4,000 -4,000 -4,000 -4,800
+ Recovery of working capital -- -- -- -- -- 14,000
Net cash flows -26,000 -2,000 14,000 14,000 16,000 9,200
Chapter 5 and 6, Capital Budgeting: 50
2. Calculation of net present value
Year 𝑪𝑭𝒕 𝑷𝑽𝑭𝟏𝟎%,𝒏 Present value at 10%
0 -26,000 1.000 -26,000
1 -2,000 0.909 -1,818
2 14,000 0.826 11,564
3 14,000 0.751 10,514
4 16,000 0.683 10,928
5 9,200 0.621 5,713
Net Present Value (NPV) 10,901

3. Calculation of tax payable


Year
Particulars 1 2 3 4 5
Net profits (after charging yearly depreciation) 10,000 10,000 10,000 10,000 --
Residual value/Capital gain -- -- -- 2,000 --
Total taxable income 10,000 10,000 10,000 12,000 --
Tax @ 40% payable (in the next year) 4,000 4,000 4,000 4,800
Decision: Since the NPV is positive, so, the company should invest in the project.

Example 30 (Illustration 23)


ABC & Co. is considering a proposal to replace one of its plants costing ₹60,000 and having a written
down value of ₹24,000. The remaining economic life of the plant is 4 years after which it will have
no salvage value. However, if sold today, it has a salvage value of ₹20,000. The new machine costing
₹1,30,000 is also expected to have a life of 4 years with a scrap value of ₹18,000. The new machine,
due to its technological superiority, is expected to contribute additional annual benefit (before
depreciation and tax) of ₹60,000. Find out the cash flows associated with the decision given that the
tax rate applicable to the firm is 40%. (The capital gain or loss may be taken as not subject to tax)
(B. Com. Honors, Delhi University, 2014)

Solution
1. Initial cash outflows/outlay
Cost of new plant 1,30,000
Less: Scrap value of the old machine -20,000
Cash outflows 1,10,000

2. Incremental depreciation
𝐴𝑛𝑛𝑢𝑎𝑙 𝑑𝑒𝑝𝑟𝑒𝑐𝑖𝑎𝑡𝑖𝑜𝑛 𝑜𝑛 𝑛𝑒𝑤 𝑚𝑎𝑐ℎ𝑖𝑛𝑒
𝐶𝑜𝑠𝑡 𝑜𝑓 𝑚𝑎𝑐ℎ𝑖𝑛𝑒𝑟𝑦 − 𝑆𝑐𝑟𝑎𝑝 𝑣𝑎𝑙𝑢𝑒
= ⇒
𝐸𝑠𝑡𝑖𝑚𝑎𝑡𝑒𝑑 𝑙𝑖𝑓𝑒 ₹28,000
1,30,000 − 18,000
=
4
𝐵𝑜𝑜𝑘 𝑣𝑎𝑙𝑢𝑒 24,000
𝐴𝑛𝑛𝑢𝑎𝑙 𝑑𝑒𝑝𝑟𝑒𝑐𝑖𝑎𝑡𝑖𝑜𝑛 𝑜𝑛 𝑜𝑙𝑑 𝑚𝑎𝑐ℎ𝑖𝑛𝑒 = ⇒= ₹6,000
𝐸𝑠𝑡𝑖𝑚𝑎𝑡𝑒𝑑 𝑙𝑖𝑓𝑒 4
Incremental depreciation ₹22,000
Note: Scrap value of old machine at the end of its life is not given in the question.

3. Net annual cash inflows


Particulars Year (1 to 3) Year 4
Profits before depreciation and tax (Additional benefit) 60,000 60,000
Less: Incremental depreciation (See table 2 above) -22,000 -22,000
Profits before tax 38,000 38,000
Less: Tax @ 40% -15,200 -15,200
Profits after tax 22,800 22,800
Add: Incremental depreciation 22,000 22,000

Chapter 5 and 6, Capital Budgeting: 51


Add: Scrap value of machine -- 18,000
Net cash inflows 44,800 62,800

Example 31 (Illustration 24)


ABC Limited is trying to decide whether it should replace a manually operated machine with a fully
automated machine of the same version. The existing machine purchased 5 years ago has now book
value of ₹1,00,000 and remaining life of 10 years. The existing machine can now be scrapped away
for ₹50,000. There is a proposal to replace the existing machine with a new automatic machine
which has a cost price of ₹3,00,000. The new machine is also expected to have 10 years of life with a
salvage value of ₹60,000. The new machine has a cost savings in materials of ₹50,000 per year. The
tax rate is 40% and cost of capital is 11%. Depreciation would be provided as per straight line
method. Should the machine be replaced?
(B. Com. Honors, Delhi University, 2015)

Solution
1. Initial cash outflows/outlay
Cost of the machine 3,00,000
Less: Scrap value of old machine -50,000
Less: Tax savings on loss on sales of old machine (See Note) -20,000
Cash outflows 2,30,000
Note:
Salvage value ₹50,000
Book value ₹1,00,000
Loss on sales of machinery ₹50,000
Tax savings on capital loss is ₹20,000 i.e. (𝐶𝑎𝑝𝑖𝑡𝑎𝑙 𝑙𝑜𝑠𝑠 × 𝑇𝑎𝑥 𝑟𝑎𝑡𝑒) ⇒ (₹50,000 × 0.40)

2. Incremental depreciation
𝐴𝑛𝑛𝑢𝑎𝑙 𝑑𝑒𝑝𝑟𝑒𝑐𝑖𝑎𝑡𝑖𝑜𝑛 𝑜𝑛 𝑛𝑒𝑤 𝑚𝑎𝑐ℎ𝑖𝑛𝑒
𝐶𝑜𝑠𝑡 𝑜𝑓 𝑚𝑎𝑐ℎ𝑖𝑛𝑒𝑟𝑦 − 𝑆𝑐𝑟𝑎𝑝 𝑣𝑎𝑙𝑢𝑒
= ⇒
𝐸𝑠𝑡𝑖𝑚𝑎𝑡𝑒𝑑 𝑙𝑖𝑓𝑒 ₹24,000
3,00,000 − 60,000
=
10
𝐵𝑜𝑜𝑘 𝑣𝑎𝑙𝑢𝑒
𝐴𝑛𝑛𝑢𝑎𝑙 𝑑𝑒𝑝𝑟𝑒𝑐𝑖𝑎𝑡𝑖𝑜𝑛 𝑜𝑛 𝑜𝑙𝑑 𝑚𝑎𝑐ℎ𝑖𝑛𝑒 = ⇒
𝐸𝑠𝑡𝑖𝑚𝑎𝑡𝑒𝑑 𝑙𝑖𝑓𝑒 ₹10,000
1,00,000
=
10
Incremental depreciation ₹14,000
Note: Scrap value of old machine at the end of its life is not given in the question.

3. Net annual cash inflows


Particulars Year (1 to 9) Year 10
Savings in material cost 50,000 50,000
Less: Incremental depreciation (See above table 2) -14,000 -14,000
Profits before tax 36,000 36,000
Less: Tax @ 40% -14,400 -14,400
Profits after tax 21,600 21,600
Add: Incremental depreciation 14,000 14,000
Add: Scrap value of machine -- 60,000
Net cash inflows 35,600 95,600

4. Calculation of net present value


Year 𝑪𝑭𝑨𝑻𝒕 𝑷𝑽𝑭𝟏𝟏%,𝒏 Present value at 11%
0 -2,30,000 1.000 -2,30,000.00
1 35,600 0.901 32,075.60
2 35,600 0.812 28,907.20

Chapter 5 and 6, Capital Budgeting: 52


3 35,600 0.731 26,023.60
4 35,600 0.659 23,460.40
5 35,600 0.593 21,110.80
6 35,600 0.535 19,046.00
7 35,600 0.482 17,159.20
8 35,600 0.434 15,450.40
9 35,600 0.391 13,919.60
10 95,600 0.352 33,651.20
Net Present Value (NPV) ₹804.00
Decision: Since the NPV is positive, so, the company should replace the machine.

Example 32 (Illustration 25)


X Limited has a machine having an additional life of 5 years which costs ₹10,00,000 and has a
book value of 4,00,000. A new machine costing ₹20,00,000 is available. Though its capacity is the
same as that of the old machine, it will mean a saving in variable cost to the extent of ₹7,00,000 per
annum. The life of the machine will be 5 years at the end of which it will have a scrap value of
₹2,00,000. The rate of income tax is 40% and X Limited’s policy is not to make an investment if the
yield is less than 12% per annum. The old machine, if sold today, will realize ₹1,00,000; it will have
no salvage value if sold at the end of 5th year. Advise X Limited whether or not the old machine
should be replaced. Capital gain is tax free. Ignore income tax savings on additional depreciation as
well as loss due to sale of existing machine.
Will it make any difference, if the additional depreciation (on new machine) and capital gain on sale
of old machine is also subject to same tax at the rate of 40%, and the scrap value of the new machine
is ₹3,00,000?
(B. Com. Honors, Delhi University, 2012, 2016)

Solution

(i) When tax on additional depreciation, loss on sale of existing machine


and capital gain is to be ignored
1. Initial cash outflows/outlay
Cost of the machine 20,00,000
Less: Scrap value of old machine -1,00,000
Cash outflows 19,00,000

2. Net annual cash inflows


Particulars Year (1 to 4) Year 5
Savings in variable cost 7,00,000 7,00,000
Less: Incremental Depreciation (See below table 3) -2,80,000 -2,80,000
Profits before tax 4,20,000 4,20,000
Less: Tax @ 40% -1,68,000 -1,68,000
Profits after tax 2,52,000 2,52,000
Add: Incremental depreciation 2,80,000 2,80,000
Add: Scrap value of machine -- 2,00,000
Net cash inflows 5,32,000 7,32,000

3. Incremental depreciation
𝐴𝑛𝑛𝑢𝑎𝑙 𝑑𝑒𝑝𝑟𝑒𝑐𝑖𝑎𝑡𝑖𝑜𝑛 𝑜𝑛 𝑛𝑒𝑤 𝑚𝑎𝑐ℎ𝑖𝑛𝑒
𝐶𝑜𝑠𝑡 𝑜𝑓 𝑚𝑎𝑐ℎ𝑖𝑛𝑒𝑟𝑦 − 𝑆𝑐𝑟𝑎𝑝 𝑣𝑎𝑙𝑢𝑒
= ⇒
𝐸𝑠𝑡𝑖𝑚𝑎𝑡𝑒𝑑 𝑙𝑖𝑓𝑒 ₹3,60,000
20,00,000 − 2,00,000
=
5

Chapter 5 and 6, Capital Budgeting: 53


𝐵𝑜𝑜𝑘 𝑣𝑎𝑙𝑢𝑒
𝐴𝑛𝑛𝑢𝑎𝑙 𝑑𝑒𝑝𝑟𝑒𝑐𝑖𝑎𝑡𝑖𝑜𝑛 𝑜𝑛 𝑜𝑙𝑑 𝑚𝑎𝑐ℎ𝑖𝑛𝑒 = ⇒
𝐸𝑠𝑡𝑖𝑚𝑎𝑡𝑒𝑑 𝑙𝑖𝑓𝑒 ₹80,000
4,00,000
=
5
Incremental depreciation ₹2,80,000
Note: Scrap value of old machine at the end of its life is not given in the question.

4. Calculation of net present value


Year 𝑪𝑭𝑨𝑻𝒕 𝑷𝑽𝑭𝟏𝟐%,𝒏 Present value at 12%
1 5,32,000 0.893 4,75,076
2 5,32,000 0.797 4,24,004
3 5,32,000 0.712 3,78,784
4 5,32,000 0.636 3,38,352
5 7,32,000 0.567 4,15,044
Present value of cash inflows 20,31,260
Less: Initial cash outlay -19,00,000
Net Present Value (NPV) 1,31,260
Decision: Since the NPV is positive, so, the company should replace the old machine.

(ii) When tax on additional depreciation, loss on sale of existing machine


and capital gain is not to be ignored and scrap value is ₹3,00,000
1. Initial cash outflows/outlay
Cost of new machine 20,00,000
Less: Scrap value of old machine -1,00,000
Less: Tax savings on loss on sales of old machine (See Note) -1,20,000
Cash outflows 17,80,000
Note:
Salvage value ₹1,00,000
Book value ₹4,00,000
Loss on sales of machinery ₹3,00,000
Tax savings on capital loss is ₹1,20,000 i.e. (𝐶𝑎𝑝𝑖𝑡𝑎𝑙 𝑙𝑜𝑠𝑠 × 𝑇𝑎𝑥 𝑟𝑎𝑡𝑒) ⇒ (₹3,00,000 × 0.40)

2. Net annual cash inflows


Particulars Year (1 to 4) Year 5
Savings in variable cost 7,00,000 7,00,000
Less: Incremental depreciation (See below table 3) -2,60,000 -2,60,000
Profits before tax 4,40,000 4,40,000
Less: Tax @ 40% -1,76,000 -1,76,000
Profits after tax 2,64,000 2,64,000
Add: Incremental depreciation 2,60,000 2,60,000
Add: Scrap value of machine -- 3,00,000
Net cash inflows 5,24,000 8,24,000

3. Incremental depreciation
𝐴𝑛𝑛𝑢𝑎𝑙 𝑑𝑒𝑝𝑟𝑒𝑐𝑖𝑎𝑡𝑖𝑜𝑛 𝑜𝑛 𝑛𝑒𝑤 𝑚𝑎𝑐ℎ𝑖𝑛𝑒
𝐶𝑜𝑠𝑡 𝑜𝑓 𝑚𝑎𝑐ℎ𝑖𝑛𝑒𝑟𝑦 − 𝑆𝑐𝑟𝑎𝑝 𝑣𝑎𝑙𝑢𝑒
= ⇒
𝐸𝑠𝑡𝑖𝑚𝑎𝑡𝑒𝑑 𝑙𝑖𝑓𝑒 ₹3,40,000
20,00,000 − 3,00,000
=
5
𝐵𝑜𝑜𝑘 𝑣𝑎𝑙𝑢𝑒
𝐴𝑛𝑛𝑢𝑎𝑙 𝑑𝑒𝑝𝑟𝑒𝑐𝑖𝑎𝑡𝑖𝑜𝑛 𝑜𝑛 𝑜𝑙𝑑 𝑚𝑎𝑐ℎ𝑖𝑛𝑒 = ⇒
𝐸𝑠𝑡𝑖𝑚𝑎𝑡𝑒𝑑 𝑙𝑖𝑓𝑒 ₹80,000
4,00,000
=
5
Incremental depreciation ₹2,60,000
Note: Scrap value of old machine at the end of its life is not given in the question.

Chapter 5 and 6, Capital Budgeting: 54


4. Calculation of net present value
Year 𝑪𝑭𝑨𝑻𝒕 𝑷𝑽𝑭𝟏𝟐%,𝒏 Present value at 12%
1 5,24,000 0.893 4,67,932
2 5,24,000 0.797 4,17,628
3 5,24,000 0.712 3,73,088
4 5,24,000 0.636 3,33,264
5 8,24,000 0.567 4,67,208
Present value of cash inflows 20,59,120
Less: Initial cash outlay -17,80,000
Net Present Value (NPV) 2,79,120
Decision: Since the NPV is positive, so, the company should replace the old machine.

Example 33 (Illustration 26)


AC Limited has just installed machine ‘X’ at a cost of ₹4,00,000 having a useful life of 5 years with
no residual value. The annual production is estimated at 1,50,000 units which can be sold at ₹12 per
unit. Annual operating costs are estimated at ₹4,00,000 (excluding depreciation) at this output
level. Fixed costs are estimated at ₹6 per unit (excluding depreciation) for the same level of output.
The company has just come across another machine ‘Y’ having same useful life and capable of giving
same output. Annual operating cost is expected at ₹3,60,000 (excluding depreciation). There will be
no change in fixed cost. Capital cost of the machine is ₹5,00,000 with nil residual value.
The company can sell the machine ‘X’ at ₹1,00,000 but the cost of dismantling and removal will
amount to ₹30,000. The operations with machine ‘X’ have not yet started and the company wants to
sell machine ‘X’ and purchase ‘Y’.
AC Limited provides depreciation under straight line method. Assume corporate tax at 40%. The
cost of capital may be assumed at 14%.
(a) Advise whether the company should opt for replacement.
(b) Will there be any change in your view if machine ‘X’ has not been installed and the company has
to select any one of the two machines?
(B. Com. Honors, Delhi University, 2016)

Solution

(a) Advise whether the company should opt for replacement.


1. Initial cash outflows/outlay
Cost of new machine 5,00,000
Less: Scrap value of old machine (₹1,00,000-₹30,000 dismantling charges) -70,000
Less: Tax savings on loss on sales of old machine (See Note) -1,32,000
Cash outflows 2,98,000
Note:
Net Salvage value of X (₹1,00,000-₹30,000 dismantling charges) ₹70,000
Book value ₹4,00,000
Loss on sales of machinery ₹3,30,000
Tax savings on capital loss is ₹1,32,000 i.e. (𝐶𝑎𝑝𝑖𝑡𝑎𝑙 𝑙𝑜𝑠𝑠 × 𝑇𝑎𝑥 𝑟𝑎𝑡𝑒) ⇒ (₹3,30,000 × 0.40)

2. Net annual cash inflows


Particulars Year (1 to 5)
Savings in operating cost (₹4,00,000-₹3,60,000) 40,000
Less: Incremental depreciation (See below table 3) -20,000
Profits before tax 20,000
Less: Tax @ 40% -8,000
Profits after tax 12,000
Add: Incremental depreciation 20,000
Net cash inflows 32,000

3. Incremental depreciation
Chapter 5 and 6, Capital Budgeting: 55
𝐴𝑛𝑛𝑢𝑎𝑙 𝑑𝑒𝑝𝑟𝑒𝑐𝑖𝑎𝑡𝑖𝑜𝑛 𝑜𝑛 𝑛𝑒𝑤 𝑚𝑎𝑐ℎ𝑖𝑛𝑒
𝐶𝑜𝑠𝑡 𝑜𝑓 𝑚𝑎𝑐ℎ𝑖𝑛𝑒𝑟𝑦 − 𝑆𝑐𝑟𝑎𝑝 𝑣𝑎𝑙𝑢𝑒
= ⇒
𝐸𝑠𝑡𝑖𝑚𝑎𝑡𝑒𝑑 𝑙𝑖𝑓𝑒 ₹1,00,000
5,00,000 − 0
=
5
𝐵𝑜𝑜𝑘 𝑣𝑎𝑙𝑢𝑒
𝐴𝑛𝑛𝑢𝑎𝑙 𝑑𝑒𝑝𝑟𝑒𝑐𝑖𝑎𝑡𝑖𝑜𝑛 𝑜𝑛 𝑜𝑙𝑑 𝑚𝑎𝑐ℎ𝑖𝑛𝑒 = ⇒
𝐸𝑠𝑡𝑖𝑚𝑎𝑡𝑒𝑑 𝑙𝑖𝑓𝑒 ₹80,000
4,00,000
=
5
Incremental depreciation ₹20,000
Note: Scrap value of old machine at the end of its life is given nil in the question.

4. Calculation of net present value


Year 𝑪𝑭𝑨𝑻𝒕 𝑷𝑽𝑭𝟏𝟒%,𝒏 Present value at 14%
1 32,000 0.877 28,064
2 32,000 0.769 24,608
3 32,000 0.675 21,600
4 32,000 0.592 18,944
5 32,000 0.519 16,608
Present value of cash inflows 1,09,824
Less: Initial cash outlay -2,98,000
Net Present Value (NPV) -1,88,176
Decision: Since the NPV is negative, so, the company should not replace the machine ‘X’.

(b) Will there be any change in your view if machine ‘X’ has not been
installed and the company has to select any one of the two machines?
In such a case the decision regarding replacement is irrelevant as the machine ‘X’ has not been
installed. The profitability of both the machines shall be compared using NPV and decision of
investment in the best one shall be taken (Mutually exclusive decision).
1. Cash inflows
Particulars Machine ‘X’ Machine ‘Y’
Sales (𝟏, 𝟓𝟎, 𝟎𝟎𝟎 𝒖𝒏𝒊𝒕𝒔 × ₹𝟏𝟐 𝒑𝒆𝒓 𝒖𝒏𝒊𝒕) 18,00,000 18,00,000
Less: Operating costs -4,00,000 -3,60,000
Less: Fixed costs (1,50,000 𝑢𝑛𝑖𝑡𝑠 × ₹6 𝑝𝑒𝑟 𝑢𝑛𝑖𝑡) -9,00,000 -9,00,000
Less: Depreciation -80,000 -1,00,000
Profits before tax (Sales – Cost of sales) 4,20,000 4,40,000
Less: Tax @ 40% -1,68,000 -1,76,000
Profits after tax 2,52,000 2,64,000
Add: Depreciation 80,000 1,00,000
Cash inflows 3,32,000 3,64,000

2. Calculation of net present value


‘Y’
‘X’ ‘Y’ ‘X’ Present value at
Year 𝑪𝑭𝑨𝑻𝒕 𝑪𝑭𝑨𝑻𝒕 𝑷𝑽𝑭𝟏𝟒%,𝒏 Present value at 14% 14%
1 3,32,000 3,64,000 0.877 2,91,164 3,19,228
2 3,32,000 3,64,000 0.769 2,55,308 2,79,916
3 3,32,000 3,64,000 0.675 2,24,100 2,45,700
4 3,32,000 3,64,000 0.592 1,96,544 2,15,488
5 3,32,000 3,64,000 0.519 1,72,308 1,88,916
Present value of cash inflows 11,39,424 12,49,248
Less: Initial cash outlay -4,00,000 -5,00,000
Net Present Value (NPV) 7,39,424 7,49,248
Alternate method to calculate NPV
Machine ‘X’

Chapter 5 and 6, Capital Budgeting: 56


Present value of cash inflows [₹3,32,000 × 𝑃𝑉𝐴𝐹14%,5 ] ⇒ 3,32,000 × 3.433 ⇒ ₹11,39,756
Less: Initial cash outlay ₹4,00,000
Net present value ₹7,39,756
Machine ‘Y’
Present value of cash inflows [₹3,64,000 × 𝑃𝑉𝐴𝐹14%,5 ] ⇒ 3,64,000 × 3.433 ⇒ ₹12,49,612
Less: Initial cash outlay ₹5,00,000
Net present value ₹7,49,612
Decision: Since the NPV of machine ‘Y’ is higher, so, it should be purchased.

Example 34 (Illustration 30)


RS Limited wants to replace its labor-intensive manufacturing facility. The company is evaluating
a project to install a machining system. It is estimated that system will result in annual saving of
₹4,50,000 in wages, ₹1,50,000 in supervisory costs, ₹50,000 in material losses during production,
₹40,000 in inventory costs and ₹30,000 in other operating costs.
The machining system is likely to cost ₹7,50,000 and will require an installation cost of ₹50,000. Its
useful life is estimated as 5 years. To operate the system, company requires the services of two
operators at an annual salary of ₹1,50,000 each. Its annual repairs and maintenance cost is likely to
be ₹40,000. Assuming corporate and capital gain tax rates uniformly at 35% and the required rate
(iv) Book value = 0
Scrap value = 50000
of return on long-term investment proposal at 12%, it is required to calculate—
This clearly indicates that (i) the relevant cash flows of the machining system.
we have to make the
book value of the asset 0 (ii) the period of recovery, i.e. Payback period (PB).

life and inspite of 0 book (iii) net present value (NPV).


at the end of it's useful

value, it will be sold at


50000, hence CAPITAL
(iv) the relevant cash flows, PB period and NPV assuming the machining system can be sold for
GAIN of 50000 occurs on ₹50,000 at the end of 5 years when its book value is supposed to be zero.
which tax liability will
have to be paid. (v) the relevant cash flows, PB and NPV assuming that the depreciated book value of the system
upon termination at the end of 5th year will stood at ₹50,000. However, it will not fetch anything i.e.
(v) Book value = 50000
its sales value will be zero.
Scrap value = 0 (B. Com. Honors, Delhi University, 2016)
This clearly indicates that at the end of useful life of the asset, its value in books must reflect 50000, whereas it will give 0 ruppees when sold, this tells us a CAPITAL LOSS of 50000 and
hence making tax savings.
Solution

(i) The relevant cash flows of the machining system


1. Initial cash outflows/outlay
Cost of the machining system 7,50,000
Add: Installation cost 50,000
Cash outflows 8,00,000

2. Subsequent annual cash inflows


Particulars Year (1 to 5)
Savings in wages 4,50,000
Savings in supervisory costs 1,50,000
Savings in material losses during production 50,000
Savings in inventory costs 40,000
Savings in other operating costs 30,000
Total savings 7,20,000
Less: Annual salaries of two operators -3,00,000
Less: Annual repairs and maintenance costs -40,000
Profits before depreciation and taxes 3,80,000
Less: Depreciation (₹8,00,000/5 𝑦𝑒𝑎𝑟𝑠) -1,60,000
Profits before taxes 2,20,000
Less: Taxes @ 35% -77,000
Profits after taxes 1,43,000
Add: Depreciation 1,60,000
Cash inflows 3,03,000

Chapter 5 and 6, Capital Budgeting: 57


(ii) The period of recovery, i.e. Payback period (PB)
Calculation of payback period
𝐼𝑛𝑖𝑡𝑖𝑎𝑙 𝐼𝑛𝑣𝑒𝑠𝑡𝑚𝑒𝑛𝑡/𝑂𝑢𝑡𝑙𝑎𝑦 8,00,000
𝑃𝑎𝑦 𝑏𝑎𝑐𝑘 𝑝𝑒𝑟𝑖𝑜𝑑 = = = 2.64 𝑦𝑒𝑎𝑟𝑠
𝐶𝑜𝑛𝑠𝑡𝑎𝑛𝑡 𝑐𝑎𝑠ℎ 𝑓𝑙𝑜𝑤𝑠 𝑜𝑟 𝑎𝑛𝑛𝑢𝑎𝑙 𝑐𝑎𝑠ℎ 𝑖𝑛𝑓𝑙𝑜𝑤𝑠 3,03,000

(iii) Net present value (NPV)


Calculation of net present value
Year 𝑪𝑭𝑨𝑻𝒕 𝑷𝑽𝑭𝟏𝟐%,𝒏 Present value at 12%
1 3,03,000 0.893 2,70,579
2 3,03,000 0.797 2,41,491
3 3,03,000 0.712 2,15,736
4 3,03,000 0.636 1,92,708
5 3,03,000 0.567 1,71,801
Present value of cash inflows 10,92,315
Less: Initial cash outlay -8,00,000
Net Present Value (NPV) 2,92,315
Alternate method
Present value of cash inflows [₹3,03,000 × 𝑃𝑉𝐴𝐹12%,5 ] ⇒ 3,03,000 × 3.605 ⇒ ₹10,92,315
Less: Initial cash outlay ₹ 8,00,000
Net present value ₹ 2,92,315

(iv) The relevant cash flows, PB and NPV assuming the machining system
can be sold for ₹50,000 at the end of 5 years when its book value is
supposed to be zero
1. Initial cash outflows/outlay
Cost of the machining system 7,50,000
Add: Installation cost 50,000
Cash outflows 8,00,000

2. Subsequent annual cash inflows


Particulars Year (1 to 4) Year 5
Saving in wages 4,50,000 4,50,000
Saving in supervisory costs 1,50,000 1,50,000
Saving in material losses during production 50,000 50,000
Saving in inventory costs 40,000 40,000
Saving in other operating costs 30,000 30,000
Total savings 7,20,000 7,20,000
Less: Annual salaries of two operators -3,00,000 -3,00,000
Less: Annual repairs and maintenance costs -40,000 -40,000
Profits before depreciation and taxes 3,80,000 3,80,000
Less: Depreciation (₹8,00,000/5 𝑦𝑒𝑎𝑟𝑠) -1,60,000 -1,60,000
Profits before taxes 2,20,000 2,20,000
Less: Taxes @ 35% -77,000 -77,000
Profits after taxes 1,43,000 1,43,000
Add: Depreciation (₹8,00,000/5 𝑦𝑒𝑎𝑟𝑠) 1,60,000 1,60,000
Add: Scrap value -- 50,000
Less: Tax liability on capital gain (See below table 3) -- -17,500
Cash inflows 3,03,000 3,35,500

3. Tax liability on capital gain


Salvage value 50,000 Tax liability on capital gain is
Book value 0 ₹17,500 i.e. (𝐶𝑎𝑝𝑖𝑡𝑎𝑙 𝑔𝑎𝑖𝑛 × 𝑇𝑎𝑥 𝑟𝑎𝑡𝑒) ⇒ (₹50,000 ×
Gain on sales of machinery 50,000 0.35)
Chapter 5 and 6, Capital Budgeting: 58
Remember to find PAYBACK PERIOD in mixed stream questions like this:
4. Payback period
Year 𝐶𝐹𝐴𝑇𝑡 𝐶𝑢𝑚𝑢𝑙𝑎𝑡𝑖𝑣𝑒 𝐶𝐹𝐴𝑇𝑡
1 3,03,000 3,03,000
2 3,03,000 6,06,000
3 3,03,000 9,09,000
4 3,03,000 12,12,000
5 3,35,500 15,47,500
(8,00,000 − 6,06,000)
𝑃𝑎𝑦𝑏𝑎𝑐𝑘 𝑝𝑒𝑟𝑖𝑜𝑑 = 2 𝑦𝑒𝑎𝑟𝑠 + ⇒ 2.64 𝑦𝑒𝑎𝑟𝑠
3,03,000
or
𝐼𝑛𝑖𝑡𝑖𝑎𝑙 𝐼𝑛𝑣𝑒𝑠𝑡𝑚𝑒𝑛𝑡/𝑂𝑢𝑡𝑙𝑎𝑦 8,00,000
𝑃𝑎𝑦𝑏𝑎𝑐𝑘 𝑝𝑒𝑟𝑖𝑜𝑑 = =
𝐶𝑜𝑛𝑠𝑡𝑎𝑛𝑡 𝑐𝑎𝑠ℎ 𝑓𝑙𝑜𝑤𝑠 𝑜𝑟 𝑎𝑛𝑛𝑢𝑎𝑙 𝑐𝑎𝑠ℎ 𝑖𝑛𝑓𝑙𝑜𝑤𝑠 3,03,000
= 2.64 𝑦𝑒𝑎𝑟𝑠

5. Calculation of net present value


Year 𝑪𝑭𝑨𝑻𝒕 𝑷𝑽𝑭𝟏𝟐%,𝒏 Present value at 12%
1 3,03,000 0.893 2,70,579
2 3,03,000 0.797 2,41,491
3 3,03,000 0.712 2,15,736
4 3,03,000 0.636 1,92,708
5 3,35,500 0.567 1,90,229
Present value of cash inflows 11,10,743
Less: Initial cash outlay -8,00,000
Net Present Value (NPV) 3,10,743

(v) The relevant cash flows, PB and NPV assuming that the depreciated
book value of the system upon termination at the end of 5th year will
stood at ₹50,000. However, it will not fetch anything i.e. its sales value
will be zero
1. Initial cash outflows/outlay
Cost of the machining system 7,50,000
Add: Installation cost 50,000
Cash outflows 8,00,000

2. Subsequent annual cash inflows


Particulars Year (1 to 4) Year 5
Saving in wages 4,50,000 4,50,000
Saving in supervisory costs 1,50,000 1,50,000
Saving in material losses during production 50,000 50,000
Saving in inventory costs 40,000 40,000
Saving in other operating costs 30,000 30,000
Total savings 7,20,000 7,20,000
Less: Annual salaries of two operators -3,00,000 -3,00,000
Less: Annual repairs and maintenance costs -40,000 -40,000
Profits before depreciation and taxes 3,80,000 3,80,000
Less: Depreciation ((₹8,00,000 − ₹50,000)/5 𝑦𝑒𝑎𝑟𝑠) -1,50,000 -1,50,000
Profits before taxes 2,30,000 2,30,000
Less: Taxes @ 35% -80,500 -80,500
Profits after taxes 1,49,500 1,49,500
Add: Depreciation ((₹8,00,000 − ₹50,000)/5 𝑦𝑒𝑎𝑟𝑠) 1,50,000 1,50,000
Add: Scrap value -- 0
Add: Tax saving on capital loss (See below table 3) -- 17,500
Cash inflows 2,99,500 3,17,000

Chapter 5 and 6, Capital Budgeting: 59


3. Tax liability on capital gain
Salvage value 0 Tax saving on capital loss is
Book value 50,000 ₹17,500 i.e. (𝐶𝑎𝑝𝑖𝑡𝑎𝑙 𝑙𝑜𝑠𝑠 × 𝑇𝑎𝑥 𝑟𝑎𝑡𝑒) ⇒ (₹50,000 ×
Loss on sales of machinery 50,000 0.35)

4. Payback period
Year 𝐶𝐹𝐴𝑇𝑡 𝐶𝑢𝑚𝑢𝑙𝑎𝑡𝑖𝑣𝑒 𝐶𝐹𝐴𝑇𝑡
1 2,99,500 2,99,500
2 2,99,500 5,99,000
3 2,99,500 8,98,500
4 2,99,500 11,98,000
5 3,17,000 15,15,000
(8,00,000 − 5,99,000)
𝑃𝑎𝑦𝑏𝑎𝑐𝑘 𝑝𝑒𝑟𝑖𝑜𝑑 = 2 𝑦𝑒𝑎𝑟𝑠 + ⇒ 2.67 𝑦𝑒𝑎𝑟𝑠
2,99,500
or
𝐼𝑛𝑖𝑡𝑖𝑎𝑙 𝐼𝑛𝑣𝑒𝑠𝑡𝑚𝑒𝑛𝑡/𝑂𝑢𝑡𝑙𝑎𝑦 8,00,000
𝑃𝑎𝑦𝑏𝑎𝑐𝑘 𝑝𝑒𝑟𝑖𝑜𝑑 = =
𝐶𝑜𝑛𝑠𝑡𝑎𝑛𝑡 𝑐𝑎𝑠ℎ 𝑓𝑙𝑜𝑤𝑠 𝑜𝑟 𝑎𝑛𝑛𝑢𝑎𝑙 𝑐𝑎𝑠ℎ 𝑖𝑛𝑓𝑙𝑜𝑤𝑠 2,99,500
= 2.67 𝑦𝑒𝑎𝑟𝑠

5. Calculation of net present value


Year 𝑪𝑭𝑨𝑻𝒕 𝑷𝑽𝑭𝟏𝟐%,𝒏 Present value at 12%
1 2,99,500 0.893 2,67,454
2 2,99,500 0.797 2,38,702
3 2,99,500 0.712 2,13,244
4 2,99,500 0.636 1,90,482
5 3,17,000 0.567 1,79,739
Present value of cash inflows 10,89,621
Less: Initial cash outlay -8,00,000
Net Present Value (NPV) 2,89,621

Example 35 (Illustration 32)


Company UVW has to make a choice between two identical machines in terms of capacity, ‘A’ and Identification -
‘B’. They have been designed differently but do exactly the same job. CASH INFLOWS
Machine ‘A’ costs ₹7,50,000 and will last for three years. It costs ₹2,00,000 per year to run. are not given in
this formula
Machine ‘B’ is an economy model costing only ₹5,00,000 but will last for only two years. It costs questions
₹3,00,000 per year to run.
The cash flows of machine ‘A’ and ‘B’ are real cash flows. The costs are forecasted in rupees of
constant purchasing power. Ignore taxes. The opportunity cost of capital is 9%.
Required: Which machine the company UVW should buy?
(CA, PE-II, November 2006; B. Com. Honors, Delhi University, 2013)

Solution
In this question cash inflows are not given rather outflows (cost) are given, so, the viability of the
machines can be assessed using the equivalent annual cost of the machines. Formula of the
calculation of equivalent annual cost is as follows—
𝑃𝑟𝑒𝑠𝑒𝑛𝑡 𝑣𝑎𝑙𝑢𝑒 𝑜𝑓 𝑐𝑎𝑠ℎ 𝑜𝑢𝑡𝑓𝑙𝑜𝑤𝑠 𝑃𝑟𝑒𝑠𝑒𝑛𝑡 𝑣𝑎𝑙𝑢𝑒 𝑜𝑓 𝑐𝑎𝑠ℎ 𝑜𝑢𝑡𝑓𝑙𝑜𝑤𝑠
𝐸𝑞𝑢𝑖𝑣𝑎𝑙𝑒𝑛𝑡 𝑎𝑛𝑛𝑢𝑎𝑙 𝑐𝑜𝑠𝑡 = ⇒
𝑃𝑟𝑒𝑠𝑒𝑛𝑡 𝑣𝑎𝑙𝑢𝑒 𝑎𝑛𝑛𝑢𝑖𝑡𝑦 𝑓𝑎𝑐𝑡𝑜𝑟𝑟,𝑛 𝑃𝑉𝐴𝐹𝑟,𝑛
The machine with lower equivalent annual cost shall be preferred.
Machine ‘A’
Initial outlay 7,50,000
Present value of annual op. cost (₹2,00,000 × 𝑃𝑉𝐴𝐹9%,3 ) ⇒ (₹2,00,000 × 2.5313) 5,06,260
Present value of cash outflows 12,56,260
𝑃𝑟𝑒𝑠𝑒𝑛𝑡 𝑣𝑎𝑙𝑢𝑒 𝑜𝑓 𝑐𝑎𝑠ℎ 𝑜𝑢𝑡𝑓𝑙𝑜𝑤𝑠 12,56,260
𝐸𝑞𝑢𝑖𝑣𝑎𝑙𝑒𝑛𝑡 𝑎𝑛𝑛𝑢𝑎𝑙 𝑐𝑜𝑠𝑡 = ⇒ 4,96,290
𝑃𝑉𝐴𝐹9%,3 2.5313

Chapter 5 and 6, Capital Budgeting: 60


Machine ‘B’
Initial outlay 5,00,000
Present value of annual op. cost (𝑅𝑠. 3,00,000 × 𝑃𝑉𝐴𝐹9%,2 ) ⇒ (₹3,00,000 × 1.7591) 5,27,730
Present value of cash outflows 10,27,730
𝑃𝑟𝑒𝑠𝑒𝑛𝑡 𝑣𝑎𝑙𝑢𝑒 𝑜𝑓 𝑐𝑎𝑠ℎ 𝑜𝑢𝑡𝑓𝑙𝑜𝑤𝑠 10,27,730
𝐸𝑞𝑢𝑖𝑣𝑎𝑙𝑒𝑛𝑡 𝑎𝑛𝑛𝑢𝑎𝑙 𝑐𝑜𝑠𝑡 = ⇒ 5,84,236
𝑃𝑉𝐴𝐹9%,2 1.7591
Decision: The equivalent annual cost of machine ‘A’ is less than the equivalent annual cost of
machine ‘B’, so, machine ‘A’ should be bought.

Example 36
A project required an initial outlay of ₹20,000. It generates year ending profits of ₹12,000, ₹6,000,
₹4,000, ₹10,000 and ₹10,000 from the end of the first year to the end of fifth year. The required rate
of return is 10% and pays tax at 50% rate. The project has a life of 5 years and is depreciated on
straight line method basis. Assume that the above year ending profits are before depreciation and
tax. You are required to compute—
(i) Payback period
(ii) Average rate of return
(iii) Net present value

Solution
Profits Profits before Profits after Cash flows
before taxes i.e. profits Tax taxes i.e. profits after tax i.e.
dep. and before dep. and @ before taxes- profits after
Year tax Dep. taxes 50% taxes taxes+Dep.
1 12,000 4,000 8,000 4,000 4,000 8,000
2 6,000 4,000 2,000 1,000 1,000 5,000
3 4,000 4,000 -- -- -- 4,000
4 10,000 4,000 6,000 3,000 3,000 7,000
5 10,000 4,000 6,000 3,000 3,000 7,000
𝐼𝑛𝑖𝑡𝑖𝑎𝑙 𝐼𝑛𝑣𝑒𝑠𝑡𝑚𝑒𝑛𝑡/𝑂𝑢𝑡𝑙𝑎𝑦 − 𝑆𝑙𝑎𝑣𝑎𝑔𝑒 𝑉𝑎𝑙𝑢𝑒 20,000 − 0
𝐷𝑒𝑝𝑟𝑒𝑐𝑖𝑎𝑡𝑖𝑜𝑛 = ⇒ ⇒ 𝑅𝑠. 4,000
𝐸𝑠𝑡𝑖𝑚𝑎𝑡𝑒𝑑 𝐿𝑖𝑓𝑒 𝑜𝑓 𝑡ℎ𝑒 𝑃𝑟𝑜𝑗𝑒𝑐𝑡 5

(i) Payback period


Year Cash flows Cumulative cash flows
1 8,000 8,000
2 5,000 13,000
3 4,000 17,000
4 7,000 24,000
5 7,000 31,000
(20,000 − 17,000)
𝑃𝑎𝑦𝑏𝑎𝑐𝑘 𝑝𝑒𝑟𝑖𝑜𝑑 = 3 + = 3.428 𝑦𝑒𝑎𝑟𝑠
7,000

(ii) Average rate of return


𝐴𝑣𝑒𝑟𝑎𝑔𝑒 𝑁𝑒𝑡 𝑃𝑟𝑜𝑓𝑖𝑡𝑠 𝑜𝑓 𝑡ℎ𝑒 𝑃𝑟𝑜𝑗𝑒𝑐𝑡 2,200
𝐴𝑅𝑅 = × 100 ⇒ × 100 ⇒ 22%
𝐴𝑣𝑒𝑟𝑎𝑔𝑒 𝐼𝑛𝑣𝑒𝑠𝑡𝑚𝑒𝑛𝑡 𝑖𝑛 𝑡ℎ𝑒 𝑃𝑟𝑜𝑗𝑒𝑐𝑡 10,000

𝑇𝑜𝑡𝑎𝑙 𝐸𝑥𝑝𝑒𝑐𝑡𝑒𝑑 𝑁𝑒𝑡 𝑃𝑟𝑜𝑓𝑖𝑡𝑠 𝑜𝑓 𝑡ℎ𝑒 𝑃𝑟𝑜𝑗𝑒𝑐𝑡


𝐴𝑣𝑒𝑟𝑎𝑔𝑒 𝑁𝑒𝑡 𝑃𝑟𝑜𝑓𝑖𝑡𝑠 𝑜𝑓 𝑡ℎ𝑒 𝑃𝑟𝑜𝑗𝑒𝑐𝑡 =
𝐸𝑠𝑡𝑖𝑚𝑎𝑡𝑒𝑑 𝑈𝑠𝑒𝑓𝑢𝑙 𝐿𝑖𝑓𝑒 𝑜𝑓 𝑡ℎ𝑒 𝑃𝑟𝑜𝑗𝑒𝑐𝑡
4,000 + 1,000 + 0 + 3,000 + 3,000
⇒ ⇒ 𝑅𝑠. 2,200
5
𝐼𝑛𝑖𝑡𝑖𝑎𝑙 𝐼𝑛𝑣𝑒𝑠𝑡𝑚𝑒𝑛𝑡 + 𝑆𝑎𝑙𝑣𝑎𝑔𝑒 𝑉𝑎𝑙𝑢𝑒
𝐴𝑣𝑒𝑟𝑎𝑔𝑒 𝐼𝑛𝑣𝑒𝑠𝑡𝑚𝑒𝑛𝑡 = + 𝑊𝑜𝑟𝑘𝑖𝑛𝑔 𝐶𝑎𝑝𝑖𝑡𝑎𝑙
2
20,000 + 0
⇒ + 0 ⇒ 𝑅𝑠. 10,000
2
Chapter 5 and 6, Capital Budgeting: 61
(iii) Net present value
Year 𝑪𝑭𝑨𝑻𝒕 𝑷𝑽𝑭𝟏𝟎%,𝒏 Present value at 10%
1 8,000 0.909 7,272
2 5,000 0.826 4,130
3 4,000 0.751 3,004
4 7,000 0.683 4,781
5 7,000 0.621 4,347
Present value of cash inflows 23,534
Less: Initial cash outlay -20,000
Net Present Value (NPV) 3,534

Example 37
A company has to make a choice between two projects A and B. The initial outlay of these projects is
₹2,70,000 and ₹4,80,000 respectively. The scrap value after 5 years ₹10,000 and ₹30,000
respectively. The opportunity cost of capital of the company is 16%. The annual cash flows are as
under—
Year Project A Project B
1 -- 1,20,000
2 60,000 1,68,000
3 2,64,000 1,92,000
4 1,68,000 2,04,000
5 1,78,000 2,10,000
You are required to calculate—
(i) Payback period
(ii) Profitability index
(B. Com Honors, Delhi University, 2013)

Solution

(i) Payback period


Project A Project B
Cumulative cash Cumulative cash
Year Cash flows flows Cash flows flows
1 -- -- 1,20,000 1,20,000
2 60,000 60,000 1,68,000 2,88,000
3 2,64,000 3,24,000 1,92,000 4,80,000
4 1,68,000 4,92,000 2,04,000 6,84,000
5 1,88,000* 6,70,000 2,40,000** 8,94,000
(2,70,000 − 60,000) 2,10,000
𝑃𝑎𝑦𝑏𝑎𝑐𝑘 𝑝𝑒𝑟𝑖𝑜𝑑(𝐴) = 2 + ⇒ ⇒ 2.795 𝑦𝑒𝑎𝑟𝑠
2,64,000 2,64,000
𝑃𝑎𝑦𝑏𝑎𝑐𝑘 𝑝𝑒𝑟𝑖𝑜𝑑(𝐵) = 3 𝑦𝑒𝑎𝑟𝑠
*1,78,000+10,000 (Scrap value)
**2,10,000+30,000 (Scrap value)

(ii) Profitability index


𝑪𝑭𝑨𝑻𝒕 Present value at 16%
Year Project A Project B 𝑷𝑽𝑭𝟏𝟔%,𝒏 Project A Project B
1 -- 1,20,000 0.862 -- 1,03,440
2 60,000 1,68,000 0.743 44,580 1,24,824
3 2,64,000 1,92,000 0.641 1,69,224 1,23,072
4 1,68,000 2,04,000 0.552 92,736 1,12,608
5 1,78,000 2,10,000 0.476 84,728 99,960
5 (Scrap value) 10,000 30,000 0.476 4,760 14,280
Present value of cash inflows 3,96,028 5,78,184

Chapter 5 and 6, Capital Budgeting: 62


𝑃𝑟𝑒𝑠𝑒𝑛𝑡 𝑣𝑎𝑙𝑢𝑒 𝑜𝑓 𝑡ℎ𝑒 𝑐𝑎𝑠ℎ 𝑖𝑛𝑓𝑙𝑜𝑤𝑠 3,96,028
𝑃𝐼(𝐴) = ⇒ ⇒ 1.467
𝑃𝑟𝑒𝑠𝑒𝑛𝑡 𝑣𝑎𝑙𝑢𝑒 𝑜𝑓 𝑡ℎ𝑒 𝑐𝑎𝑠ℎ 𝑜𝑢𝑡𝑓𝑙𝑜𝑤𝑠 2,70,000
𝑃𝑟𝑒𝑠𝑒𝑛𝑡 𝑣𝑎𝑙𝑢𝑒 𝑜𝑓 𝑡ℎ𝑒 𝑐𝑎𝑠ℎ 𝑖𝑛𝑓𝑙𝑜𝑤𝑠 5,78,184
𝑃𝐼(𝐵) = ⇒ ⇒ 1.205
𝑃𝑟𝑒𝑠𝑒𝑛𝑡 𝑣𝑎𝑙𝑢𝑒 𝑜𝑓 𝑡ℎ𝑒 𝑐𝑎𝑠ℎ 𝑜𝑢𝑡𝑓𝑙𝑜𝑤𝑠 4,80,000

Example 38
A company is considering which of two mutually exclusive projects it should undertake. The finance
director thinks that project with higher net present value should be chosen as both projects have
the same financial outlay and length of life. The company anticipates a cost of capital of 10% and the
net after tax cash flows of the project are as follows—
Year 0 1 2 3 4 5
Project X (2,10,000) 40,000 80,000 90,000 75,000 25,000
Project Y (2,10,000) 2,22,000 10,000 10,000 6,000 6,000
Compute: The NPV and PI of each project and state with reasons which project you would
recommend?
(B. Com Honors, Delhi University, 2011)

Solution

(i) Net present value


𝑪𝑭𝑨𝑻𝒕 Present value at 10%
Year Project A Project B 𝑷𝑽𝑭𝟏𝟎%,𝒏 Project A Project B
1 40,000 2,22,000 0.909 36,360 2,01,798
2 80,000 10,000 0.826 66,080 8,260
3 90,000 10,000 0.751 67,590 7,510
4 75,000 6,000 0.683 51,225 4,098
5 25,000 6,000 0.621 15,525 3,726
Present value of cash inflows 2,36,780 2,25,392
Less: Initial cash outlay -2,10,000 -2,10,000
Net Present Value (NPV) 26,780 15,392

(ii) Profitability index


𝑃𝑟𝑒𝑠𝑒𝑛𝑡 𝑣𝑎𝑙𝑢𝑒 𝑜𝑓 𝑡ℎ𝑒 𝑐𝑎𝑠ℎ 𝑖𝑛𝑓𝑙𝑜𝑤𝑠 2,36,780
𝑃𝐼(𝐴) = ⇒ ⇒ 1.1275
𝑃𝑟𝑒𝑠𝑒𝑛𝑡 𝑣𝑎𝑙𝑢𝑒 𝑜𝑓 𝑡ℎ𝑒 𝑐𝑎𝑠ℎ 𝑜𝑢𝑡𝑓𝑙𝑜𝑤𝑠 2,10,000
𝑃𝑟𝑒𝑠𝑒𝑛𝑡 𝑣𝑎𝑙𝑢𝑒 𝑜𝑓 𝑡ℎ𝑒 𝑐𝑎𝑠ℎ 𝑖𝑛𝑓𝑙𝑜𝑤𝑠 2,25,392
𝑃𝐼(𝐵) = ⇒ ⇒ 1.0733
𝑃𝑟𝑒𝑠𝑒𝑛𝑡 𝑣𝑎𝑙𝑢𝑒 𝑜𝑓 𝑡ℎ𝑒 𝑐𝑎𝑠ℎ 𝑜𝑢𝑡𝑓𝑙𝑜𝑤𝑠 2,10,000

(ii) Decision/Recommendation
We recommend the project A. This has a higher Net Present Value and higher Profitability Index as
compared to B. Since the initial outlay is same, so, both the methods are giving same results. In case
initial outlay is not same then the project with higher Net Present Value should be recommended.

Example 39
A company is considering the replacement of an existing obsolete machine. It is faced with two
alternatives—
(i) Buy machine A which is similar to the existing machine.
(ii) To buy machine B which is more expensive and has higher capacity,
The cash flows after tax at the present level of operations for the two alternatives are as follows—
Year 0 1 2 3 4 5
Machine A (25,00,000) -- 5,00,000 20,00,000 14,00,000 14,00,000
Machine B (40,00,000) 10,00,000 14,00,000 16,00,000 17,00,000 15,00,000
Cost of capital is 10% calculate—
(i) Net present value
(ii) Profitability index
Chapter 5 and 6, Capital Budgeting: 63
(B. Com. Honors, Delhi University, 2006)

Solution

(i) Net present value


𝑪𝑭𝑨𝑻𝒕 Present value at 10%
Year Project A Project B 𝑷𝑽𝑭𝟏𝟎%,𝒏 Project A Project B
1 -- 10,00,000 0.909 -- 9,09,900
2 5,00,000 14,00,000 0.826 4,13,000 11,56,400
3 20,00,000 16,00,000 0.751 15,02,000 12,01,600
4 14,00,000 17,00,000 0.683 9,56,200 11,61,100
5 14,00,000 15,00,000 0.621 8,69,400 9,31,500
Present value of cash inflows 37,40,600 53,59,600
Less: Initial cash outlay -25,00,000 -40,00,000
Net Present Value (NPV) 12,40,600 13,59,600

(ii) Profitability index


𝑃𝑟𝑒𝑠𝑒𝑛𝑡 𝑣𝑎𝑙𝑢𝑒 𝑜𝑓 𝑡ℎ𝑒 𝑐𝑎𝑠ℎ 𝑖𝑛𝑓𝑙𝑜𝑤𝑠 37,40,600
𝑃𝐼(𝐴) = ⇒ ⇒ 1.496
𝑃𝑟𝑒𝑠𝑒𝑛𝑡 𝑣𝑎𝑙𝑢𝑒 𝑜𝑓 𝑡ℎ𝑒 𝑐𝑎𝑠ℎ 𝑜𝑢𝑡𝑓𝑙𝑜𝑤𝑠 25,00,000
𝑃𝑟𝑒𝑠𝑒𝑛𝑡 𝑣𝑎𝑙𝑢𝑒 𝑜𝑓 𝑡ℎ𝑒 𝑐𝑎𝑠ℎ 𝑖𝑛𝑓𝑙𝑜𝑤𝑠 53,59,600
𝑃𝐼(𝐵) = ⇒ ⇒ 1.34
𝑃𝑟𝑒𝑠𝑒𝑛𝑡 𝑣𝑎𝑙𝑢𝑒 𝑜𝑓 𝑡ℎ𝑒 𝑐𝑎𝑠ℎ 𝑜𝑢𝑡𝑓𝑙𝑜𝑤𝑠 40,00,000

(ii) Decision/Recommendation
We recommend the machine B. This has a higher Net Present Value as compared to machine A. But
if we compare the Profitability Index then machine A is better. It is because the Profitability Index is
a relative measurement. So, in case of these machines contradictory results are there. In such a case
the results given by Net Present Value method shall be used to accept/reject the proposal as it helps
in achieving the wealth maximization goal.

Example 40
A company has to make a choice between two projects namely A and B. The initial outlay of two
projects is ₹1,35,000 and ₹2,40,000 respectively for A and B. Opportunity cost of capital is 16%. The
annual cash inflows and discounting factors are as under—
Year Project A Project B 𝑷𝑽𝑭𝟏𝟔%,𝒏
1 -- 60,000 0.862
2 30,000 84,000 0.743
3 1,32,000 96,000 0.641
4 84,000 1,02,000 0.552
5 84,000 90,000 0.476
Calculate the discounted payback period.
(CA PE-II, 2002)

Solution In DISCOUNTED Payback period, we take into account the cumulative PRESENT VALUES

𝑪𝑭𝑨𝑻𝒕 Present value at 16% Cumulative present value


Project Project
Year A B 𝑷𝑽𝑭𝟏𝟔%,𝒏 Project A Project B Project A Project B
1 -- 60,000 0.862 -- 51,720 -- 51,720
2 30,000 84,000 0.743 22,290 62,412 22,290 1,14,132
3 1,32,000 96,000 0.641 84,612 61,536 1,06,902 1,75,668
4 84,000 1,02,000 0.552 46,368 56,304 1,53,270 2,31,972
5 84,000 90,000 0.476 39,984 42,840 1,93,254 2,74,812
1,35,000 − 1,06,902
𝐷𝑖𝑠𝑐𝑜𝑢𝑛𝑡𝑒𝑑 𝑝𝑎𝑦𝑏𝑎𝑐𝑘 𝑝𝑒𝑟𝑖𝑜𝑑(𝐴) = 3 + ⇒ 3.61 𝑦𝑒𝑎𝑟𝑠
46,368

Chapter 5 and 6, Capital Budgeting: 64


2,40,000 − 2,31,972
𝐷𝑖𝑠𝑐𝑜𝑢𝑛𝑡𝑒𝑑 𝑝𝑎𝑦𝑏𝑎𝑐𝑘 𝑝𝑒𝑟𝑖𝑜𝑑(𝐵) = 4 + ⇒ 4.19 𝑦𝑒𝑎𝑟𝑠
42,840

Example 41
Lotus Limited purchased a special machine one year ago at a cost of ₹12,000. At that time the
machine was estimated to have a useful life of 6 years and no scrap value. The annual cash
operating expenses are ₹20,000. A new machine is available in the market which can do the same
job and its annual operating expenses are ₹17,000. The cost of new machine is ₹21,000 and has an
estimated useful life of 5 years with zero scrap value. The old machine can be sold for ₹10,000.
Straight line method of depreciation is in use and tax rate is 40%. Cost of capital is 8%. Compute the
incremental cash inflows.

Solution
1. Initial cash outflows/outlay
Cost of new machine 21,000
Less: Scrap value of old machine -10,000
Add/Less: Tax savings/liability on loss on sales of old machine (See Note) 0
Cash outflows 11,000
Note:
Net Salvage value ₹10,000
Book value (₹12,000-₹2,000 (Dep. of 1 year)) ₹10,000
Profit/Loss on sales of machinery ₹ 0

2. Net annual cash inflows


Particulars Year (1 to 5)
Savings in operating cost 3,000
Less: Incremental depreciation (See below table 3) -2,200
Incremental profits before tax 800
Less: Tax @ 40% -320
Incremental profits after tax 480
Add: Incremental depreciation 2,200
Incremental net cash inflows 2,680

3. Incremental depreciation
𝐴𝑛𝑛𝑢𝑎𝑙 𝑑𝑒𝑝𝑟𝑒𝑐𝑖𝑎𝑡𝑖𝑜𝑛 𝑜𝑛 𝑛𝑒𝑤 𝑚𝑎𝑐ℎ𝑖𝑛𝑒
𝐶𝑜𝑠𝑡 𝑜𝑓 𝑚𝑎𝑐ℎ𝑖𝑛𝑒𝑟𝑦 − 𝑆𝑐𝑟𝑎𝑝 𝑣𝑎𝑙𝑢𝑒 21,000 − 0 ₹4,200
= ⇒=
𝐸𝑠𝑡𝑖𝑚𝑎𝑡𝑒𝑑 𝑙𝑖𝑓𝑒 5
𝐵𝑜𝑜𝑘 𝑣𝑎𝑙𝑢𝑒 12,000
𝐴𝑛𝑛𝑢𝑎𝑙 𝑑𝑒𝑝𝑟𝑒𝑐𝑖𝑎𝑡𝑖𝑜𝑛 𝑜𝑛 𝑜𝑙𝑑 𝑚𝑎𝑐ℎ𝑖𝑛𝑒 = ⇒= ₹2,000
𝐸𝑠𝑡𝑖𝑚𝑎𝑡𝑒𝑑 𝑙𝑖𝑓𝑒 6
Incremental depreciation ₹2,200
Note: Scrap value of old machine at the end of its life is given nil in the question.

Example 42
The manager of the Abdullah Limited is contemplating the purchase of new machine to replace a
machine which has been in operation in the factory from the last 5 years. Ignoring interest but
considering tax at 50% of net earnings, suggest which of the two alternatives should be preferred.
The following are the details—
Particulars Old machine New machine
Purchase price ₹40,000 ₹60,000
Estimated useful life 10 years 10 years
Machine running hours per annum 2,000 2,000
Units produced per hour 24 36
Wages per running per hour ₹3 ₹5.25
Power per annum ₹2,000 ₹4,500
Consumable stores per annum ₹6,000 ₹7,500
Chapter 5 and 6, Capital Budgeting: 65
All other charges per annum ₹8,000 ₹9,000
Material cost per unit ₹0.50 ₹0.50
Selling price per unit ₹1.25 ₹1.25

Solution
1. Calculation of Units produced
Particulars Old machine New machine
𝑅𝑢𝑛𝑛𝑖𝑛𝑔 ℎ𝑜𝑢𝑟𝑠 𝑅𝑢𝑛𝑛𝑖𝑛𝑔 ℎ𝑜𝑢𝑟𝑠
× 𝑁𝑢𝑚𝑏𝑒𝑟 𝑜𝑓 𝑢𝑛𝑖𝑡𝑠 𝑝𝑒𝑟 ℎ𝑜𝑢𝑟 × 𝑁𝑢𝑚𝑏𝑒𝑟 𝑜𝑓 𝑢𝑛𝑖𝑡𝑠 𝑝𝑒𝑟 ℎ𝑜𝑢𝑟
Units produced
⇒ 2,000 × 24 𝑢𝑛𝑖𝑡𝑠 𝑝𝑒𝑟 ℎ𝑜𝑢𝑟 ⇒ 2,000 × 36 𝑢𝑛𝑖𝑡𝑠 𝑝𝑒𝑟 ℎ𝑜𝑢𝑟
= 48,000 𝑢𝑛𝑖𝑡𝑠 = 72,000 𝑢𝑛𝑖𝑡𝑠

2. Calculation of profits
Old New
machine machine
Particulars ₹ ₹
Sales (𝑁𝑢𝑚𝑏𝑒𝑟 𝑜𝑓 𝑢𝑛𝑖𝑡𝑠 × 𝑆𝑒𝑙𝑙𝑖𝑛𝑔 𝑝𝑟𝑖𝑐𝑒 𝑝𝑒𝑟 𝑢𝑛𝑖𝑡 ) 60,000 90,000
Less: Cost of sales
Direct material (𝑁𝑢𝑚𝑏𝑒𝑟 𝑜𝑓 𝑢𝑛𝑖𝑡𝑠 × 𝑀𝑎𝑡𝑒𝑟𝑖𝑎𝑙 𝑐𝑜𝑠𝑡 𝑝𝑒𝑟 𝑢𝑛𝑖𝑡) -24,000 -36,000
Wages (𝑁𝑢𝑚𝑏𝑒𝑟 𝑜𝑓 ℎ𝑜𝑢𝑟𝑠 × 𝑊𝑎𝑔𝑒𝑠 𝑝𝑒𝑟 𝑟𝑢𝑛𝑛𝑖𝑛𝑔 ℎ𝑜𝑢𝑟) -6000 -10,500
Power per annum -2,000 -4,500
Consumable stores per annum -6,000 -7,500
Other charges per annum -8,000 -9,000
Depreciation (𝐶𝑜𝑠𝑡 𝑜𝑓 𝑡ℎ𝑒 𝑚𝑎𝑐ℎ𝑖𝑛𝑒/𝐸𝑠𝑡𝑖𝑚𝑎𝑡𝑒𝑑 𝑙𝑖𝑓𝑒 𝑜𝑓 𝑡ℎ𝑒 𝑚𝑎𝑐ℎ𝑖𝑛𝑒) -4,000 -6,000
Profits before tax 10,000 16,500
Less: Tax @ 50% -5,000 -8,250
Profits after tax 5,000 8,250
𝐼𝑛𝑐𝑟𝑒𝑚𝑒𝑛𝑡𝑎𝑙 𝑒𝑎𝑟𝑛𝑖𝑛𝑔𝑠 = ₹8,250 − ₹5,000 ⇒ ₹3,250
𝐼𝑛𝑐𝑟𝑒𝑚𝑒𝑛𝑡𝑎𝑙 𝑖𝑛𝑣𝑒𝑠𝑡𝑚𝑒𝑛𝑡 = ₹60,000 − ₹40,000 ⇒ ₹20,000
In this question discount rate is not given, so, Average Rate of Return can be used to decide which
machine to purchase.
𝐴𝑣𝑒𝑟𝑎𝑔𝑒 𝐼𝑛𝑐𝑟𝑒𝑚𝑒𝑛𝑡𝑎𝑙 𝑁𝑒𝑡 𝑃𝑟𝑜𝑓𝑖𝑡𝑠 3,250
𝐴𝑅𝑅 = × 100 ⇒ × 100 ⇒ 16.25%
𝐼𝑛𝑐𝑟𝑒𝑚𝑒𝑛𝑡𝑎𝑙 𝐼𝑛𝑣𝑒𝑠𝑡𝑚𝑒𝑛𝑡 20,000
Decision: The ARR is positive; it means that the new machine is providing additional benefits. So,
the new machine can be purchased.

Note: ARR can also be calculated as follows—


𝐴𝑣𝑒𝑟𝑎𝑔𝑒 𝐼𝑛𝑐𝑟𝑒𝑚𝑒𝑛𝑡𝑎𝑙 𝑁𝑒𝑡 𝑃𝑟𝑜𝑓𝑖𝑡𝑠 3,250
𝐴𝑅𝑅 = × 100 ⇒ × 100 ⇒ 32.50%
𝐴𝑣𝑒𝑟𝑎𝑔𝑒 𝐼𝑛𝑐𝑟𝑒𝑚𝑒𝑛𝑡𝑎𝑙 𝐼𝑛𝑣𝑒𝑠𝑡𝑚𝑒𝑛𝑡 10,000
𝐼𝑛𝑐𝑟𝑒𝑚𝑒𝑛𝑡𝑎𝑙 𝐼𝑛𝑣𝑒𝑠𝑡𝑚𝑒𝑛𝑡 20,000
𝐴𝑣𝑒𝑟𝑎𝑔𝑒 𝐼𝑛𝑐𝑟𝑒𝑚𝑒𝑛𝑡𝑎𝑙 𝐼𝑛𝑣𝑒𝑠𝑡𝑚𝑒𝑛𝑡 = ⇒ ⇒ ₹10,000
2 2

Example 43
Blue Diamond is a marketing organization that is considering launch of a product with two different
strategies: Strategy A and Strategy B. It has to choose either of the two strategies. Both the
strategies have initial outlay of ₹20 (2,000 lakhs) crores but have streams of cash inflows. Over the
period of 5 years, the cash flows of the strategies are estimated as follows—
Cash flows in lakhs of rupees
Year Strategy A Strategy B
0 -2,000 -2,000
1 500 1,000
2 850 875
3 550 500
4 650 140
5 400 200
1. Find the NPV of both the strategies at discount rates between 11% and 17% at the intervals
of 2%.

Chapter 5 and 6, Capital Budgeting: 66


2. Find IRR of both the strategies.
3. Which of the strategies is preferable if the cost of capital is 11% and 13% according to NPV
method? Justify your answer.
(B. Com. Honors, Delhi University, 2022) [Open Book Examination]

Solution

1. Find the NPV of both the strategies at discount rates between 11% and
17% at the intervals of 2%.
STRATEGY–A
Year Cash PVF @ PVF @ PVF @ PVF PV @ PV @ PV @ PV @
Flows 11% 13% 15% @ 11% 13% 15% 17%
17%
0 -2,000 1 1 1 1 -2,000.00 -2,000.00 -2,000.00 -2,000.00
1 500 0.900 0.885 0.870 0.855 450.45 442.48 434.78 427.35
2 850 0.811 0.783 0.756 0.731 689.88 665.67 642.72 620.94
3 550 0.731 0.693 0.658 0.624 402.16 381.18 361.63 343.40
4 650 0.659 0.613 0.572 0.534 428.18 398.66 371.64 346.87
5 400 0.593 0.543 0.497 0.456 237.38 217.10 198.87 182.44
NPV 208.05 105.09 9.64 -79.00

STRATEGY–B
Year Cash PVF @ PVF @ PVF @ PVF PV @ PV @ PV @ PV @
Flows 11% 13% 15% @ 11% 13% 15% 17%
17%
0 -2,000 1 1 1 1 -2000.00 -2000.00 -2000.00 -2000.00
1 1,000 0.900 0.885 0.870 0.855 900.90 884.96 869.57 854.70
2 875 0.811 0.783 0.756 0.731 710.17 685.25 661.63 639.20
3 500 0.731 0.693 0.658 0.624 365.60 346.53 328.76 312.19
4 140 0.659 0.613 0.572 0.534 92.2223 85.86 80.05 74.71
5 200 0.593 0.543 0.497 0.456 118.69 108.55 99.44 91.22
NPV 187.578 111.15 39.43 -27.98

2. Find IRR of both the strategies.


STRATEGY–A
From the perusal of the first part of the solution that NPV is positive at 15% and negative at 17%.
Now, let us apply the interpolation formula for the calculation of IRR—
𝑁𝑃𝑉𝐿𝐷𝑅 9.64
× |𝐷𝑖𝑓𝑓𝑒𝑟𝑒𝑛𝑐𝑒 𝑜𝑓 𝑅𝑎𝑡𝑒𝑠| ⇒ 𝐼𝑅𝑅 = 15 + × |2|
𝑁𝑃𝑉𝐿𝐷𝑅 − 𝑁𝑃𝑉𝐻𝐷𝑅 9.64 − (−79.00)
9.64
⇒ 15 + × 2 ⇒ 15 + 0.2175 ⇒ 15.2175% 𝑜𝑟 0.1522
88.64
Note: Using Microsoft Excel® the IRR is 15.2107%.
STRATEGY–A B
𝑁𝑃𝑉𝐿𝐷𝑅 39.43
× |𝐷𝑖𝑓𝑓𝑒𝑟𝑒𝑛𝑐𝑒 𝑜𝑓 𝑅𝑎𝑡𝑒𝑠| ⇒ 𝐼𝑅𝑅 = 15 + × |2|
𝑁𝑃𝑉𝐿𝐷𝑅 − 𝑁𝑃𝑉𝐻𝐷𝑅 39.43 − (−27.98)
39.43
⇒ 15 + × 2 ⇒ 15 + 1.1699 ⇒ 16.1699% 𝑜𝑟 0.1617
67.41
Note: Using Microsoft Excel® the IRR is 16.1549%.

3. Which of the strategies is preferable if the cost of capital is 11% and


13% according to NPV method? Justify your answer.
When cost of capital is 11%: Strategy A is better because it gives NPV of ₹208.04 whereas Strategy
B gives NPV of ₹187.58.
When cost of capital is 13%: Strategy B is better because it gives NPV of ₹105.09 whereas Strategy
A gives NPV of ₹111.15.

Chapter 5 and 6, Capital Budgeting: 67


Risk and Uncertainty in Capital
Budgeting or Capital Budgeting Under
Risk
Risk adjusted discount rate approach (RADR)
Formula for the RADR is—
𝑅𝐴𝐷𝑅(𝑘𝑎𝑑 ) = 𝑘𝑅𝑓 + 𝑃
Where,
𝑅𝐴𝐷𝑅 = Risk adjusted discount rate
𝑘𝑎𝑑 = Adjusted discount rate
𝑘𝑅𝑓 = Risk free discount rate
𝑃 = Premium for risk

Risk adjusted discount rate can also be calculated as follows—


𝑅𝐴𝐷𝑅(𝑘𝑎𝑑 ) = 𝑘𝑅𝑓 + [𝑅𝑖 × (𝑘𝑂 − 𝑘𝑅𝑓 )]
Where,
𝑅𝐴𝐷𝑅 = Risk adjusted discount rate
𝑘𝑎𝑑 = Risk adjusted discount rate
𝑘𝑅𝑓 = Risk free discount rate
𝑅𝑖 = Risk index for project
𝑘𝑂 = Overall cost of capital

The RADR calculated above now can be used to calculate the risk adjusted NPV of the project.
Formula to calculate the NPV is—
𝑛
𝐶𝐹𝑡
𝑁𝑃𝑉𝑅𝐴 = ∑ − 𝐶𝑂0
(1 + 𝑘𝑎𝑑 )𝑡
𝑡=1
Where,
𝑁𝑃𝑉𝑅𝐴 = Risk adjusted NPV
𝑡 = Time
𝑛 = Life of the project
𝐶𝐹𝑡 = Cash flows after tax in 𝑡𝑡ℎ period
𝑘𝑎𝑑 = Risk adjusted discount rate
𝐶𝑂0 = Cash outflows in 0 year

Certainty equivalent approach (CE) MORE IMPORTANT


𝑛
𝛼𝑡 𝐶𝐸𝑡
𝑁𝑃𝑉𝑅𝐴 = ∑ 𝑡 − 𝐶𝑂0
𝑡=1 (1 + 𝑘𝑅𝑓 )
Where,
𝑁𝑃𝑉𝑅𝐴 = Risk adjusted NPV
𝑡 = Time
𝑛 = Life of the project
𝛼𝑡 = Certainty equivalent factor (Certain cash flows/Uncertain cash flows)
𝐶𝐸𝑡 = Expected/uncertain cash flows
𝑘𝑅𝑓 = Risk free discount rate
𝐶𝑂0 = Cash outflows in 0 year

Chapter 5 and 6, Capital Budgeting: 68


Illustration
Example 1
Google Enterprise Limited has two proposals—Proposal A and B. These are mutually exclusive.
Proposal A requires an initial cash outlay (𝐶𝑂0 ) of ₹3,40,000 and proposal B requires an initial
outlay of ₹3,30,000 (𝐶𝑂0 ). Riskless/risk free discount rate (𝑘𝑅𝑓 ) is 8%. Use certainty equivalent
(CE) approach to determine which of the project should be accepted? The expected net cash inflows
(𝐶𝐸𝑒𝑡 ) and their certainty equivalent factors (𝛼𝑡 ) are given below—
Proposal A Proposal B
Expected cash Certainty Expected cash Certainty
inflows equivalent factors inflows equivalent factors
Year (𝐶𝐸𝑡 ) (𝛼𝑡 ) (𝐶𝐸𝑡 ) (𝛼𝑡 )
1 1,80,000 0.80 1,80,000 0.90
2 2,00,000 0.70 1,80,000 0.80
3 2,00,000 0.60 2,00,000 0.70
(B. Com. Honors, Delhi University, 2015)

Solution

Proposal A
Computation of NPV
Certainty
equivalent Adjusted cash
Cash inflows factors flows Present
Year (𝐶𝐸𝑡 ) (𝛼𝑡 ) (𝐶𝐸𝑡 × 𝛼𝑡 ) 𝑷𝑽𝑭𝟖%,𝒏 value at 8%
0 -3,40,000 1.00 -3,40,000 1.000 -3,40,000
1 1,80,000 0.80 1,44,000 0.926 1,33,344
2 2,00,000 0.70 1,40,000 0.857 1,19,980
3 2,00,000 0.60 1,20,000 0.794 95,280
NPV 8,604

Proposal B
Computation of NPV
Certainty
equivalent Adjusted cash
Cash inflows factors flows Present
Year (𝐶𝐸𝑡 ) (𝛼𝑡 ) (𝐶𝐸𝑡 × 𝛼𝑡 ) 𝑷𝑽𝑭𝟖%,𝒏 value at 8%
0 -3,30,000 1.00 -3,40,000 1.000 -3,30,000
1 1,80,000 0.90 1,6,2000 0.926 1,50,012
2 1,80,000 0.80 1,44,000 0.857 1,23,408
3 2,00,000 0.70 1,40,000 0.794 1,11,160
NPV 54,580
Decision: Since the NPV of proposal B is higher, so, it should be accepted.

Example 2
A company is considering two mutually exclusive projects. The company uses a certainty equivalent
approach. The estimated cash flows and certainty equivalents for each of the projects are as
follows—
Proposal A Proposal B
Expected cash Certainty Expected cash Certainty
inflows equivalent factors inflows equivalent factors
Year (𝐶𝐸𝑡 ) (𝛼𝑡 ) (𝐶𝐸𝑡 ) (𝛼𝑡 )
0 -30,000 1.00 -40,000 1.00
1 15,000 0.95 25,000 0.90
Chapter 5 and 6, Capital Budgeting: 69
2 18,000 0.85 20,000 0.80
3 20,000 0.70 25,000 0.70
4 20,000 0.65 18,000 0.60
Which project should be accepted, if the required rate of return of the firm is 10%?
(B. Com. Honors, Delhi University, 2007)

Solution

Project A
Computation of NPV
Certainty
equivalent Adjusted cash
Cash inflows factors flows Present
Year (𝐶𝐸𝑡 ) (𝛼𝑡 ) (𝐶𝐸𝑡 × 𝛼𝑡 ) 𝑷𝑽𝑭𝟏𝟎%,𝒏 value at 8%
0 -30,000 1.00 -30,000 1.000 -30,000
1 15,000 0.95 14,250 0.909 12,953
2 18,000 0.85 15,300 0.826 12,638
3 20,000 0.70 14,000 0.751 10,514
4 20,000 0.65 13,000 0.683 8,879
NPV 14,984

Project B
Computation of NPV
Certainty
equivalent Adjusted cash
Cash inflows factors flows Present
Year (𝐶𝐸𝒕 ) (𝛼𝑡 ) (𝐶𝐸𝑡 × 𝛼𝑡 ) 𝑷𝑽𝑭𝟏𝟎%,𝒏 value at 8%
0 -40,000 1.00 -40,000 1.000 -40,000
1 25,000 0.90 22,500 0.909 20,453
2 20,000 0.80 16,000 0.826 13,216
3 25,000 0.70 17,500 0.751 13,143
4 18,000 0.60 10,800 0.683 7,376
NPV 14,188
Decision: Since the NPV of project A is higher, so, it should be accepted.

Example 3
XYZ Limited is considering the proposal of buying of the two machines to manufacture a new
product. Each of these machines requires an investment of ₹50,000 and is expected to provide
benefits over a period of 4 years. After the expiry of useful life of machine, the sellers of both the
machines have guaranteed to buy back the machine at ₹5,000. The management of the company use
CE approach to evaluate risky investment. The company’s risk adjusted discount rate is 16% (𝑘𝑎𝑑 )
and risk-free rate is 10% (𝑘𝑅𝑓 ). The expected values of the net cash flows (𝐶𝐸𝑒𝑡 ) with their
certainty equivalents (𝛼𝑡 ) are as follows—
Proposal A Proposal B
Expected cash Certainty Expected cash Certainty
inflows equivalent factors inflows equivalent factors
Year (𝐶𝐸𝑡 ) (𝛼𝑡 ) (𝐶𝐸𝑡 ) (𝛼𝑡 )
1 30,000 0.8 18,000 0.9
2 30,000 0.7 36,000 0.8
3 30,000 0.6 24,000 0.7
4 30,000 0.5 32,000 0.4
4 5,000 1.0 5,000 1.0
Which machine, should be purchased by the company?
(B. Com. Honors, Delhi University, 2017)

Chapter 5 and 6, Capital Budgeting: 70


Solution

Machine 1
Computation of NPV
Certainty
equivalent Adjusted cash
Cash inflows factors flows Present
Year (𝐶𝐸𝑡 ) 𝛼𝑡 (𝐶𝐸𝑡 × 𝛼𝑡 ) 𝑷𝑽𝑭𝟏𝟎%,𝒏 value at 8%
0 -50,000 1.0 -50,000 1.000 -50,000
1 30,000 0.8 24,000 0.909 21,816
2 30,000 0.7 21,000 0.826 17,346
3 30,000 0.6 18,000 0.751 13,518
4 30,000 0.5 15,000 0.683 10,245
4 5,000 1.0 5,000 0.683 3,415
NPV 16,340

Machine 1
Computation of NPV
Certainty
Cash equivalent Adjusted cash
inflows factors flows Present
Year (𝐶𝐸𝑡 ) (𝛼)𝒕 (𝐶𝐸 𝑡 × 𝛼𝑡 ) 𝑷𝑽𝑭𝟏𝟎%,𝒏 value at 8%
0 -50,000 1.0 -50,000 1.000 -50,000.00
1 18,000 0.9 16,200 0.909 14,725.80
2 36,000 0.8 28,800 0.826 23,788.80
3 24,000 0.7 16,800 0.751 12,616.80
4 32,000 0.4 12,800 0.683 8,742.40
4 5,000 1.0 5,000 0.683 3,415.00
NPV 13,288.80
Decision: Since the NPV of proposal Ai is higher, so, it should be accepted.

Example 4
Sushant Singh Limited gives you the following information related to a project—
Year Cash outflows and cash inflows after taxes
0 -5,00,000
1 3,00,000
2 2,50,000
3 2,50,000
4 2,00,000
5 2,00,000
Cost of capital is 12%, interest rate on government securities is 8% and the risk index is 2.5.
Determine the net present value using risk adjusted discount rate approach and suggest whether
the investment can be made in this project or not.

Solution
Let us calculate the risk adjusted discount rate first—
𝑅𝐴𝐷𝑅(𝑘𝑎𝑑 ) = 𝑘𝑅𝑓 + [𝑅𝑖 × (𝑘𝑂 − 𝑘𝑅𝑓 )] ⇒ 0.08 + [2.5 × (0.12 − 0.08)] = 0.18
Where,
𝑅𝐴𝐷𝑅 = Risk adjusted discount rate
𝑘𝑎𝑑 = Risk adjusted discount rate
𝑘𝑅𝑓 = Risk free discount rate
𝑅𝑖 = Risk index for project
𝑘𝑂 = Overall cost of capital
Now the calculation of net present value will be as follows—

Chapter 5 and 6, Capital Budgeting: 71


Cash flows after Present value factors Present value at
taxes at RADR of 18% (𝑘𝑎𝑑 ) RADR of 18% (𝑘𝑎𝑑 )
Year (₹) (𝑃𝑉𝐹18%,𝑛 ) (₹)
1 3,00,000 0.847 2,54,100
2 2,50,000 0.718 1,79,500
3 2,50,000 0.609 1,52,250
4 2,00,000 0.516 1,03,200
5 2,00,000 0.437 87,400
Total present value 7,76,450
Less: Cash outflows -5,00,000
Risk adjusted net present value or NPV at RADR of 18% 2,76,450
Decision: Since the Risk adjusted net present value or net present value at RADR of 18% is positive,
so, the investment can be made in the project.

Chapter 5 and 6, Capital Budgeting: 72


Theoretical questions
Q. 1.: “NPV and IRR are equivalent methods for evaluating mutually
exclusive projects.” Comment.

Q. 2.: What are the similarities and dissimilarities between Net Present
Value (NPV) and Internal Rate of Return (IRR)? Which of these
methods will you prefer when they give different ranking of
investment proposals? Why?

Q. 3.: Compare the NPV method with IRR method. What are the steps
involved in the calculation of IRR in case of unequal cash flows?

Q. 4.: “While evaluating single project with conventional cash flows,


both NPV and IRR methods give identical decisions? Elucidate this
statement.
Q. 5.: Why is it possible for IRR and NPV methods to result in different
ranking of investment proposal? Which will you prefer in that
situation and why?
Answer
Though the underlying concept behind the NPV and IRR s same i.e. present value. NPV and IRR both
are discounted cash flow techniques. Also, both the methods use all the cash flows. Though
conceptually both are similar (they use the concept of time value of money, use all the cash flows,
use discount rate, etc.) but still these methods are not equivalent for evaluating mutually exclusive
projects. Also, both the methods may give identical as well as conflicting results. Let us discuss these
situations.

Similarities and dissimilarities between NPV and IRR


Concept of NPV: NPV is the difference between present value of cash inflows and present value of
cash outflows. Since the cash outflows and inflows arise at different points of time and cannot be
compared, both are reduced to their present values at the rate of return acceptable to the firm. The
rate of return is the overall cost of capital, which takes into account the shareholders expectations,
business risk and the leverage. It is assumed that cash inflows are reinvested at the same discount
rate.
𝑁𝑃𝑉 = 𝑃𝑟𝑒𝑠𝑒𝑛𝑡 𝑣𝑎𝑙𝑢𝑒 𝑜𝑓 𝐶𝑎𝑠ℎ 𝐼𝑛𝑓𝑙𝑜𝑤𝑠 − 𝑃𝑟𝑒𝑠𝑒𝑛𝑡 𝑣𝑎𝑙𝑢𝑒 𝑜𝑓 𝐶𝑎𝑠ℎ 𝑂𝑢𝑡𝑓𝑙𝑜𝑤𝑠
𝐼𝑓 𝑁𝑃𝑉 > 0: Accept the Proposal
𝐼𝑓 𝑁𝑃𝑉 = 0: Indifferent
𝐼𝑓 𝑁𝑃𝑉 < 0: Reject the Proposal
Concept of IRR: IRR is the discount rate which equates the present value of cash outflows and
present value of cash inflows. In other words, it is the rate at which NPV is zero. Unlike NPV, here
the discount rate is unknown; rather the rate is ascertained by the trial and error procedure. This
rate is then compared with a cut-off rate (generally taken to be the cost of capital) to arrive at a
decision.
𝐼𝑓 𝑟 > 𝑘: Accept the Proposal
𝐼𝑓 𝑟 = 𝑘: Indifferent
𝐼𝑓 𝑟 < 𝑘: Reject the Proposal

Similarities
Following are the points of similarities between NPV and IRR—

Chapter 5 and 6, Capital Budgeting: 73


1. Time value of money: Both methods recognize the time value of money. That is to say, a
rupee earned today is always preferred over a rupee earned tomorrow.
2. Consider total cash flows: Both methods take into account all the cash flows over the
entire proposal life to give results and thus, give a true measure of profitability.
3. Consistent with the objective of wealth maximization: The objective of financial
management is to maximize the economic wellbeing of the owners. And in the corporate
scenario, shareholders are the real owners, so, the objective is to maximize their wealth.
Both NPV and IRR result in maximization of shareholders wealth.

Dissimilarities
1. Discount rate: Under NPV method, the underlying discount rate is known and is equal to
the opportunity cost of capital as it represents the minimum rate of return required to
accept a project. However, under IRR method, the discount rate is unknown and is
generated (IRR) by trial and error procedure. This IRR equates the present value of outflows
with the present value of cash inflows. Thus, this generated IRR is the rate at which 𝑁𝑃𝑉 =
0.
2. Reinvestment assumption: Though both the methods assume that intermediate cash
inflows are reinvested at discount rates, NPV method assumes that these cash inflows are
reinvested at cost of capital while IRR method assumes the reinvestment at IRR only. The
assumption of NPV is more realistic as it indicates that all proposals earn at least the cost of
capital. However, the unrealistic assumption of IRR is relaxed under its modified version i.e.
Modified Internal Rate of Return (MIRR), wherein the cash flows are assumed to be
reinvested at specified rate of return (generally cost of capital), and hence, the results of
NPV and MIRR always match.
3. Multiple rates: Due to the mathematics inherently involved in IRR, sometimes it may result
in negative or multiple internal rates. It mainly happens in non-conventional investments.
However, this limitation is not suffered by NPV method.
4. Value-additivity principle: NPV of two different proposals can be added like NPV (𝐴 +
𝐵) = 𝑁𝑃𝑉(𝐴) + 𝑁𝑃𝑉(𝐵). This is known as the principle of value additivity. If NPV of all the
projects are known, then one can calculate the firm’s NPV by simply adding them all up.
However, unlike NPV, IRR of two proposals cannot be added i.e. IRR (𝐴 + 𝐵) ≠ 𝐼𝑅𝑅(𝐴) +
𝐼𝑅𝑅(𝐵). This can be shown with the help of an example—
A B 𝑨+𝑩
Proposal (₹) (₹) (₹)
𝐶0 (16,000) (18,000) (34,000)
𝐶1 8,000 9,000 17,000
𝐶2 7,000 7,500 14,500
𝐶3 6,000 6,500 12,500
NPV at 𝑘 = 15% 194.62 (228.98) (34.36)
IRR 16% 14% 15%
Here,
𝑁𝑃𝑉(𝐴) + 𝑁𝑃𝑉(𝐵) = 94.62 + (228.98) = (34.36) = 𝑁𝑃𝑉(𝐴 + 𝐵)
𝐼𝑅𝑅(𝐴) + 𝐼𝑅𝑅(𝐵) = 16% + 14% = 30% ≠ 𝐼𝑅𝑅(𝐴 + 𝐵)
Thus, IRR method fails to meet the principle of value-additivity.

NPV and IRR: Similar results


Both these methods give the identical results in some situations. These are—
1. When the projects have conventional cash flows: Conventional cash flows means-single
outflow in the zero year and then there is a series of cash inflows for a finite period of time.
2. When the projects are independent in nature: Independent project means that the
selection of one project does not result in the rejection of another project. All the project
fulfilling the selection criteria are selected.
The reason for having consistent results is simple and logical. Under NPV method, we accept a
proposal when we get positive NPV i.e. 𝑁𝑃𝑉 > 0. And under IRR method, we accept a proposal

Chapter 5 and 6, Capital Budgeting: 74


when internal rate of return (𝑟) is greater than the firm’s cost of capital (𝑘) i.e. 𝑟 > 𝑘. Whenever 𝑟 >
𝑘, NPV shall be positive.
Even in the case of 𝑁𝑃𝑉 = 0, where the financial manager may accept a proposal, the resulting
internal rate of return (𝑟) is equal to the firm’s cost of capital (𝑘) i.e. 𝑟 = 𝑘.
Therefore, we find that NPV and IRR give same results in terms of acceptance or rejection of
independent conventional investment proposals.

NPV and IRR: Contradictory results


The two methods give contradictory results in the case of mutually exclusive proposals. These are
the proposals where the acceptance of one proposal leads to non-acceptance of the other. There are
two types of mutually exclusive proposals-technical and financial.
1. Technical exclusiveness: It refers to the selection of the most profitable proposal out of
proposals under consideration.
2. Financial exclusiveness: It is the exclusiveness which is imposed due to financial
constraints i.e. where firm’s funds are limited. Since the funds are fixed (rationed), it is also
known as capital rationing.

Decision making in case of NPV and IRR/Ranking in cash of NPV and IRR
In case of NPV: Project is accepted if the NPV>0 and the project is rejected if the NPV<0.
In case of IRR: Project is selected if the 𝐼𝑅𝑅 > 𝑆𝑡𝑎𝑛𝑑𝑎𝑟𝑑 𝑟𝑎𝑡𝑒 𝑜𝑓 𝑟𝑒𝑡𝑢𝑟𝑛/𝑐𝑜𝑠𝑡 𝑜𝑓 𝑐𝑎𝑝𝑖𝑡𝑎𝑙/
𝑟𝑒𝑞𝑢𝑖𝑟𝑒𝑑 𝑟𝑎𝑡𝑒 𝑜𝑓 𝑟𝑒𝑡𝑢𝑟𝑛 and the project is rejected if the 𝐼𝑅𝑅 < 𝑆𝑡𝑎𝑛𝑑𝑎𝑟𝑑 𝑟𝑎𝑡𝑒 𝑜𝑓 𝑟𝑒𝑡𝑢𝑟𝑛/
𝑐𝑜𝑠𝑡 𝑜𝑓 𝑐𝑎𝑝𝑖𝑡𝑎𝑙/𝑟𝑒𝑞𝑢𝑖𝑟𝑒𝑑 𝑟𝑎𝑡𝑒 𝑜𝑓 𝑟𝑒𝑡𝑢𝑟𝑛.

A project with positive 𝑁𝑃𝑉 i.e. 𝑁𝑃𝑉 > 0 always has IRR greater than the discount rate (𝑘). So, both
the methods give the same results at the time of accepting or rejecting a project.

Example
There are two proposals A and B with a life of one year and discount rate is 10%. Calculate the NPV
and IRR.
Cash outflows Cash inflows
Particulars (₹) (₹)
Proposal A 30,000 40,000
Proposal B 20,000 25,000

Solution
10,000 5,000
𝐼𝑅𝑅𝐴 = ⇒ 0.3333 𝐼𝑅𝑅𝐵 = ⇒ 0.25
30,000 20,000

𝑁𝑃𝑉𝐴 = (𝐶𝐹𝐴𝑇 × 𝑃𝑉𝐹𝑟,𝑛 ) − 𝐶𝑂0 𝑁𝑃𝑉𝐵 = (𝐶𝐹𝐴𝑇 × 𝑃𝑉𝐹𝑟,𝑛 ) − 𝐶𝑂0


⇒ (40,000 × 𝑃𝑉𝐹10%,1 ) ⇒ (25,000 × 𝑃𝑉𝐹10%,1 )
− 30,000 − 20,000
⇒ (40,000 × 0.909) − 30,000 ⇒ (25,000 × 0.909) − 20,000
⇒ 36,360 − 30,000 = ₹6,360 ⇒ 22,725 − 20,000 = ₹2,725
In the above cash both the methods are giving same results i.e. accept the proposal A and B.
So, from the above example it is clear that NPV and IRR gives the same results in projects having
cash conventional cash flows and independent projects.

NPV and IRR: Conflicting results or conflict in ranking


In projects having cash conventional cash flows and independent projects NPV and IRR both gives
the same results but there are certain situations under which these methods may give contradictory
results in case of mutually exclusive projects. These situations are—
1. Alternative proposals have different size or scale of investment (Size-disparity problem).
2. Timings or pattern of cash flows differ among alternative proposals (Time-disparity
problem).
Chapter 5 and 6, Capital Budgeting: 75
3. Projects with unequal lives (Life-disparity problem).

1. Alternative proposals have different size or scale of investment (Size-disparity


problem)
When the size/scale of investment of two proposals differs then NPV and IRR methods may give
contradictory results. The reason is, the large size proposal has higher cash inflows and thus, the
NPV would also be high and small size proposal has lower cash flows resulting in the low NPV. It is
because the NPV is an absolute number. But in case of IRR the project is evaluated in relative terms
i.e. percentage and hence, it ignores the scale of investment. It is independent of size of the proposal.
Let us explain this with the help of an example.

Example
Cash flows of project A Cash flows of project B
Year (₹) (₹)
0 -20,000 -60,000
1 13,000 35,000
2 15,000 45,000
Cost of capital is 10%. Evaluate the projects using NPV and IRR.

Solution
Particulars Project A Project B Higher rank
NPV ₹4,207 ₹8,985 Project B
IRR 25% 20.52% Project A
As per NPV the project B is to be accepted while on the basis of IRR the project A is to be accepted.
The conflict in the results is due to the fact that there is huge difference in the size of two projects.
Project B has high investment, so, its NPV is also high. But IRR is a relative measurement, so, it does
not get affected by the size of the investment. In case of such a conflict the results given by the NPV
are used because NPV method is superior and more in line with the wealth maximization goal.
However, when the firm is suffering from liquidity crunch, it should decide according to Profitability
Index method.

2. Timings or pattern of cash flows differ among alternative proposals (Time-


disparity problem)
NPV and IRR methods may also give contradictory results when the timing or pattern of the cash
flows is different. It might be possible that in case of one proposal if larger cash flows are occurring
in the initial years and in case of another proposal larger cash flows are occurring in the later
period. Due to this difference in the cash flows and pattern both the methods may give
contradictory results. Let us explain this with the help of an example.

Example
Cash flows
X Y
Year (₹) (₹)
0 -60,000 -60,000
1 50,000 5,000
2 25,000 30,000
3 5,000 55,000
X and Y are mutually exclusive proposals. The company’s required rate of return from such projects
is 10%. Evaluate the proposals using NPV and IRR.

Solution
Particulars Project X Project Y Higher rank
NPV ₹9,855 ₹10,630 Project Y
IRR 22.80% 17.42% Project X
As per NPV the project Y is to be accepted while on the basis of IRR the project X is to be accepted.
The conflict in the results is due to the different timings or pattern of cash inflow of proposal X and
Y. Also, the conflict in ranking arises due to the reinvestment rate assumption. According to NPV
Chapter 5 and 6, Capital Budgeting: 76
method, cash inflows are getting reinvested at 10% while IRR assumes the reinvestment is made at
22.80% and 17.42% for Project X and Project Y respectively.
Since the inherent assumption of reinvestment at generated IRR is absurd and unrealistic, one
should use NPV as a decision criterion under time disparity. Results given by the NPV are also used
because NPV method is superior and more in line with the wealth maximization goal.

3. Projects with unequal lives (Life-disparity problem)


When the life of two mutually exclusive projects is different (or unequal lives), then this may result
in the conflicting ranking of the proposals given by the NPV and IRR. Let us explain this with the
help of an example.

Example
Cash flows
A B
Year (₹) (₹)
0 -1,00,000 -1,00,000
1 20,000 1,25,000
2 20,000 --
3 1,20,000 --
A and B are mutually exclusive proposals. Discount rate is 10%. Evaluate the proposals using NPV
and IRR.

Solution
Particulars Project A Project B Higher rank
NPV ₹24,820 ₹13,625 Project A
IRR 20% 25% Project B
As per NPV the project A is to be accepted while on the basis of IRR the project B is to be accepted.
The conflict in the results is due to unequal lives of proposal A and B. Whenever a firm faces life
disparity problem, it is advisable to use EANPV (Equivalent Annual Net Present Value) method to
arrive at a concrete decision. The EANPV method makes projects with unequal lives comparable
and thus, a better approach over NPV in life disparity problems.

What to do in case of conflict in ranking between NPV and IRR?


Whenever there is conflict in ranking between NPV and IRR then the results given by NPV method
should be used. It is because—
1. The NPV method is superior to IRR. This is because in the IRR there is the assumption of
reinvestment which results on the contradictory results. There is no reinvestment rate
assumption in case of NPV.
2. IRR is a relative number whereas the NPV is an absolute number. Higher NPV contributes
more towards the value of a company. It is more in support of the wealth maximization goal
of the company.

Q. 6.: What is capital budgeting decision? Explain its significance for


a firm.
Q. 7. What is capital budgeting? “Capital budgeting decisions are
irreversible”. Do you agree? Comment.

Answer

Meaning of capital budgeting decisions


Capital budgeting decisions are the decisions regarding investment in long-term assets. These are
also called capital expenditure decisions. Capital budgeting decisions include the following—
1. Acquisition of long-term assets
2. Modification/addition to the existing plant and machinery to increase the capacity
Chapter 5 and 6, Capital Budgeting: 77
3. Replacement of existing plant and machinery wither to generate more revenue or to reduce
the cost
4. Incorporation of new technology
5. Improvement of the existing manufacturing process
6. New product development
7. Research and development
8. Purchase of new business

Significance for a firm


1. Long-term implications: Capital budgeting decisions have long-term implications. These
decisions determine the benefits a company can earn over a long period of time. A quality
decision will benefit the company for a long period of time whereas a poor decision will
result in losses and can put the existence of the company in danger. So, the capital budgeting
decisions should be taken after proper estimation of the cash flows of the project.
2. Involves larger amount of funds: Capital expenditure decisions involves huge amount of
funds. Funds are scarce, so, before committing huge funds to any project there is need that
the project has been evaluated properly in term of costs and benefits. If the funds have been
committed to a poor project, then such a decision may place the company on the verge of
liquidation.
3. Change the risk complexion of the firm: When a decision regarding investment in long-
term asset is taken it changes the risk complexion of the firm. Capital budgeting decisions
are based on future cash flows and future is always uncertain. A finance manager must
incorporate the risk while evaluating capital budgeting proposals using various techniques
available viz. Risk adjusted discount rate technique, Certainty equivalent method, Decision
tree approach, etc. In this the way the quality of the investment decision may be improved.
Also, the risk incorporation helps in wise allocation of the funds.
4. Irreversible decisions: Capital budgeting decisions are irreversible in the nature. Once
taken, these decisions cannot be reversed. However, if company wants to reverse the
decision, then company has to suffer huge financial losses. This is because that after the
installation of the proposal it is very difficult to find a market for the second-hand plant and
machinery. Also, the reversal of the decision may deteriorate the image of the company in
the market. Because of the irreversible nature of the capital budgeting decisions, it is
necessary for the financial manager to make proper evaluation of projects.
5. Helps in getting competitive advantage: Capital budgeting decisions help a company to
get the competitive advantage. A sound capital expenditure decision taken at the right time
may help the company to manufacture quality products at the competitive cost. Also, this
helps the company to become market leader.
6. Difficult and complex decision: Capital decisions are difficult to take in the sense that it is
difficult to estimate future cash flows. Estimated cash flows are never correct because the
future is uncertain. So, many factors like—seasonal business fluctuations, foreign exchange
policy, taxation policy, technological changes, economic up and downs, financial
environment, political environment, etc. affect the cash flows. The proposals with longer life
are more difficult and complex to decide as compared to proposals with shorter life.

Q. 8. Explain factors which should be taken in to account while


making a capital budgeting decision.
Answer
1. Project generation
2. Preliminary screening process
3. Detailed project evaluation
a. Estimation of project’s cash outflows
b. Estimation of project’s cash inflows
c. Estimation of the minimum or required rate of return (𝑘𝑂 )
d. Application of suitable capital budgeting evaluation method/technique
(iv) Payback period method
Chapter 5 and 6, Capital Budgeting: 78
(v) Accounting rate of return method
(vi) Net present value method
(vii) Profitability index method
(viii) Internal rate of return method
(ix) Modified internal rate of return method
(x) Terminal value method
e. Selection of a project
4. Project selection and its implementation
5. Control of capital expenditure

Q. 9.: How do you deal with ‘Sunk Cost’ and ‘Allocated Overheads’ in
analyzing investment decisions?

Answer
While analyzing the investment decisions the sunk cost and allocated overheads are ignored. Let us
explain why—

Sunk cost
Sunk costs are basically past (historical) costs which were incurred in the past and have no
relevance with the particular decision making problem under consideration. These costs were
incurred ears ago before the selection of the current proposal. Neither the acceptance/rejection of
the proposal can affect the sunk cost not sunk cost can affect the current proposal. This is the reason
that these costs should be ignored. For example—fee paid to a marketing consultant to get an idea
about the sales potential of a new product or fee paid to a financial consultant to explore the
possibility of issuing equity shares. In the above case the payment has already been made, so, these
costs are not relevant for deciding whether to accept/reject the proposal.

Allocated overheads
Overheads mean the total of all the indirect expenses. In case of manufacturing concerns these
overheads are distributed or allocate on some logical or equitable basis to different department,
project, and cost centers. Allocated overheads are like sunk costs which were incurred in the past
and have no relevance with the particular decision making problem under consideration. The
reason is whether the project is undertaken or not there will be no impact on the existing amount of
the overheads. So, allocated overheads shall be ignored while making investment decisions.
However, it is important to note that if due to the acceptance of any proposal there is an increase in
the overheads then the increased amount is relevant for taking decision regarding the
acceptance/rejection of the proposal. For example—A new manager or staff is to be appointed for
the new plant then the amount to be paid to the manager or staff is relevant in decision making.

Q. 10.: Why do we focus on cash flows rather than on profits while


evaluating capital budgeting decisions?
Q. 11.: Define cash flows. How is it different from profit? Explain the
superiority of cash flows in investment decision making.

Answer

Meaning of cash flows


In capital budgeting, cost and benefits are measured in terms of cash flows. Cash flow is the
difference between amount received and paid. Cash flows are not profits. Cash flows are basically
estimates of cash receipts and cash payments cash flow is the movement of cash of a proposal. Cash
flows are divided in two part i.e. cash outflows and cash inflows. Cost of the proposal is measured in
terms of cash outflows and benefits are measured in terms of cash inflows. While estimating cash

Chapter 5 and 6, Capital Budgeting: 79


flows non-cash expenses are excluded because there is not flow of the cash. Cash flow can be
calculated as follows—
𝐶𝑎𝑠ℎ 𝑓𝑙𝑜𝑤 = 𝑃𝑟𝑜𝑓𝑖𝑡 𝑎𝑓𝑡𝑒𝑟 𝑡𝑎𝑥 + 𝑁𝑜𝑛 𝑐𝑎𝑠ℎ 𝑒𝑥𝑝𝑒𝑛𝑠𝑒𝑠

Why do we focus on cash flows rather than on profits while evaluating


capital budgeting decisions? or Superiority of cash flows in investment
decision making
Though accounting profit is an indicator of the economic efficiency of any company but while
measuring costs and benefits of an investment proposal, cash flows are preferred over accounting
profits (cash flows are superior in investment decision making) due to the following reasons—
1. Accounting profits are calculated after incorporating, so, many non-cash items like
depreciation, goodwill written off, accumulated losses written off, etc. No cash flows
were involved. Because of this accounting profit do not measure the true benefits of an
investment proposal.
2. While calculating accounting profits various accounting policies are followed. These
policies are in respect of deprecation, valuation of stock, treatment of various expenses or
incomes, etc. Sometimes these policies are subjective also. Because of this accounting profits
may differ in respect of these items. But cash flows are certain and no subjective policies are
there.
3. While calculating accounting profit the concept of time value of money is not taken in
to account. Accounting profits of different time are not adjusted for time and hence, cannot
be compared. Whereas cash flows of different periods can be adjusted for time value of
money and can be compared.
4. While calculating accounting profits accrual principle is followed. Accrued items may
or may not happen in future. But cash flows are certain and estimated on receipt and
payment basis. So, cash flow principle is a better choice for the long-term investment
decision.
5. Accounting profit ignores risk: Accounting profit is a performance measure and favors a
project which is likely to provide maximum profit irrespective of risk involved. However,
cash flow approach determines cash flows by incorporating the risk factor in the calculation.
In capital budgeting, certain techniques like—Certainty equivalent approach, Decision tree
approach, etc. are used to incorporate the risk factor in cash flows.

Q. 12.: Certainty equivalent approach is theoretically superior to the


risk adjusted discount rate approach. Do you agree?
Answer
Yes we agree that the certainty equivalent approach is superior alternative to the RADR approach to
combine risk into the capital budgeting decision. Under the CE method, the risk of a project is
combined by adjusting the future cash inflows instead of adjusting the rate of return used as a
discount rate. The future cash inflows are risky and uncertain. These are converted into certain cash
inflows. Thus, if future uncertain cash inflows can be converted into certain cash inflows, there
would be possibility that the risk element can be eliminated.
Why Certainty equivalent approach is theoretically superior to the risk adjusted discount
rate approach?: The certainty equivalent approach recognizes the risk explicitly and combines it
by lowering the uncertain cash flows to certain cash flows. This approach is theoretically and
conceptually superior to the risk adjusted discount rate approach because it does not assume that
risk increases over a period of time at a constant rate. Thought the task of estimating the certainty
equivalent factors is tedious but still is superior approach to the risk adjusted discount rate
approach.

Chapter 5 and 6, Capital Budgeting: 80


Q. 13.: Compare the Net Present Value (NPV) with Profitability Index
(PI) method of evaluating a capital budgeting proposal. Which one is
better and why?

Answer
Following differences are there between Net Present Value method and Profitability Index method—
Basis of
Net Present Value Method (NPV) Profitability Index Method (PI)
difference
Net present value is the difference
Profitability index is the ratio of present
between the present value of cash
1. Meaning value of cash inflows to the present
inflows and present value of cash
value of cash outflows.
outflows.
2. Absolute or Net present value is an absolute amount Profitability index is a pure number. It is
relative in terms of rupees. a relative measurement.
𝐶𝐹𝐴𝑇𝑡
𝑛
𝐶𝐹𝐴𝑇𝑡
𝑛
𝐶𝑂𝑡 𝑃𝑉 𝑜𝑓 𝐶𝐼 ∑𝑛𝑡=1
(1 + 𝑘𝑂 )𝑡
3. Formula 𝑁𝑃𝑉 = ∑ −∑ 𝑃𝐼 = 𝑜𝑟
(1 + 𝑘𝑂 )𝑡 (1 + 𝑘𝑂 )𝑡 𝑃𝑉 𝑜𝑓 𝐶𝑂 ∑𝑛 𝐶𝑂𝑡
𝑡=1 𝑡=0 𝑡−0 (1 + 𝑘 )𝑡
𝑂
4. Decision If net present value is positive then the If the profitability index is more than
making project is accepted. one then the project is accepted.
Net present value can be used in any
Profitability index is particularly useful
5. Use case be it accept/reject, mutually
in case of capital rationing.
exclusive or capital rationing.
Though the basis for the calculation of
Net present value is superior to all other
profitability index is the present value of
6. Superiority methods because it helps in maximizing
cash inflows and cash outflows but it’s
the wealth of the shareholders.
not superior.
7.
Achievement Profitability index may or may not be
Net present value is helpful in achieving
of wealth helpful in achieving the goal of wealth
the goal of wealth maximization.
maximization maximization.
goal
Which one is better/superior and why? or Which is superior ranking criteria, PI or NPV?
Net present value is better as compared to the profitability index because net present value is a
universally accepted method for the evaluation of the capital budgeting proposals. Further, the net
present value method gives an amount in terms of rupees. If this amount is positive then the project
is accepted otherwise not. Whenever it comes to maximize the wealth of the shareholders then net
present value method is considered better because the net present value is the wealth in itself.
Profitability index may or may not help in achieving this objective. On the basis of the comparisons
made above and the discussion we can conclude that net present value method is better as
compared to the profitability index.

Q. 14.: Differentiate between risk adjusted discount rate approach and


certainty equivalent methods of incorporating risk in capital
budgeting.
Answer
Following differences are there between risk adjusted discount rate approach and certainty
equivalent methods of incorporating risk in capital budgeting—
Basis of Risk adjusted discount rate approach
Certainty equivalent approach (CE)
difference (RADR)
1. CE approach incorporates the risk by
RADR incorporates the risk by
Incorporation converting the uncertain future cash
increasing the discount rate.
of risk inflows in to certain future cash inflows.

Chapter 5 and 6, Capital Budgeting: 81


There in implied assumption in the Under CE approach it is assumed that
2. Assumption RADR that risk of the project increase at the degree of risk is different for
a constant rate over a period of time. different years.
3.
According to CE approach there is
Consideration RADR combines the risk free rate, risk
difference between the risk and risk free
of risk and involved and the risk premium.
rate.
risk free rate
4. Discount
Under RADR the discount rate taken is Under the CE approach the discount rate
rate
risk adjusted. taken is the risk free discount rate.
considered
5. Process See figure below See figure below

Future Cash Flows


(Risky)

Certainty Equivalent Risk-Adjusted Discount Rate


(Adjustment for Risk) (Adjustment for Risk and Time)

Time Value of Money


(Adjustment for Time)

Present Value

RADR and CE Approach: A Comparison

Chapter 5 and 6, Capital Budgeting: 82


Annexure-1: Table A-3, Present Value Factor (PVFr, n)
n 1% 2% 3% 4% 5% 6% 7% 8% 9% 10%
1 0.990 0.980 0.971 0.962 0.952 0.943 0.935 0.926 0.917 0.909
2 0.980 0.961 0.943 0.925 0.907 0.890 0.873 0.857 0.842 0.826
3 0.971 0.942 0.915 0.889 0.864 0.840 0.816 0.794 0.772 0.751
4 0.961 0.924 0.888 0.855 0.823 0.792 0.763 0.735 0.708 0.683
5 0.951 0.906 0.863 0.822 0.784 0.747 0.713 0.681 0.650 0.621
6 0.942 0.888 0.837 0.790 0.746 0.705 0.666 0.630 0.596 0.564
7 0.933 0.871 0.813 0.760 0.711 0.665 0.623 0.583 0.547 0.513
8 0.923 0.853 0.789 0.731 0.677 0.627 0.582 0.540 0.502 0.467
9 0.914 0.837 0.766 0.703 0.645 0.592 0.544 0.500 0.460 0.424
10 0.905 0.820 0.744 0.676 0.614 0.558 0.508 0.463 0.422 0.386
11 0.896 0.804 0.722 0.650 0.585 0.527 0.475 0.429 0.388 0.350
12 0.887 0.788 0.701 0.625 0.557 0.497 0.444 0.397 0.356 0.319
13 0.879 0.773 0.681 0.601 0.530 0.469 0.415 0.368 0.326 0.290
14 0.870 0.758 0.661 0.577 0.505 0.442 0.388 0.340 0.299 0.263
15 0.861 0.743 0.642 0.555 0.481 0.417 0.362 0.315 0.275 0.239
16 0.853 0.728 0.623 0.534 0.458 0.394 0.339 0.292 0.252 0.218
17 0.844 0.714 0.605 0.513 0.436 0.371 0.317 0.270 0.231 0.198
18 0.836 0.700 0.587 0.494 0.416 0.350 0.296 0.250 0.212 0.180
19 0.828 0.686 0.570 0.475 0.396 0.331 0.277 0.232 0.194 0.164
20 0.820 0.673 0.554 0.456 0.377 0.312 0.258 0.215 0.178 0.149
21 0.811 0.660 0.538 0.439 0.359 0.294 0.242 0.199 0.164 0.135
22 0.803 0.647 0.522 0.422 0.342 0.278 0.226 0.184 0.150 0.123
23 0.795 0.634 0.507 0.406 0.326 0.262 0.211 0.170 0.138 0.112
24 0.788 0.622 0.492 0.390 0.310 0.247 0.197 0.158 0.126 0.102
25 0.780 0.610 0.478 0.375 0.295 0.233 0.184 0.146 0.116 0.092
26 0.772 0.598 0.464 0.361 0.281 0.220 0.172 0.135 0.106 0.084
27 0.764 0.586 0.450 0.347 0.268 0.207 0.161 0.125 0.098 0.076
28 0.757 0.574 0.437 0.333 0.255 0.196 0.150 0.116 0.090 0.069
29 0.749 0.563 0.424 0.321 0.243 0.185 0.141 0.107 0.082 0.063
30 0.742 0.552 0.412 0.308 0.231 0.174 0.131 0.099 0.075 0.057
1
𝑃𝑉𝐹𝑟,𝑛 =
(1 + 𝑟)𝑛

Chapter 5 and 6, Capital Budgeting: 83


Annexure-1: Table A-3, Present Value Factor (PVFr, n)
N 11% 12% 13% 14% 15% 16% 17% 18% 19% 20%
1 0.901 0.893 0.885 0.877 0.870 0.862 0.855 0.847 0.840 0.833
2 0.812 0.797 0.783 0.769 0.756 0.743 0.731 0.718 0.706 0.694
3 0.731 0.712 0.693 0.675 0.658 0.641 0.624 0.609 0.593 0.579
4 0.659 0.636 0.613 0.592 0.572 0.552 0.534 0.516 0.499 0.482
5 0.593 0.567 0.543 0.519 0.497 0.476 0.456 0.437 0.419 0.402
6 0.535 0.507 0.480 0.456 0.432 0.410 0.390 0.370 0.352 0.335
7 0.482 0.452 0.425 0.400 0.376 0.354 0.333 0.314 0.296 0.279
8 0.434 0.404 0.376 0.351 0.327 0.305 0.285 0.266 0.249 0.233
9 0.391 0.361 0.333 0.308 0.284 0.263 0.243 0.225 0.209 0.194
10 0.352 0.322 0.295 0.270 0.247 0.227 0.208 0.191 0.176 0.162
11 0.317 0.287 0.261 0.237 0.215 0.195 0.178 0.162 0.148 0.135
12 0.286 0.257 0.231 0.208 0.187 0.168 0.152 0.137 0.124 0.112
13 0.258 0.229 0.204 0.182 0.163 0.145 0.130 0.116 0.104 0.093
14 0.232 0.205 0.181 0.160 0.141 0.125 0.111 0.099 0.088 0.078
15 0.209 0.183 0.160 0.140 0.123 0.108 0.095 0.084 0.074 0.065
16 0.188 0.163 0.141 0.123 0.107 0.093 0.081 0.071 0.062 0.054
17 0.170 0.146 0.125 0.108 0.093 0.080 0.069 0.060 0.052 0.045
18 0.153 0.130 0.111 0.095 0.081 0.069 0.059 0.051 0.044 0.038
19 0.138 0.116 0.098 0.083 0.070 0.060 0.051 0.043 0.037 0.031
20 0.124 0.104 0.087 0.073 0.061 0.051 0.043 0.037 0.031 0.026
21 0.112 0.093 0.077 0.064 0.053 0.044 0.037 0.031 0.026 0.022
22 0.101 0.083 0.068 0.056 0.046 0.038 0.032 0.026 0.022 0.018
23 0.091 0.074 0.060 0.049 0.040 0.033 0.027 0.022 0.018 0.015
24 0.082 0.066 0.053 0.043 0.035 0.028 0.023 0.019 0.015 0.013
25 0.074 0.059 0.047 0.038 0.030 0.024 0.020 0.016 0.013 0.010
26 0.066 0.053 0.042 0.033 0.026 0.021 0.017 0.014 0.011 0.009
27 0.060 0.047 0.037 0.029 0.023 0.018 0.014 0.011 0.009 0.007
28 0.054 0.042 0.033 0.026 0.020 0.016 0.012 0.010 0.008 0.006
29 0.048 0.037 0.029 0.022 0.017 0.014 0.011 0.008 0.006 0.005
30 0.044 0.033 0.026 0.020 0.015 0.012 0.009 0.007 0.005 0.004
1
𝑃𝑉𝐹𝑟,𝑛 =
(1 + 𝑟)𝑛

Chapter 5 and 6, Capital Budgeting: 84


Annexure-1: Table A-3, Present Value Factor (PVFr, n)
N 21% 22% 23% 24% 25% 26% 27% 28% 29% 30%
0.826 0.820 0.813 0.806 0.800 0.794 0.787 0.781 0.775 0.769
1 0.683 0.672 0.661 0.650 0.640 0.630 0.620 0.610 0.601 0.592
2 0.564 0.551 0.537 0.524 0.512 0.500 0.488 0.477 0.466 0.455
3 0.467 0.451 0.437 0.423 0.410 0.397 0.384 0.373 0.361 0.350
4 0.386 0.370 0.355 0.341 0.328 0.315 0.303 0.291 0.280 0.269
5 0.319 0.303 0.289 0.275 0.262 0.250 0.238 0.227 0.217 0.207
6 0.263 0.249 0.235 0.222 0.210 0.198 0.188 0.178 0.168 0.159
7 0.218 0.204 0.191 0.179 0.168 0.157 0.148 0.139 0.130 0.123
8 0.180 0.167 0.155 0.144 0.134 0.125 0.116 0.108 0.101 0.094
9 0.149 0.137 0.126 0.116 0.107 0.099 0.092 0.085 0.078 0.073
10 0.123 0.112 0.103 0.094 0.086 0.079 0.072 0.066 0.061 0.056
11 0.102 0.092 0.083 0.076 0.069 0.062 0.057 0.052 0.047 0.043
12 0.084 0.075 0.068 0.061 0.055 0.050 0.045 0.040 0.037 0.033
13 0.069 0.062 0.055 0.049 0.044 0.039 0.035 0.032 0.028 0.025
14 0.057 0.051 0.045 0.040 0.035 0.031 0.028 0.025 0.022 0.020
15 0.047 0.042 0.036 0.032 0.028 0.025 0.022 0.019 0.017 0.015
16 0.039 0.034 0.030 0.026 0.023 0.020 0.017 0.015 0.013 0.012
17 0.032 0.028 0.024 0.021 0.018 0.016 0.014 0.012 0.010 0.009
18 0.027 0.023 0.020 0.017 0.014 0.012 0.011 0.009 0.008 0.007
19 0.022 0.019 0.016 0.014 0.012 0.010 0.008 0.007 0.006 0.005
20 0.018 0.015 0.013 0.011 0.009 0.008 0.007 0.006 0.005 0.004
21 0.015 0.013 0.011 0.009 0.007 0.006 0.005 0.004 0.004 0.003
22 0.012 0.010 0.009 0.007 0.006 0.005 0.004 0.003 0.003 0.002
23 0.010 0.008 0.007 0.006 0.005 0.004 0.003 0.003 0.002 0.002
24 0.009 0.007 0.006 0.005 0.004 0.003 0.003 0.002 0.002 0.001
25 0.007 0.006 0.005 0.004 0.003 0.002 0.002 0.002 0.001 0.001
26 0.006 0.005 0.004 0.003 0.002 0.002 0.002 0.001 0.001 0.001
27 0.005 0.004 0.003 0.002 0.002 0.002 0.001 0.001 0.001 0.001
28 0.004 0.003 0.002 0.002 0.002 0.001 0.001 0.001 0.001 0.000
29 0.003 0.003 0.002 0.002 0.001 0.001 0.001 0.001 0.000 0.000
30 0.003 0.003 0.002 0.002 0.001 0.001 0.001 0.001 0.000 0.000
1
𝑃𝑉𝐹𝑟,𝑛 =
(1 + 𝑟)𝑛

Chapter 5 and 6, Capital Budgeting: 85


Annexure-1: Table A-3, Present Value Factor (PVFr, n)
N 31% 32% 33% 34% 35% 36% 37% 38% 39% 40%
1 0.763 0.758 0.752 0.746 0.741 0.735 0.730 0.725 0.719 0.714
2 0.583 0.574 0.565 0.557 0.549 0.541 0.533 0.525 0.518 0.510
3 0.445 0.435 0.425 0.416 0.406 0.398 0.389 0.381 0.372 0.364
4 0.340 0.329 0.320 0.310 0.301 0.292 0.284 0.276 0.268 0.260
5 0.259 0.250 0.240 0.231 0.223 0.215 0.207 0.200 0.193 0.186
6 0.198 0.189 0.181 0.173 0.165 0.158 0.151 0.145 0.139 0.133
7 0.151 0.143 0.136 0.129 0.122 0.116 0.110 0.105 0.100 0.095
8 0.115 0.108 0.102 0.096 0.091 0.085 0.081 0.076 0.072 0.068
9 0.088 0.082 0.077 0.072 0.067 0.063 0.059 0.055 0.052 0.048
10 0.067 0.062 0.058 0.054 0.050 0.046 0.043 0.040 0.037 0.035
11 0.051 0.047 0.043 0.040 0.037 0.034 0.031 0.029 0.027 0.025
12 0.039 0.036 0.033 0.030 0.027 0.025 0.023 0.021 0.019 0.018
13 0.030 0.027 0.025 0.022 0.020 0.018 0.017 0.015 0.014 0.013
14 0.023 0.021 0.018 0.017 0.015 0.014 0.012 0.011 0.010 0.009
15 0.017 0.016 0.014 0.012 0.011 0.010 0.009 0.008 0.007 0.006
16 0.013 0.012 0.010 0.009 0.008 0.007 0.006 0.006 0.005 0.005
17 0.010 0.009 0.008 0.007 0.006 0.005 0.005 0.004 0.004 0.003
18 0.008 0.007 0.006 0.005 0.005 0.004 0.003 0.003 0.003 0.002
19 0.006 0.005 0.004 0.004 0.003 0.003 0.003 0.002 0.002 0.002
20 0.005 0.004 0.003 0.003 0.002 0.002 0.002 0.002 0.001 0.001
21 0.003 0.003 0.003 0.002 0.002 0.002 0.001 0.001 0.001 0.001
22 0.003 0.002 0.002 0.002 0.001 0.001 0.001 0.001 0.001 0.001
23 0.002 0.002 0.001 0.001 0.001 0.001 0.001 0.001 0.001 0.000
24 0.002 0.001 0.001 0.001 0.001 0.001 0.001 0.000 0.000 0.000
25 0.001 0.001 0.001 0.001 0.001 0.000 0.000 0.000 0.000 0.000
26 0.001 0.001 0.001 0.000 0.000 0.000 0.000 0.000 0.000 0.000
27 0.001 0.001 0.000 0.000 0.000 0.000 0.000 0.000 0.000 0.000
28 0.001 0.000 0.000 0.000 0.000 0.000 0.000 0.000 0.000 0.000
29 0.000 0.000 0.000 0.000 0.000 0.000 0.000 0.000 0.000 0.000
30 0.000 0.000 0.000 0.000 0.000 0.000 0.000 0.000 0.000 0.000
1
𝑃𝑉𝐹𝑟,𝑛 =
(1 + 𝑟)𝑛

Chapter 5 and 6, Capital Budgeting: 86


Annexure 2: Table A-4, Present Value Annuity Factors (PVAFr,
n)
N 1% 2% 3% 4% 5% 6% 7% 8% 9% 10%
1 0.990 0.980 0.971 0.962 0.952 0.943 0.935 0.926 0.917 0.909
2 1.970 1.942 1.913 1.886 1.859 1.833 1.808 1.783 1.759 1.736
3 2.941 2.884 2.829 2.775 2.723 2.673 2.624 2.577 2.531 2.487
4 3.902 3.808 3.717 3.630 3.546 3.465 3.387 3.312 3.240 3.170
5 4.853 4.713 4.580 4.452 4.329 4.212 4.100 3.993 3.890 3.791
6 5.795 5.601 5.417 5.242 5.076 4.917 4.767 4.623 4.486 4.355
7 6.728 6.472 6.230 6.002 5.786 5.582 5.389 5.206 5.033 4.868
8 7.652 7.325 7.020 6.733 6.463 6.210 5.971 5.747 5.535 5.335
9 8.566 8.162 7.786 7.435 7.108 6.802 6.515 6.247 5.995 5.759
10 9.471 8.983 8.530 8.111 7.722 7.360 7.024 6.710 6.418 6.145
11 10.368 9.787 9.253 8.760 8.306 7.887 7.499 7.139 6.805 6.495
12 11.255 10.575 9.954 9.385 8.863 8.384 7.943 7.536 7.161 6.814
13 12.134 11.348 10.635 9.986 9.394 8.853 8.358 7.904 7.487 7.103
14 13.004 12.106 11.296 10.563 9.899 9.295 8.745 8.244 7.786 7.367
15 13.865 12.849 11.938 11.118 10.380 9.712 9.108 8.559 8.061 7.606
16 14.718 13.578 12.561 11.652 10.838 10.106 9.447 8.851 8.313 7.824
17 15.562 14.292 13.166 12.166 11.274 10.477 9.763 9.122 8.544 8.022
18 16.398 14.992 13.754 12.659 11.690 10.828 10.059 9.372 8.756 8.201
19 17.226 15.678 14.324 13.134 12.085 11.158 10.336 9.604 8.950 8.365
20 18.046 16.351 14.877 13.590 12.462 11.470 10.594 9.818 9.129 8.514
21 18.857 17.011 15.415 14.029 12.821 11.764 10.836 10.017 9.292 8.649
22 19.660 17.658 15.937 14.451 13.163 12.042 11.061 10.201 9.442 8.772
23 20.456 18.292 16.444 14.857 13.489 12.303 11.272 10.371 9.580 8.883
24 21.243 18.914 16.936 15.247 13.799 12.550 11.469 10.529 9.707 8.985
25 22.023 19.523 17.413 15.622 14.094 12.783 11.654 10.675 9.823 9.077
26 22.795 20.121 17.877 15.983 14.375 13.003 11.826 10.810 9.929 9.161
27 23.560 20.707 18.327 16.330 14.643 13.211 11.987 10.935 10.027 9.237
28 24.316 21.281 18.764 16.663 14.898 13.406 12.137 11.051 10.116 9.307
29 25.066 21.844 19.188 16.984 15.141 13.591 12.278 11.158 10.198 9.370
30 25.808 22.396 19.600 17.292 15.372 13.765 12.409 11.258 10.274 9.427
1
1−
(1 + 𝑟)𝑛
𝑃𝑉𝐴𝐹𝑟,𝑛 = [ ]
𝑟

Chapter 5 and 6, Capital Budgeting: 87


Annexure 2: Table A-4, Present Value Annuity Factors (PVAFr,
n)
N 11% 12% 13% 14% 15% 16% 17% 18% 19% 20%
1 0.901 0.893 0.885 0.877 0.870 0.877 0.855 0.847 0.840 0.833
2 1.713 1.690 1.668 1.647 1.626 1.647 1.585 1.566 1.547 1.528
3 2.444 2.402 2.361 2.322 2.283 2.322 2.210 2.174 2.140 2.106
4 3.102 3.037 2.974 2.914 2.855 2.914 2.743 2.690 2.639 2.589
5 3.696 3.605 3.517 3.433 3.352 3.433 3.199 3.127 3.058 2.991
6 4.231 4.111 3.998 3.889 3.784 3.889 3.589 3.498 3.410 3.326
7 4.712 4.564 4.423 4.288 4.160 4.288 3.922 3.812 3.706 3.605
8 5.146 4.968 4.799 4.639 4.487 4.639 4.207 4.078 3.954 3.837
9 5.537 5.328 5.132 4.946 4.772 4.946 4.451 4.303 4.163 4.031
10 5.889 5.650 5.426 5.216 5.019 5.216 4.659 4.494 4.339 4.192
11 6.207 5.938 5.687 5.453 5.234 5.453 4.836 4.656 4.486 4.327
12 6.492 6.194 5.918 5.660 5.421 5.660 4.988 4.793 4.611 4.439
13 6.750 6.424 6.122 5.842 5.583 5.842 5.118 4.910 4.715 4.533
14 6.982 6.628 6.302 6.002 5.724 6.002 5.229 5.008 4.802 4.611
15 7.191 6.811 6.462 6.142 5.847 6.142 5.324 5.092 4.876 4.675
16 7.379 6.974 6.604 6.265 5.954 6.265 5.405 5.162 4.938 4.730
17 7.549 7.120 6.729 6.373 6.047 6.373 5.475 5.222 4.990 4.775
18 7.702 7.250 6.840 6.467 6.128 6.467 5.534 5.273 5.033 4.812
19 7.839 7.366 6.938 6.550 6.198 6.550 5.584 5.316 5.070 4.843
20 7.963 7.469 7.025 6.623 6.259 6.623 5.628 5.353 5.101 4.870
21 8.075 7.562 7.102 6.687 6.312 6.687 5.665 5.384 5.127 4.891
22 8.176 7.645 7.170 6.743 6.359 6.743 5.696 5.410 5.149 4.909
23 8.266 7.718 7.230 6.792 6.399 6.792 5.723 5.432 5.167 4.925
24 8.348 7.784 7.283 6.835 6.434 6.835 5.746 5.451 5.182 4.937
25 8.422 7.843 7.330 6.873 6.464 6.873 5.766 5.467 5.195 4.948
26 8.488 7.896 7.372 6.906 6.491 6.906 5.783 5.480 5.206 4.956
27 8.548 7.943 7.409 6.935 6.514 6.935 5.798 5.492 5.215 4.964
28 8.602 7.984 7.441 6.961 6.534 6.961 5.810 5.502 5.223 4.970
29 8.650 8.022 7.470 6.983 6.551 6.983 5.820 5.510 5.229 4.975
30 8.694 8.055 7.496 7.003 6.566 7.003 5.829 5.517 5.235 4.979
1
1−
(1 + 𝑟)𝑛
𝑃𝑉𝐴𝐹𝑟,𝑛 = [ ]
𝑟

Chapter 5 and 6, Capital Budgeting: 88


Annexure 2: Table A-4, Present Value Annuity Factors (PVAFr,
n)
N 21% 22% 23% 24% 25% 26% 27% 28% 29% 30%
1 0.826 0.820 0.813 0.806 0.800 0.794 0.787 0.781 0.775 0.769
2 1.509 1.492 1.474 1.457 1.440 1.424 1.407 1.392 1.376 1.361
3 2.074 2.042 2.011 1.981 1.952 1.923 1.896 1.868 1.842 1.816
4 2.540 2.494 2.448 2.404 2.362 2.320 2.280 2.241 2.203 2.166
5 2.926 2.864 2.803 2.745 2.689 2.635 2.583 2.532 2.483 2.436
6 3.245 3.167 3.092 3.020 2.951 2.885 2.821 2.759 2.700 2.643
7 3.508 3.416 3.327 3.242 3.161 3.083 3.009 2.937 2.868 2.802
8 3.726 3.619 3.518 3.421 3.329 3.241 3.156 3.076 2.999 2.925
9 3.905 3.786 3.673 3.566 3.463 3.366 3.273 3.184 3.100 3.019
10 4.054 3.923 3.799 3.682 3.571 3.465 3.364 3.269 3.178 3.092
11 4.177 4.035 3.902 3.776 3.656 3.543 3.437 3.335 3.239 3.147
12 4.278 4.127 3.985 3.851 3.725 3.606 3.493 3.387 3.286 3.190
13 4.362 4.203 4.053 3.912 3.780 3.656 3.538 3.427 3.322 3.223
14 4.432 4.265 4.108 3.962 3.824 3.695 3.573 3.459 3.351 3.249
15 4.489 4.315 4.153 4.001 3.859 3.726 3.601 3.483 3.373 3.268
16 4.536 4.357 4.189 4.033 3.887 3.751 3.623 3.503 3.390 3.283
17 4.576 4.391 4.219 4.059 3.910 3.771 3.640 3.518 3.403 3.295
18 4.608 4.419 4.243 4.080 3.928 3.786 3.654 3.529 3.413 3.304
19 4.635 4.442 4.263 4.097 3.942 3.799 3.664 3.539 3.421 3.311
20 4.657 4.460 4.279 4.110 3.954 3.808 3.673 3.546 3.427 3.316
21 4.675 4.476 4.292 4.121 3.963 3.816 3.679 3.551 3.432 3.320
22 4.690 4.488 4.302 4.130 3.970 3.822 3.684 3.556 3.436 3.323
23 4.703 4.499 4.311 4.137 3.976 3.827 3.689 3.559 3.438 3.325
24 4.713 4.507 4.318 4.143 3.981 3.831 3.692 3.562 3.441 3.327
25 4.721 4.514 4.323 4.147 3.985 3.834 3.694 3.564 3.442 3.329
26 4.728 4.520 4.328 4.151 3.988 3.837 3.696 3.566 3.444 3.330
27 4.734 4.524 4.332 4.154 3.990 3.839 3.698 3.567 3.445 3.331
28 4.739 4.528 4.335 4.157 3.992 3.840 3.699 3.568 3.446 3.331
29 4.743 4.531 4.337 4.159 3.994 3.841 3.700 3.569 3.446 3.332
30 4.746 4.534 4.339 4.160 3.995 3.842 3.701 3.569 3.447 3.332
1
1−
(1 + 𝑟)𝑛
𝑃𝑉𝐴𝐹𝑟,𝑛 = [ ]
𝑟

Chapter 5 and 6, Capital Budgeting: 89


Annexure 2: Table A-4, Present Value Annuity Factors (PVAFr,
n)
N 31% 32% 33% 34% 35% 36% 37% 38% 39% 40%
1 0.763 0.758 0.752 0.746 0.741 0.735 0.730 0.725 0.719 0.714
2 1.346 1.331 1.317 1.303 1.289 1.276 1.263 1.250 1.237 1.224
3 1.791 1.766 1.742 1.719 1.696 1.673 1.652 1.630 1.609 1.589
4 2.130 2.096 2.062 2.029 1.997 1.966 1.935 1.906 1.877 1.849
5 2.390 2.345 2.302 2.260 2.220 2.181 2.143 2.106 2.070 2.035
6 2.588 2.534 2.483 2.433 2.385 2.339 2.294 2.251 2.209 2.168
7 2.739 2.677 2.619 2.562 2.508 2.455 2.404 2.355 2.308 2.263
8 2.854 2.786 2.721 2.658 2.598 2.540 2.485 2.432 2.380 2.331
9 2.942 2.868 2.798 2.730 2.665 2.603 2.544 2.487 2.432 2.379
10 3.009 2.930 2.855 2.784 2.715 2.649 2.587 2.527 2.469 2.414
11 3.060 2.978 2.899 2.824 2.752 2.683 2.618 2.555 2.496 2.438
12 3.100 3.013 2.931 2.853 2.779 2.708 2.641 2.576 2.515 2.456
13 3.129 3.040 2.956 2.876 2.799 2.727 2.658 2.592 2.529 2.469
14 3.152 3.061 2.974 2.892 2.814 2.740 2.670 2.603 2.539 2.478
15 3.170 3.076 2.988 2.905 2.825 2.750 2.679 2.611 2.546 2.484
16 3.183 3.088 2.999 2.914 2.834 2.757 2.685 2.616 2.551 2.489
17 3.193 3.097 3.007 2.921 2.840 2.763 2.690 2.621 2.555 2.492
18 3.201 3.104 3.012 2.926 2.844 2.767 2.693 2.624 2.557 2.494
19 3.207 3.109 3.017 2.930 2.848 2.770 2.696 2.626 2.559 2.496
20 3.211 3.113 3.020 2.933 2.850 2.772 2.698 2.627 2.561 2.497
21 3.215 3.116 3.023 2.935 2.852 2.773 2.699 2.629 2.562 2.498
22 3.217 3.118 3.025 2.936 2.853 2.775 2.700 2.629 2.562 2.498
23 3.219 3.120 3.026 2.938 2.854 2.775 2.701 2.630 2.563 2.499
24 3.221 3.121 3.027 2.939 2.855 2.776 2.701 2.630 2.563 2.499
25 3.222 3.122 3.028 2.939 2.856 2.777 2.702 2.631 2.563 2.499
26 3.223 3.123 3.028 2.940 2.856 2.777 2.702 2.631 2.564 2.500
27 3.224 3.123 3.029 2.940 2.856 2.777 2.702 2.631 2.564 2.500
28 3.224 3.124 3.029 2.940 2.857 2.777 2.702 2.631 2.564 2.500
29 3.225 3.124 3.030 2.941 2.857 2.777 2.702 2.631 2.564 2.500
30 3.225 3.124 3.030 2.941 2.857 2.778 2.702 2.631 2.564 2.500
1
1−
(1 + 𝑟)𝑛
𝑃𝑉𝐴𝐹𝑟,𝑛 = [ ]
𝑟

Chapter 5 and 6, Capital Budgeting: 90


Annexure 3: Table A-1, The Compound Value of ₹1 or Future
Value Interest Factor at 𝒓 after n periods i.e. Future Value
Interest Factors (PVIFr, n)
₹1 invested at the beginning of the time period
N 1% 2% 3% 4% 5% 6% 7% 8% 9% 10%
1 1.010 1.020 1.030 1.040 1.050 1.060 1.070 1.080 1.090 1.100
2 1.020 1.040 1.061 1.082 1.103 1.124 1.145 1.166 1.188 1.210
3 1.030 1.061 1.093 1.125 1.158 1.191 1.225 1.260 1.295 1.331
4 1.041 1.082 1.126 1.170 1.216 1.262 1.311 1.360 1.412 1.464
5 1.051 1.104 1.159 1.217 1.276 1.338 1.403 1.469 1.539 1.611
6 1.062 1.126 1.194 1.265 1.340 1.419 1.501 1.587 1.677 1.772
7 1.072 1.149 1.230 1.316 1.407 1.504 1.606 1.714 1.828 1.949
8 1.083 1.172 1.267 1.369 1.477 1.594 1.718 1.851 1.993 2.144
9 1.094 1.195 1.305 1.423 1.551 1.689 1.838 1.999 2.172 2.358
10 1.105 1.219 1.344 1.480 1.629 1.791 1.967 2.159 2.367 2.594
11 1.116 1.243 1.384 1.539 1.710 1.898 2.105 2.332 2.580 2.853
12 1.127 1.268 1.426 1.601 1.796 2.012 2.252 2.518 2.813 3.138
13 1.138 1.294 1.469 1.665 1.886 2.133 2.410 2.720 3.066 3.452
14 1.149 1.319 1.513 1.732 1.980 2.261 2.579 2.937 3.342 3.797
15 1.161 1.346 1.558 1.801 2.079 2.397 2.759 3.172 3.642 4.177
16 1.173 1.373 1.605 1.873 2.183 2.540 2.952 3.426 3.970 4.595
17 1.184 1.400 1.653 1.948 2.292 2.693 3.159 3.700 4.328 5.054
18 1.196 1.428 1.702 2.026 2.407 2.854 3.380 3.996 4.717 5.560
19 1.208 1.457 1.754 2.107 2.527 3.026 3.617 4.316 5.142 6.116
20 1.220 1.486 1.806 2.191 2.653 3.207 3.870 4.661 5.604 6.727
21 1.232 1.516 1.860 2.279 2.786 3.400 4.141 5.034 6.109 7.400
22 1.245 1.546 1.916 2.370 2.925 3.604 4.430 5.437 6.659 8.140
23 1.257 1.577 1.974 2.465 3.072 3.820 4.741 5.871 7.258 8.954
24 1.270 1.608 2.033 2.563 3.225 4.049 5.072 6.341 7.911 9.850
25 1.282 1.641 2.094 2.666 3.386 4.292 5.427 6.848 8.623 10.835
26 1.295 1.673 2.157 2.772 3.556 4.549 5.807 7.396 9.399 11.918
27 1.308 1.707 2.221 2.883 3.733 4.822 6.214 7.988 10.245 13.110
28 1.321 1.741 2.288 2.999 3.920 5.112 6.649 8.627 11.167 14.421
29 1.335 1.776 2.357 3.119 4.116 5.418 7.114 9.317 12.172 15.863
30 1.348 1.811 2.427 3.243 4.322 5.743 7.612 10.063 13.268 17.449
𝑃𝑉𝐼𝐹𝑟,𝑛 = (1 + 𝑟)𝑛

Chapter 5 and 6, Capital Budgeting: 91


Annexure 3: Table A-1, The Compound Value of ₹1 or Future
Value Interest Factor at 𝒓 after n periods i.e. Future Value
Interest Factors (PVIFr, n)
₹1 invested at the beginning of the time period
n 11% 12% 13% 14% 15% 16% 17% 18% 19% 20%
1 1.110 1.120 1.130 1.140 1.150 1.160 1.170 1.180 1.190 1.200
2 1.232 1.254 1.277 1.300 1.323 1.346 1.369 1.392 1.416 1.440
3 1.368 1.405 1.443 1.482 1.521 1.561 1.602 1.643 1.685 1.728
4 1.518 1.574 1.630 1.689 1.749 1.811 1.874 1.939 2.005 2.074
5 1.685 1.762 1.842 1.925 2.011 2.100 2.192 2.288 2.386 2.488
6 1.870 1.974 2.082 2.195 2.313 2.436 2.565 2.700 2.840 2.986
7 2.076 2.211 2.353 2.502 2.660 2.826 3.001 3.185 3.379 3.583
8 2.305 2.476 2.658 2.853 3.059 3.278 3.511 3.759 4.021 4.300
9 2.558 2.773 3.004 3.252 3.518 3.803 4.108 4.435 4.785 5.160
10 2.839 3.106 3.395 3.707 4.046 4.411 4.807 5.234 5.695 6.192
11 3.152 3.479 3.836 4.226 4.652 5.117 5.624 6.176 6.777 7.430
12 3.498 3.896 4.335 4.818 5.350 5.936 6.580 7.288 8.064 8.916
13 3.883 4.363 4.898 5.492 6.153 6.886 7.699 8.599 9.596 10.699
14 4.310 4.887 5.535 6.261 7.076 7.988 9.007 10.147 11.420 12.839
15 4.785 5.474 6.254 7.138 8.137 9.266 10.539 11.974 13.590 15.407
16 5.311 6.130 7.067 8.137 9.358 10.748 12.330 14.129 16.172 18.488
17 5.895 6.866 7.986 9.276 10.761 12.468 14.426 16.672 19.244 22.186
18 6.544 7.690 9.024 10.575 12.375 14.463 16.879 19.673 22.901 26.623
19 7.263 8.613 10.197 12.056 14.232 16.777 19.748 23.214 27.252 31.948
20 8.062 9.646 11.523 13.743 16.367 19.461 23.106 27.393 32.429 38.338
21 8.949 10.804 13.021 15.668 18.822 22.574 27.034 32.324 38.591 46.005
22 9.934 12.100 14.714 17.861 21.645 26.186 31.629 38.142 45.923 55.206
23 11.026 13.552 16.627 20.362 24.891 30.376 37.006 45.008 54.649 66.247
24 12.239 15.179 18.788 23.212 28.625 35.236 43.297 53.109 65.032 79.497
25 13.585 17.000 21.231 26.462 32.919 40.874 50.658 62.669 77.388 95.396
26 15.080 19.040 23.991 30.167 37.857 47.414 59.270 73.949 92.092 114.475
27 16.739 21.325 27.109 34.390 43.535 55.000 69.345 87.260 109.589 137.371
28 18.580 23.884 30.633 39.204 50.066 63.800 81.134 102.967 130.411 164.845
29 20.624 26.750 34.616 44.693 57.575 74.009 94.927 121.501 155.189 197.814
30 22.892 29.960 39.116 50.950 66.212 85.850 111.065 143.371 184.675 237.376
𝑃𝑉𝐼𝐹𝑟,𝑛 = (1 + 𝑟)𝑛

Chapter 5 and 6, Capital Budgeting: 92


Annexure 3: Table A-1, The Compound Value of ₹1 or Future
Value Interest Factor at 𝒓 after n periods i.e. Future Value
Interest Factors (PVIFr, n)
₹1 invested at the beginning of the time period
n 21% 22% 23% 24% 25% 26% 27% 28% 29% 30%
1 1.210 1.220 1.230 1.240 1.250 1.260 1.270 1.280 1.290 1.300
2 1.464 1.488 1.513 1.538 1.563 1.588 1.613 1.638 1.664 1.690
3 1.772 1.816 1.861 1.907 1.953 2.000 2.048 2.097 2.147 2.197
4 2.144 2.215 2.289 2.364 2.441 2.520 2.601 2.684 2.769 2.856
5 2.594 2.703 2.815 2.932 3.052 3.176 3.304 3.436 3.572 3.713
6 3.138 3.297 3.463 3.635 3.815 4.002 4.196 4.398 4.608 4.827
7 3.797 4.023 4.259 4.508 4.768 5.042 5.329 5.629 5.945 6.275
8 4.595 4.908 5.239 5.590 5.960 6.353 6.768 7.206 7.669 8.157
9 5.560 5.987 6.444 6.931 7.451 8.005 8.595 9.223 9.893 10.604
10 6.727 7.305 7.926 8.594 9.313 10.086 10.915 11.806 12.761 13.786
11 8.140 8.912 9.749 10.657 11.642 12.708 13.862 15.112 16.462 17.922
12 9.850 10.872 11.991 13.215 14.552 16.012 17.605 19.343 21.236 23.298
13 11.918 13.264 14.749 16.386 18.190 20.175 22.359 24.759 27.395 30.288
14 14.421 16.182 18.141 20.319 22.737 25.421 28.396 31.691 35.339 39.374
15 17.449 19.742 22.314 25.196 28.422 32.030 36.062 40.565 45.587 51.186
16 21.114 24.086 27.446 31.243 35.527 40.358 45.799 51.923 58.808 66.542
17 25.548 29.384 33.759 38.741 44.409 50.851 58.165 66.461 75.862 86.504
18 30.913 35.849 41.523 48.039 55.511 64.072 73.870 85.071 97.862 112.455
19 37.404 43.736 51.074 59.568 69.389 80.731 93.815 108.890 126.242 146.192
20 45.259 53.358 62.821 73.864 86.736 101.721 119.145 139.380 162.852 190.050
21 54.764 65.096 77.269 91.592 108.420 128.169 151.314 178.406 210.080 247.065
22 66.264 79.418 95.041 113.574 135.525 161.492 192.168 228.360 271.003 321.184
23 80.180 96.889 116.901 140.831 169.407 203.480 244.054 292.300 349.593 417.539
24 97.017 118.205 143.788 174.631 211.758 256.385 309.948 374.144 450.976 542.801
25 117.391 144.210 176.859 216.542 264.698 323.045 393.634 478.905 581.759 705.641
26 142.043 175.936 217.537 268.512 330.872 407.037 499.916 612.998 750.468 917.333
27 171.872 214.642 267.570 332.955 413.590 512.867 634.893 784.638 968.104 1192.533
28 207.965 261.864 329.112 412.864 516.988 646.212 806.314 1004.336 1248.855 1550.293
29 251.638 319.474 404.807 511.952 646.235 814.228 1024.019 1285.550 1611.022 2015.381
30 304.482 389.758 497.913 634.820 807.794 1025.927 1300.504 1645.505 2078.219 2619.996
𝑛
𝑃𝑉𝐼𝐹𝑟,𝑛 = (1 + 𝑟)

Chapter 5 and 6, Capital Budgeting: 93


Annexure 3: Table A-1, The Compound Value of ₹1 or Future
Value Interest Factor at 𝒓 after n periods i.e. Future Value
Interest Factors (PVIFr, n)
₹1 invested at the beginning of the time period
n 31% 32% 33% 34% 35% 36% 37% 38% 39% 40%
1 1.310 1.320 1.330 1.340 1.350 1.360 1.370 1.380 1.390 1.400

2 1.716 1.742 1.769 1.796 1.823 1.850 1.877 1.904 1.932 1.960

3 2.248 2.300 2.353 2.406 2.460 2.515 2.571 2.628 2.686 2.744

4 2.945 3.036 3.129 3.224 3.322 3.421 3.523 3.627 3.733 3.842

5 3.858 4.007 4.162 4.320 4.484 4.653 4.826 5.005 5.189 5.378

6 5.054 5.290 5.535 5.789 6.053 6.328 6.612 6.907 7.213 7.530

7 6.621 6.983 7.361 7.758 8.172 8.605 9.058 9.531 10.025 10.541

8 8.673 9.217 9.791 10.395 11.032 11.703 12.410 13.153 13.935 14.758

9 11.362 12.166 13.022 13.930 14.894 15.917 17.001 18.151 19.370 20.661

10 14.884 16.060 17.319 18.666 20.107 21.647 23.292 25.049 26.925 28.925

11 19.498 21.199 23.034 25.012 27.144 29.439 31.910 34.568 37.425 40.496

12 25.542 27.983 30.635 33.516 36.644 40.037 43.717 47.703 52.021 56.694

13 33.460 36.937 40.745 44.912 49.470 54.451 59.892 65.831 72.309 79.371

14 43.833 48.757 54.190 60.182 66.784 74.053 82.052 90.846 100.510 111.120

15 57.421 64.359 72.073 80.644 90.158 100.713 112.411 125.368 139.708 155.568

16 75.221 84.954 95.858 108.063 121.714 136.969 154.003 173.008 194.194 217.795

17 98.540 112.139 127.491 144.804 164.314 186.278 210.984 238.751 269.930 304.913

18 129.087 148.024 169.562 194.038 221.824 253.338 289.048 329.476 375.203 426.879

19 169.104 195.391 225.518 260.011 299.462 344.540 395.996 454.677 521.532 597.630

20 221.527 257.916 299.939 348.414 404.274 468.574 542.514 627.454 724.930 836.683

21 290.200 340.449 398.919 466.875 545.769 637.261 743.245 865.886 1007.653 1171.356

22 380.162 449.393 530.562 625.613 736.789 866.674 1018.245 1194.923 1400.637 1639.898

23 498.012 593.199 705.647 838.321 994.665 1178.677 1394.996 1648.994 1946.885 2295.857

24 652.396 783.023 938.511 1123.350 1342.797 1603.001 1911.145 2275.611 2706.171 3214.200

25 854.638 1033.590 1248.220 1505.289 1812.776 2180.081 2618.268 3140.344 3761.577 4499.880

26 1119.576 1364.339 1660.132 2017.088 2447.248 2964.911 3587.028 4333.674 5228.593 6299.831

27 1466.645 1800.927 2207.976 2702.897 3303.785 4032.279 4914.228 5980.470 7267.744 8819.764

28 1921.305 2377.224 2936.608 3621.882 4460.109 5483.899 6732.493 8253.049 10102.164 12347.670

29 2516.909 3137.935 3905.688 4853.323 6021.148 7458.102 9223.515 11389.208 14042.008 17286.737

30 3297.151 4142.075 5194.566 6503.452 8128.550 10143.019 12636.215 15717.106 19518.391 24201.432
𝑛
𝑃𝑉𝐼𝐹𝑟,𝑛 = (1 + 𝑟)

Chapter 5 and 6, Capital Budgeting: 94

You might also like